Section 4 Deck Updated (407)

Ace your homework & exams now with Quizwiz!

A parent tells a nurse that her toddler drinks a quart of milk a day and has a poor appetite for solid foods. The nurse should explain that the toddler is at risk for which of the following disorders? a. Iron deficiency anemia b. Rickets c. Diabetes mellitus d. Obesity

Iron deficiency anemia Children between the ages of 12 and 36 months are at risk for iron deficiency anemia when cow's milk, which is poor in iron, is a major component of the diet.

A nurse observes a client's spouse sitting alone in the waiting room crying. When approached, the spouse says, "I am really concerned about my husband." Which of the following is a therapeutic nursing response? a)Your husband is making really good progress b)crying helps us let things out and we feel better c)Did your husband say something to upset you? d)Tell me what is concerning you

Tell me what is concerning you

A nurse is teaching a client about foods that are included on a clear liquid diet. Which of the following food choices made by the client indicates the need for further teaching? a)yogurt b)popsicle c)gelatin d)broth

yogurt

Nurse is assessing a client for physical problems that affect food intake. Which factor should the nurse consider? Select all that apply A- use of supplements B- trouble swallowing C- problems with dentition D- Financial Resources E- lactose intolerance

Trouble Swallowing & Problems with Dentition

A nurse is instructing the mother of a toddler who has iron-deficiency anemia to increase iron in the child's diet in addition to the prescribed iron supplement. Which of the following foods should the nurse recommend? a. Skim milk b. Bananas c. Tuna fish d. Cucumbers

Tuna fish Good sources of iron that are more readily absorbed than plant sources include seafood, meat, and eggs.

A nurse is teaching a client about foods that are included on a clear liquid diet. Which of the following food choices made by the client indicates the need for further teaching? a. Yogurt b. Popsicle c. Gelatin d. Broth

Yogurt Yogurt is allowed on a full liquid diet, not a clear liquid diet.

A nurse is teaching a client about modifiable risk factors to their health. Which of the following should the nurse include as an example of a modifiable risk factor? a. Tobacco use b. Age c. Family history d. Race

a

A nurse teaches clients about the benefits of a healthy diet and regular exercise to achieve weight loss. Which of the following topics is the nurse teaching to the client? a. Health promotion b. Disease prevention c. Nonmodifiable risk factors d. Tertiary prevention

a

A nurse is admitting a client who has heart failure. Which of the following tools should the nurse use to implement nursing interventions for the client? a) Standard of care b) A PICOT question c) Non-peer-reviewed sources d) Critical appraisal

a) Standard of care - Standard of care is a tool used by nurses to guide the minimal level of care for clients. It is used to assist nurses in planning consistent, high-quality care and interventions for clients who have similar disorders.

A nurse is preparing to perform a procedure on a client. Which of the following resources should the nurse use to meet the standard of care? (Select all that apply.) a) The health care organization policy and procedure manual b) Information from a government website c) Article published by a peer-reviewed journal d) An open-source article paid for by the researcher e) Information on a website of a company selling the product

a) The health care organization policy and procedure manual b) Information from a governmental website c) Article published by a peer-reviewed journal - The nurse should meet the standard of care by using credible evidence such as the health care organization policy and procedure manual, information from credible government websites, and peer-reviewed applicable literature.

The nurse assisting in the admission of a client with diabetic ketoacidosis will anticipate the physician ordering which of the following types of intravenous solution if the client cannot take any fluids orally? " a. 0.45% normal saline solution b. Lactated Ringer's solution c. 0.9 normal saline solution d. 5% dextrose in water (D5W)"

a. 0.45% normal saline solution Helps to hydrate patient and keep electrolyte levels balanced

A nurse in the emergency department is caring for a client who has extensive partial and full thickness burns of the head, neck, and chest. While planning the client's care, the nurse should identify which of the following risks as the priority for assessment and intervention? a)airway obstruction b)infection c)fluid imbalance d)paralytic ileus

airway obstruction

A nurse at a clinic is providing free blood pressure screenings for clients. Which of the following levels of health prevention is the nurse demonstrating? a. Tertiary prevention b. Secondary prevention c. Primary prevention d. Quaternary care

b

A nurse is preparing to administer an influenza vaccine to a client. The client states that they understand being immunized will help protect them against the influenza virus. Which of the following concepts is the nurse demonstrating by administering the vaccine? a. Health promotion b. Disease prevention c. Health outcomes d. Wellness

b

A nurse is teaching a client who has a new prescription for antihypertensive medication. The nurse should identify that antihypertensive medications are used for which of the following types of prevention? a. Secondary b. Tertiary c. Primary d. Quaternary

b

A nurse is caring for a client who has type one diabetes mellitus and reports feeling anxious and having palpitations. The glucometer reads 50 mg/dL. Which of the following actions should the nurse take? a. give the client 1 tsp of honey b. give the client 4 oz of apple juice c. give the client 4 oz of skim milk d. give the client one or to glucose tablets

b After confirming hypoglycemia, the nurse should give the client 15 to 20 g of a rapid-acting, concentrated carbohydrate source, such as 4 to 6 oz of fruit juice, 8 oz of skim milk, 1 tbsp of honey, or commercially prepared glucose tablets per package instructions.

A nurse is reviewing he results of routine lab test performed as a part of a client's annual physical examination. Which of the following values indicates a fasting blood glucose measurement that is outside of the expected reference range? a. 78 mg/dL b. 118 mg/dL c. 85 mg/dL d. 104 mg/dL

b This result exceeds the expected reference range for a fasting blood glucose measurement, which is generally between 74 and 106 mg/dL.

A nurse is using evidence-based research to care for a client who has diabetes mellitus. Which of the following resources should the nurse use to manage the delivery of care? a) An article published in a non-peer-reviewed journal b) Clinical Practice Guidelines c) An insulin manufacturer website d) A podcast about living with diabetes mellitus

b) Clinical Practice Guidelines - Clinical practice guidelines are evidence-based recommendations that are used as guidelines for the diagnosis and treatment of diseases.

"Polydipsia and polyuria related to diabetes mellitus are primarily due to: a. The release of ketones from cells during fat metabolism b. Fluid shifts resulting from the osmotic effect of hyperglycemia c. Damage to the kidneys from exposure to high levels of glucose d. Changes in RBCs resulting from attachment of excessive glucose to hemoglobin"

b. Fluid shifts resulting from the osmotic effect of hyperglycemia Rationale: The osmotic effect of glucose produces the manifestations of polydipsia and polyuria.

A nurse is discussing the Healthy People initiative with a newly licensed nurse. Which of the following information should nurse include? a. The program focuses on decreasing the occurrence of cancer in people. b. The program focuses upon issues related to global health. c. The program focuses on providing goals and data for improved public health. d. The program focuses on reducing the viruses acquired by people.

c

A nurse is teaching a group of guardians about primary prevention techniques. Which of the following topics should the nurse include as an example of primary prevention? a. Emphasizing the importance of well-child visits for the child b. Encouraging children who have asthma to participate in extracurricular activities c. Taking measures to decrease the risk of childhood injuries within the home d. Promoting a healthy lifestyle for children who are overweight

c

A nurse is teaching a newly licensed nurse about the purpose of peer review in scholarly publications. Which of the following statements should the nurse make? a) "This process relies on the internet to verify the accuracy of the information. b) "This process involves financial reimbursement that can bias the results." c) "This process ensures scientific quality and rigor." d) "This process ensures evidence can be translated to client care situations."

c) "This process ensures scientific quality and rigor." - Peer review allows colleagues with expertise in the subject matter and the research methodology the opportunity to review a journal article's quality and rigor and deem it worthy of publication.

A nurse is teaching a client who has cholecystitis about required dietary modifications. The nurse should include which of the following foods as appropriate for the client's diet? a)creamed chicken b)roast turkey c)ice cream d)Macaroni and cheese

roast turkey

A client with type 1 diabetes mellitus is being taught to monitor her blood glucose level. Which factor affecting accurate glucose monitoring should the nurse include in the​ instruction? (Select all that​ apply.) A. Medication overdoses B. White blood cell​ (WBC) count C.Low hematocrit level D. Creatinine level E. High hematocrit level

​A,C,E Rationale: Factors that affect accurate glucose monitoring include medication​ overdoses, a low hematocrit​ level, and a high hematocrit level. The WBC count and creatinine levels do not affect accurate glucose monitoring.

A nurse is teaching a client about foods that are included on a clear liquid diet. Which of the following food choices made by the client indicates the need for further teaching? A) Yogurt B) Popsicle C) Gelatin D) Broth

A) Yogurt

The nurse is presenting to a community group about mental disorders that are more common among women than among men. Which disorder should the nurse include? SATA a. anxiety disorder b. phobia c. obsessive compulsive disorder d. generalized anxiety disorder e. insomnia

A, B, D

Which tool is available to the nurse to assess a client's eating habits? Select all that apply. A- Food diary B- food frequency questionnaire C- waist-to-height ratio D- body mass index E- six-item mini-nutritional assessment

A, B, E

The nurse is planning to collect anthropometric data from a client who is being evaluated for undernutrition. Which information should the nurse include? Select all that apply A-Height B- food allergies C- skinfold thickness D- capillary blood glucose level E- weight

A, C, E

A client with DKA is being treated in the ED. What would the nurse suspect? 1. Comatose state 2. Decreased Urine Output 3. Increased respirations and an increase in pH. 4. Elevated blood glucose level and low plasma bicarbonate level.

Correct Answer: 4 Rationale: In DKA the arteriole pH is lower than 7.35, plasma bicarbonate is lower than 15 mEq/L, the blood glucose is higher than 250, and ketones are present in the blood and urine. The client would be experiencing polyuria and Kussmauls respirations would be present. A comatose state may occur if DKA is not treated.

"Which of the following factors are risks for the development of diabetes mellitus? (Select all that apply.) "a) Age over 45 years b) Overweight with a waist/hip ratio >1 c) Having a consistent HDL level above 40 mg/dl d) Maintaining a sedentary lifestyle

Correct: a,b,d"Rationale: Aging results in reduced ability of beta cells to respond with insulin effectively. Overweight with waist/hip ratio increase is part of the metabolic syndrome of DM II. There is an increase in atherosclerosis with DM due to the metabolic syndrome and sedentary lifestyle.

A certified nurse midwife works a 12-hour night shift and is able to go into the "on-call room" and get some sleep if there is no one in labor. A patient calls while the nurse is in stage N3 of the sleep cycle. What happens after the nurse is awakened and tries to go back to sleep? A. The nurse begins the sleep cycle where they left off. B. The nurse begins at the next cycle, stage N4 of the sleep cycle. C. The nurse begins at the REM stage of the sleep cycle. D. The nurse begins the sleep cycle anew at stage NI sleep.

D

A mother has brought her 12 yr old daughter for therapy. The daughter witnessed a horrific motor vehicle crash in which several people, including children, were killed. The mother, who was with the child at the time, says "I don't understand why she still seems affected by the crash. I'm not. Which statement by the nurse is most appropriate to help the mother better understand the daughters situation? A. A young persons negative experience can be erased by piling on positive events. B. A young persons coping responses are part of the skills that they are born with. C. A young persons memory bank is much more detailed than an adults. D. A young persons response can vary significantly from that of an adult

D

The nurse has completed an individual and home assessment for an older adult patient with diabetes and hypertension who is at risk for falling. After the assessments are completed, which activity by the nurse would be considered primary prevention related to the patient's risk for falling? A. Assessment for presence of orthostatic hypotension B. Review of the results of the Mini- Mental State Exam (MMSE) C. Education concerning the need to monitor blood sugar and blood pressure routinely D. Discussion with the patient concerning removal of throw rugs and clutter

D

The prenatal nurse is completing an assessment on a client who is currently at 20 wks. of gestation. The client indicates that she is struggling with an overwhelming sense of anxiety and fears that she is becoming depressed. Prior to becoming pregnant, the client had been taking selective serotonin reuptake inhibitor (ssri) but stopped the medication due to the pregnancy . Which intervention should the nurse suggest initially to help the client deal with anxiety? A. Asking the healthcare provider to prescribe another (ssri) that has been found not to increase the risk of birth defects. B. encouraging the client to find a good support system that can help manage the anxiety. C. Asking the healthcare provider about resuming the ssri but a lower dose D. Recommending cognitive behavioral therapy

D

When considering the sleep needs of a teenager versus an adult, the nurse needs to remember which differences that can impact sleep needs? A. In adolescence, sleep patterns shift, requiring more hours of sleep, whereas for adults less sleep is needed. B. Adolescents have a tendency to go to bed earlier and wake early versus older adults, who go to sleep later and wake later. C. Normal biological patterns of adolescence result in the ability to be productive on less sleep, which mirrors that of adults. D. In adolescence, normal biological patterns shift to sleeping and waking up later, whereas adult sleep patterns vary.

D

When considering the sleep needs of a teenager versus an adult, the nurse needs to remember which differences that can impact sleep needs? A. Normal biological patterns of adolescence result in the ability to be productive on less sleep, which mirrors that of adults. B. In adolescence, sleep patterns shift, requiring more hours of sleep, whereas for adults less sleep is needed. C. Adolescents have a tendency to go to bed earlier and wake early versus older adults, who go to sleep later and wake later. D. In adolescence, normal biological patterns shift to sleeping and waking up later, whereas adult sleep patterns vary.

D

Which action by the nurse is considered primary prevention? A. Providing rehabilitation classes for a group of patients who have recently suffered a stroke B. Performing monthly wellness screenings for blood pressure and blood glucose levels C. Providing free testing for sexually transmitted infections in a college-based clinic D. Teaching removal of throw rugs and clutter to patients at risk for falling

D

Which statement demonstrates an understanding of Maslow's hierarchy of needs. When a client prioritizes a choice to react to a stressor? A. Coping with stressors is part of safety needs B. Everyone chooses to satisfy basic requirement's first C. Self esteem is the most important level of need D. Individuals might have their own priorities

D

an older adult client is asking the healthcare provider for a medication to help deal with the anxiety they are feeling due to the recent deaths of two very close friends. They indicate that they use a benzodiazepine when they were younger and it worked fairly well. Which response by the nurse should address the clients request? A. A benzodiazepine is a good choice when you are older, so I'll try to get that prescribed for you. B. thank you for letting me know, I will see if the provider wants to prescribe a benzodiazepine. C. What dose of medication did you take when you were younger and how long did you take the medication? D. benzodiazepines can cause a decrease in cognition, so we need to avoid the use of these medications as possible.

D

The nurse informs a 60yo man that his weight has increased by 10 lbs since his last check-up a year ago. Which factor should the nurse explain about caloric intake? A) "Adult men tend to gain weight around your age due to hormonal changes." B) "You should be eating the same number of calories that you have been throughout your adult life." C) "Adult men must make sure to get enough calories to promote healthy bone growth." D) "Adults tend to need fewer calories around age 50."

D) "Adults tend to need fewer calories around age 50." Rationale: Around age​ 50, the metabolic rates of most adults​ decrease, so calorie needs decrease in response to this. Adult men do not gain weight due to hormonal changes in their adulthood.​ Adolescents, not​ adults, need adequate calories for bone growth. An​ adult's calorie intake needs are reduced throughout adulthood and should not remain constant.

A home-health nurse is teaching a client about strategies to change their lifestyle to lose weight. Which statement made by the client indicates that the teaching has been successful? A) "I will eat a few large meals daily, and drink lots of water." B) "I will eat fewer foods that are high in carbs." C) "I will lift weights for 60-80 minutes." D) "I will walk for 30-40 minutes, 5 days per week."

D) "I will walk for 30-40 minutes, 5 days per week." Rationale: An aerobic exercise program of​ 30-40 minutes of exercise five or more days a week promotes weight loss while reducing adipose​ tissue, increasing lean body​ mass, and promoting​ long-term weight control. The client should not be taught to avoid particular food groups because those foods provide essential nutrients and vitamins important to maintain health. Lifting weights is not as effective for weight loss as aerobic​ exercise, unless done 3 or 4 times per week for​ 30-45 minutes. Obese clients should eat regular meals with small​ servings, not a few large meals.

The nurse is providing teaching to a female client about dietary modifications to promote weight loss. Which statement by the nurse is accurate? A) "Your diet should consist of 1200-1600 calories/day, with calorie consumption increasing toward the end of the day." B) "Your diet should consist of 750-1000 calories/day, with just one big meal and then intermittent snacking." C) "Your diet should simply cut 500 calories per day from your normal intake, with a special attention to eliminating all fats from your diet." D) "Your diet should consist of 1000-1200 calories/day and be low in fat, high in fiber, and include a variety of foods."

D) "Your diet should consist of 1000-1200 calories/day and be low in fat, high in fiber, and include a variety of foods." Rationale: Collaboration with a nutritionist helps clients to identify healthy foods that appeal to them and that can make up a diet plan to create a daily 500- to 1000-kcal deficit. Ideally, the recommended diet should be low in kilocalories and fat, contain adequate nutrients and minerals, and be high in dietary fiber. The client should eat regular meals with small servings. A gradual, slow weight loss of no more than 1-2 lb/week is recommended. For most individuals, this means a diet of 1000-1200 kcal/day for most women and 1200-1600 kcal/day for men. Fewer than 1200 kcal each day may lead to loss of lean tissue and nutritional deficiencies.

A nurse is assessing four clients for indications of general adaptation syndrome (GAS). Which of the following clients should the nurse monitorclosely for GAS? A) A 68-year-old client who has viral pneumonia. B) A 22-year-old client who has type 1 diabetes mellitus. C) A 59-year-old client who has Stage I Alzheimer's disease. D) A 40-year-old client who has ulcerative colitis

D) A 40-year-old client who has ulcerative colitis

A nurse is assisting with teaching a class about incident reports. The nurse should include that which of the following situations requires an incident report? A) A client vomits after receiving an oral medication B) A client refuses to take a medication C) A nurse administers an antibiotic to a client 25 min after the scheduled time D) A nurse administers the wrong medication to a client

D) A nurse administers the wrong medication to a client.

A nurse is assisting with teaching a class about types of research studies. The nurse should include that which of the following is an example of a randomized controlled trial (RCT)? A) A study examining environmental factors that lead to obesity. B) A study comparing a group of people who have diabetes mellitus with a group of people who do not have diabetes mellitus. C) A study examining the cause of falls in a long term care facility D) A study arbitrarily assigning people who smoke into either an experimental group or a control group to determine the effects of a new therapy to reduce smoking.

D) A study arbitrarily assigning people who smoke into either an experimental group or a control group to determine the effects of a new therapy to reduce smoking

A nurse is admitting a client who reports anorexia and is experiencing malnutrition. Which of the following findings should the nurse expect to be altered? A) Creatine kinase B) Troponin C) Total bilirubin D) Albumin

D) Albumin

The nurse is caring for an adult female client who is admitted to the hospital with a possible hip fracture. Following the admission assessment, the nurse determines that the client is obese. The client's BMI is 33.2 and her waist circumference is 90 cm. How should the nurse classify her obesity and associated disease risk? A) Class I, high B) Class II, very high C) Class III, extremely high D) Class I, very high

D) Class I, very high Rationale: A BMI of 33.2 would be in the range for Class I obesity (30.0-34.9), which is the same for men and women. However, although a 90 cm waist circumference for a man who is Class I obese would indicate high associated disease risk, for woman a 90 cm waist circumference is above the 88 cm threshold for women, and therefore her associated disease risk is very high. Her BMI would have to be higher to be classified as Class II or Class III.

A nurse is caring for a client who is prescribed bedrest. The plan of care indicates that the client should perform isometric exercises every 2 hr.Which of the following actions should the nurse take as directed by the plan of care? A) Ask the client to move her arms and legs while applying slight resistance. B) Move the client's limbs through their complete range of motion. C0 Have the client move each limb independently through its complete range of motion. D) Instruct the client to tighten muscle groups for a short period, and then relax.

D) Instruct the client to tighten muscle groups for a short period, and then relax.

The nurse is caring for a client newly diagnosed with type 2 diabetes mellitus. Prior to any teaching about​ medications, the client informs the​ nurse, "I cannot give myself any​ injections." How should the nurse​ respond? A.​"Insulin administration helps with better blood glucose​ management." B. "It is understandable to be upset about a new medical​ diagnosis." C. "Why do you think you will have to give yourself​ injections?" D.​"Type 2 diabetes mellitus can usually be managed with​ pills, diet, and​ exercise."

D."Type 2 diabetes mellitus can usually be managed with​ pills, diet, and​ exercise." Rationale: The nurse should reassure the client with type 2 diabetes mellitus that the disease can be managed with oral hypoglycemic​ medications, diet, and exercise and may not require insulin. The nurse would not ask why the client thinks he would have to administer insulin. This is not therapeutic communication and is not the best way to obtain information. It is understandable for the client to be​ upset, but this statement does not alleviate the fears or provide information. Insulin administration helps with blood glucose management in type 1 diabetes mellitus.

A nurse is caring for an older adult client who reports being stressed about their health status due to problems with short - term memory , slower reaction times when driving , and urinary frequency . The nurse should recognize that the client is experiencing which of the following types of stressors ? Developmental stressors Situational stressors Adventitious stressors Socioeconomic stressors

Developmental stressors Developmental or maturational stressors vary throughout the lifespan and occur as individuals move through the stages of life older adults may experience stressors related to health problems and changes in mobility and cognition. Situational stressors typically stem from personal family and work related issues. Adventitious stressors typically result from events such as floods, earthquakes, war, and physical assault.

A nurse is caring for a client who has delivered a healthy newborn . The client tells the nurse that while they are somewhat stressed about being new parent , they are thrilled by the birth of their child . The nurse should identify that the client is experiencing which of the following types of a stress ? Allostatic load Distress Eustress Fight - or - flight response

Eustress Positive stress or eustress is often associated with accomplishment or achievement and generally produces feelings of well-being inspiration and motivation Allostatic load refers to elevated or fluctuating endocrine or neural responses causing excessive wear and tear on the body organs

A nurse is caring for a young female adult client who reports weakness, fatigue, and heavy menstrual periods. The client has a hemoglobin level of 8 g/dL and a hematocrit level of 28 g/dL. The nurse suspect which of the following types of anemia? a. Folic acid deficiency anemia b. Pernicious anemia c. Iron-deficiency anemia d. Sickle cell anemia

Iron-deficiency anemia Iron-deficiency anemia results from poor gastrointestinal absorption of iron, a diet that is deficient in iron, or blood loss. The nurse should expect a client who has iron-deficiency anemia to have weakness, pallor, fatigue, reduced tolerance for activity, and cheilosis (ulcerations of the corners of the mouth).

A nurse is assessing a client who is working at home due to COVID - 19 restrictions . The client reports abdominal cramping and bloating with diarrhea and states , " I am completely stressed out from working at home . " The nurse should identify that the client is experiencing manifestations of which of the following stress - related conditions ? Irritable bowel syndrome Food poisoning Panic disorder Major depressive disorder

Irritable bowel syndrome IBS is a condition often triggered by stress and characterized by abdominal discomfort cramping bloating and diarrhea. Panic disorder is a type of anxiety disorder characterized by sudden feelings of terror

Which electrolyte replacement should the nurse anticipate being ordered by thehealth-care provider in the client diagnosed with DKA who has just been admitted tothe ICD? 1.Glucose. 2.)Potassium. 3.Calcium. 4.Sodium

Potassium"1.Glucose is elevated in DKA; therefore, theHCP would not be replacing glucose. 2.(CORRECT)-->The client in DKA loses potassium from increased urinary output, acidosis, cata-bolic state, and vomiting. Replacement isessential for preventing cardiac dysrhyth-mias secondary to hypokalemia. 3.Calcium is not affected in the client with DKA.4.The IV that is prescribed 0.9% normal salinehas sodium, but it is not specifically orderedfor sodium replacement. This is an isotonicsolution. TEST-TAKING HINT: Option "1" should be elim-inated because the problem with DKA iselevated glucose so the HCP would not bereplacing it. The test taker should use physiol-ogy knowledge and realize potassium is in thecell."

A nurse is providing discharge teaching about health promotion to a client who has a new diagnosis of type 2 diabetes mellitus . Which of the following instructions should the nurse include ? ( Select all that apply ) . Practice mindful breathing . Start each day with a to - do list . Include simple carbohydrates in the diet . Develop habits to mitigate stress . Preserve energy by reducing physical activity .

Practice mindful breathing . Start each day with a to - do list . Develop habits to mitigate stress . Complex carbohydrates, not simple

A nurse is instructing a group of clients about nutrition. The nurse should include that which of the following foods is a good source of high-quality protein? a. Soybeans b. Grains c. Legumes d. Green vegetables

Soybeans The nurse should instruct that soybeans and soybean products are high-quality, or complete, sources of proteins. Complete proteins contain all nine essential amino acids required for growth and maintenance of the body.

A nurse is providing teaching for a client who has anemia and a new prescription for ferrous sulfate liquid. Which of the following instructions should the nurse provide? a. Take the medication on an empty stomach to decrease gastrointestinal irritation. b. Take the medication with orange juice to enhance absorption. c. Take the medication with milk. d. Rinse the mouth before taking the iron.

Take the medication with orange juice to enhance absorption. Ascorbic acid (vitamin C), which is found in orange juice, will enhance the absorption of iron and increase its bioavailability. This will also help to decrease the gastrointestinal side effects of iron.

A nurse is providing teaching about dietary recommendations to a client who has iron deficiency anemia. Which of the following dietary recommendations should the nurse include as a food that enhances iron absorption when consumed with nonheme iron? a. Tomato juice b. Tea c. Milk d. Dried beans

Tomato juice Food sources rich in Vitamin C enhance nonheme iron absorption. Tomato products contain vitamin C; therefore, tomato juice is appropriate to include as a food that that enhances iron absorption when consumed with nonheme iron.

A nurse at a provider's office is providing teaching to a client who is taking chemotherapy and losing weight. Which of the following should the nurse recommend to increase calorie and protein intake? (Select all that apply.) a. Top fruits with yogurt. b. Add cream to soups. c. Use milk instead of water in recipes. d. Increase fluids during meals. e. Dip meats in eggs and bread crumbs before cooking.

Top fruits with yogurt is correct. Topping fruits with yogurt is an appropriate recommendation to increase the client's protein and calorie intake. Add cream to soups is correct. Adding cream to soups is an appropriate recommendation to increase the client's protein and calorie intake. Use milk instead of water in recipes is correct. Using milk instead of water in recipes is an appropriate recommendation to increase the client's protein and calorie intake. Dip meats in eggs and bread crumbs before cooking is correct. Dipping meats in eggs and bread crumbs before cooking is an appropriate recommendation to increase the client's protein and calorie intake.

A charge nurse is supervising a nurse providing care to several clients. For which of the following actions by the nurse should the charge nurse intervene? a) The nurse alters the steps in a procedure established in the clinical practice guidelines b) The nurse uses a governmental website to research data. c) The nurse provides clear liquids for a client who has a prescription for a regular diet as tolerated. d) The nurse encourages a client who has been immobile for 24 hours to cough and deep breathe.

a) A nurse alters the steps in a procedure established in the clinical practice guidelines. - The charge nurse should intervene if the nurse changes the steps in a procedure established in the clinical practice guidelines. Clinical practice guidelines are developed using evidence-based practice to provide safe client care.

Patients with type 1 diabetes mellitus may require which of the following changes to their daily routine during times of infection? a. no change b. less insulin c. more insulin d. oral diabetic agents"

answer C: during times of infection and illness diabetic patients may need even more insulin to compensate for increased blood glucose levels.

A nurse is caring for a client who requires cold applications with an ice bag to reduce swelling and pain of an ankle injury. Which of the following actions should the nurse take? a)apply the bag for 30 min at a time b)reapply the bag 30 in after removing it c)allow room for some air inside the bag d)place the bag directly on the skin

apply the bag for 30 min at a time

A client that has presented to the healthcare with symptoms of hypertension. The client tells the nurse that for the past year they have been dealing with legal ramifications of an accident while under the influence of alcohol. According to Selye's general adaption syndrome GAS. The client is which stage of adaption? a. Alarm b. Resistance c. Exhaustion d. homeostasis

b

A nurse is using evidence-based practice to reduce pain during venipunctures for adolescent clients by asking a PICOT question: "Does providing music to adolescent clients during venipunctures decrease their discomfort compared to those clients who do not listen to music? Which of the following is a component of the "I" in this PICOT question? a) Adolescent clients b) Providing music c) Decrease discomfort d) Clients who do not listen to music

b) Providing music - Providing music refers to the intervention, or the "I" component of the PICOT question.

A nurse is teaching a client who was recently diagnosed with type one diabetes mellitus how to check blood glucose levels. Which of the following instructions should the nurse include in her teaching? 1. "Blood can be smeared from the fingertip onto the test strip." 2. "Use a syringe and needle to collect and transfer blood to the test strip." 3."To collect a sample for testing, hold the test strip next to the blood on the fingertip." 4. "Use a capillary tube to collect and transfer the blood from the fingertip."

c This allows the blood to flow over the reagent pad until the amount of blood on the strip is adequate. A sample that is too small can result in falsely low readings.

A nurse is providing an in-service about research methods for a group of nurses. Which of the following methods provides the most credible evidence to support practice changes? a) A qualitative research study b) A randomized control study c) A systematic review summarizing findings of studies d) A presentation by an expert at the international conference

c) A systematic review summarizing findings of studies - According to evidence-based practice, the nurse should identify that a systematic review or meta-analysis, which summarizes findings of multiple peer-reviewed studies, provides the most credible evidence to support practice changes. A meta-analysis of research studies uses statistics to determine conclusions.

A nurse is using evidence-based practice to develop a PICOT question. Which of the following describes the "O" in PICOT? a) The plan of action intended to solve the problem described in the study b) The group of clients who are the focus of the study c) The desired result of the intervention to the problem d) The current way of addressing the problem

c) The desired result of the intervention to the problem - This describes "O," or outcome, which is the desired result or outcome following an intervention.

"The nurse is working with an overweight client who has a high-stress job and smokes. This client has just received a diagnosis of Type II Diabetes and has just been started on an oral hypoglycemic agent. Which of the following goals for the client which if met, would be most likely to lead to an improvement in insulin efficiency to the point the client would no longer require oral hypoglycemic agents? a. Comply with medication regimen 100% for 6 months b. Quit the use of any tobacco products by the end of three months c. Lose a pound a week until weight is in normal range for height and exercise 30 minutes daily d. Practice relaxation techniques for at least five minutes five times a day for at least five months"

c. Lose a pound a week until weight is in normal range for height and exercise 30 minutes daily When type II diabetics lose weight through diet and exercise they sometimes have an improvement in insulin efficiency sufficient to the degree they no longer require oral hypoglycemic agents.

A home health nurse is visiting a client who lives in an older home and is concerned about their child's exposure to lead paint in the house. The nurse should identify which of the following is a potential health risk from exposure to lead paint? a. Strabismus b. Dental caries c. Accelerated growth and development d. Learning disabilities

d

A nurse is caring for a client who reports an improved diet, exercising 30 min a day for 5 days a week, and an overall sense of improved health. The nurse should identify that the client is describing a positive state of health known as which of the following? a. Health promotion b. Disease prevention c. Health outcomes d. Wellness

d

A nurse is speaking to a client who smokes tobacco and has a child living in the home. The nurse should identify that the child's exposure to second-hand smoke is an example of the following risk factors? a. Cultural b. Societal c. Heredity d. Environmental

d

A nurse is teaching a client who has type one diabetes about the use of an insulin pump. Which of the following information should the nurse include in the teaching? a. the pump should remain in place while bathing b. insulin is injected intermittently based on the client's glucose level c. the pump uses intermediate-acting insulin d. the risk for developing DKA can be increased with the use of an insulin pump

d Malfunction of the pump from low battery power, occlusion of tubing or needles, or lack of insulin in the pump increases the risk of DKA, particularly if the client is not aware of it.

"An adolescent client with type I diabetes mellitus is admitted to the emergency department for treatment of diabetic ketoacidosis. Which assessment findings should the nurse expect to note? a) sweating and tremors b) hunger and hypertension c) cold, clammy skin and irritability d) fruity breath and decreasing level of consciousness

d) fruity breath and decreasing level of consciousness"Hyperglycemia occurs with diabetic ketoacidosis. Signs of hyperglycemia include fruity breath and a decreasing level of consciousness. Hunger can be a sign of hypoglycemia or hyperglycemia, but hypertension is not a sign of diabetic ketoacidosis. Instead, hypotension occurs because of a decrease in blood volume related to the dehydrated state that occurs during diabetic ketoacidosis. Cold, clammy skin, irritability, sweating, and tremors are all signs of hypoglycemia."

A nurse at a health fair is assessing the weight status of four clients. Which of the following clients are classified as overweight? a)female client with a BMI of 24 b)male client with BMI of 29 c)female client with waist circumference 101.6 cm d)male client with waist circumference 96.52 cm

male client with BMI of 29

A nurse is teaching a client who has a new prescription for colchicine to treat gout. Which of the following instructions should the nurse include? a)take the medication with food if nausea develops b)monitor for muscle pain c)expect to have increased bruising d)increase your intake of grapefruit juice

monitor for muscle pain

A nurse is caring for an older adult client who was alert and oriented at admission, but now seems increasingly restless and intermittently confused. Which of the following actions should the nurse take to address the client's safety needs? a)call the family and ask them to stay with the client b)move the client to a room closer to the nurses station c)apply wrist and leg restraints to the client d)administer medication to sedate the client

move the client to a room closer to the nurses station

A home health nurse is assessing an older adult client who reports falling a couple times over the past week. Which of the following findings should the nurse suspect is contributing to the client's falls? a)the client takes alprazolam b)the client has a nonslip bath mat in his shower c)the client uses a raised toilet seat d)the client wears fitted slippers

the client takes alprazolam

A nurse is caring for a client who has fallen while getting out of bed and states, "I'm okay! I guess I should have called for help to the bathroom." After assessing the client, the nurse notifies the provider. Which of the following documentations should the nurse include in the client's medical record? a)There were no injuries sustained b)An incident report was completed c)an incident report was forwarded to risk management d)the provider was notified

the provider was notified

The nurse is conducting discharge teaching with a client who has been newly diagnosed with type 1 diabetes mellitus. Which statement from the client indicates the need for additional​ teaching? A."It is important to test my blood sugar at least four times a​ day." B. "I need to stay hydrated during the​ day." C."As long as​ I'm in my​ house, I can walk​ barefoot." D. "I need to be alert for​ infections."

​ C. ​"As long as​ I'm in my​ house, I can walk​ barefoot." Rationale: Clients with diabetes should always wear shoes to protect their feet from injury. The client should be alert for infection or​ injuries, stay well​ hydrated, and test the blood sugar four times a day.

The nurse is delivering a presentation about​ evidence-based practice​ (EBP). Which statement best reflects the​ nurse's correct understanding of​ EBP? A."EBP incorporates clinical​ knowledge, expert​ opinion, or information resulting from​ research." B."EBP is the gathering of objective facts and information to advance knowledge about a specific​ topic." C. "EBP uses a systematic and strict scientific process to test hypotheses about​ health-related conditions and nursing​ care." D. "EBP involves combining quality​ research, clinical​ expertise, and client preferences to achieve the best client​ outcomes."

​"EBP involves combining quality​ research, clinical​ expertise, and client preferences to achieve the best client​ outcomes." Rationale: Evidence-based practice​ (EBP) seeks to achieve optimal client outcomes by combining the best evidence from the most current research​available, the​ nurse's clinical​ expertise, and the​ client's preferences, including​needs, values, and choices. Research involves gathering objective facts and information to advance knowledge about a specific topic. Nursing research is a systematic and strict scientific process that tests hypotheses about​ health-related conditions and nursing care. Evidence can be defined as clinical​ knowledge, expert​opinion, or information resulting from research.

A community health nurse is providing education to a group of adults about the types of exercise that promote health. Which statement will the nurse include in the​ teaching? (Select all that​ apply.) A. "The talk test may be easier to use than the​ heart-rate calculation for determining effort in aerobic​exercise." B. "Isotonic exercises like walking and swimming are also known as dynamic​ exercises." C. "Isometric exercise, or isolated muscle​ contraction, creates an increase in blood flow to all parts of the​body." D. "Anaerobic exercises are exercises where the amount of oxygen taken into the body during exercise is greater than that used to perform the​ activity." E. "An example of an anaerobic exercise is​ weightlifting."

​A. "The talk test may be easier to use than the​ heart-rate calculation for determining effort in aerobic​exercise." B. "Isotonic exercises like walking and swimming are also known as dynamic​ exercises." E. ​"An example of an anaerobic exercise is​ weightlifting." Rationale: An example of anaerobic exercise is weightlifting. Isotonic exercises are also known as dynamic exercises. The talk test may be easier to use than the​heart-rate calculation for determining effort in aerobic exercise. Aerobic​ exercises, not​ anaerobic, are exercises where the amount of oxygen taken into the body during exercise is greater than that used to perform the activity. Isometric exercise causes a slight increase in heart rate and cardiac​ output, but it causes no noticeable increase in blood flow to other parts of the body.

A client with blood glucose of 450​ mg/dL is diagnosed with hyperosmolar hyperglycemic state​ (HHS). Which assessment finding should the nurse​ expect? A. Increase in urinary output B. Open wound to the foot C. Lower extremity edema D. Capillary refill of 2 seconds

​A. Increase in urinary output Rationale: A client in HHS would have an increase in urinary output due to the hyperosmolarity of the blood. Capillary refill should be less than 3 seconds. Lower extremity edema occurs from fluid volume overload. A client in HHS has a fluid volume deficit. An open wound to the foot is not directly related to HHS because it can come from an injury to the foot.

Which information should the nurse provide the client with type 2 diabetes​ mellitus? A. Inspect your feet on a daily basis for open sores. B. Treat hyperglycemia with concentrated sweets. C. Include 100 minutes per week of activity and exercise. D. Increase carbohydrate consumption in the diet.

​A. Inspect your feet on a daily basis for open sores. Rationale: The client should inspect both feet every​ day, using a mirror if​ needed, to look for open sores.​ Hypoglycemia, not​ hyperglycemia, is treated with 15 grams of concentrated carbohydrates. The client should include 150 minutes of activity and exercise per week. The client should decrease carbohydrate consumption.

The nurse is developing a plan of care for a client with ineffective peripheral tissue perfusion related to microvascular changes. Which assessment finding supports this nursing​ diagnosis? A. Absent pedal pulses B. Capillary refill of 3 seconds C. Fasting blood glucose of 100​ mg/dL D. Hemoglobin A1C of​ 6.4%

​A.Absent pedal pulses Rationale: Absence of pedal pulses indicates the peripheral tissue is not receiving adequate oxygenation and in turn is the basis of the nursing diagnosis Tissue​ Perfusion: Peripheral, Ineffective. A hemoglobin A1C of​ 6.4%, capillary refill of 3​ seconds, and fasting blood glucose of 100​ mg/dL are all normal findings.​

The nurse is teaching a group of older adults with type 2 diabetes mellitus. Which complication of the disease should the nurse​ include? (Select all that​ apply.) A.Pulmonary disease B. Functional disabilities C. Polypharmacy D. Cognitive impairment E. Autoimmune diseases

​B,C,D Rationale: Older adults diagnosed with type 2 diabetes mellitus are at an increased risk of developing other complications as compared with younger clients. These include​ polypharmacy, or taking other medications that can increase the​ risk; functional disabilities that may lead to a​ slower, more sedentary​ lifestyle; and cognitive impairment. A diagnosis of type 2 diabetes mellitus does not place a client at higher risk of pulmonary or autoimmune disease.

The nurse is caring for a​ 15-year-old child newly diagnosed with type 2 diabetes mellitus. Which task should the nurse expect to be completed quarterly for this​ child? (Select all that​ apply.) A. Make a foot assessment. B. Review glucose records. C. Discuss​ alcohol, tobacco, and drug use. D. Refer for an eye exam. E. Measure fasting glucose levels.

​B,C,E Rationale: When a child is diagnosed with type 2 diabetes​ mellitus, certain tasks should be scheduled quarterly and annually. Discussing​ alcohol, tobacco, and drug​ use, measuring fasting glucose​ levels, and reviewing glucose records are completed quarterly. Referral for an eye exam and a foot assessment should be completed​ annually; quarterly would be too often.

The nurse preceptor is teaching a new graduate nurse about hypoglycemic agents used to treat type 2 diabetes mellitus. Which information should the preceptor include related to how these medications lower blood​ sugar? (Select all that​ apply.) A.Stimulate hormones for hemodilution B. Prevent breakdown of glycogen C. Increase breakdown of insulin D. Increase insulin secretion E. Increase uptake of glucose by cells

​B,D,E Rationale: Hypoglycemic agents are used to treat individuals with type 2 diabetes mellitus. These medications lower blood sugar by stimulating or increasing insulin​ secretion, preventing breakdown of glycogen to glucose by the​ liver, and increasing peripheral uptake of glucose by making cells less resistant to insulin. Peripheral uptake is uptake by muscles and fat in the arms and legs rather than in the trunk. Some hypoglycemic agents keep blood sugar low by blocking absorption of carbohydrates in the intestines. The most recent pharmacologic therapy in treating type 2 diabetes mellitus includes the incretin effect. Incretin​ hormones, which are hormones released from the gut endocrine cells during​ meals, play a significant role in insulin secretion.

A young client is admitted for lethargy and weight loss. Which clinical manifestation supports the​ nurse's suspicion of diagnosis of type 1 diabetes​ mellitus? (Select all that​ apply.) A.Fever B.Blurred vision C.Weight gain D. Glucosuria E. Polyuria

​B,D,E Rationale: Manifestations of type 1 diabetes mellitus are caused by the lack of insulin to transport glucose into the cells for energy. The resulting hyperglycemia leads to​ polyuria, glucosuria, and blurred vision. Polyuria occurs because water is drawn into the general​ circulation, increasing renal blood flow. Once the blood glucose exceeds the renal​ threshold, which is 180​ mg/dL, glucose will spill into the urine. Blurred vision is caused by swelling of the lenses of the eyes in response to increased fluid volume. Clients with type 1 diabetes mellitus usually lose​ weight, because proteins and fats are metabolized for energy and water is lost in the urine. In​ addition, clients with type 1 diabetes mellitus are frequently unable to develop a fever when cellular fuel stores are depleted because of a lack of insulin.

The nurse is evaluating the plan of care for an obese client diagnosed with type 2 diabetes mellitus 6 months prior. Which finding indicates the client is successfully managing the​ disease? A. Hemoglobin A1C of​ 10.0% B. Weight loss of 40 pounds C. New foot wound with purulent drainage D. Fasting blood sugars averaging 150​ mg/dL

​B. Weight loss of 40 pounds Rationale: The obese client demonstrating a​ 40-pound weight loss over the past 6 months indicates improvement in dietary compliance with lowering carbohydrate intake and exercising. The normal hemoglobin A1C for a client with diabetes mellitus is​ 6?6.5%. Fasting blood sugars should be less than 100​ mg/dL if the client has good control. A new foot wound with purulent drainage indicates an infection and poor​ circulation, so this does not show good glycemic control.

A nurse researcheris designing a study intended to explore how certification exam preparation courses affect nursing​ students' success rates in passing the exam. Which element best reflects the​ nurse's use of a randomized control trial​ (RCT) design? A.Compare graduate nurses who successfully passed the certification exam with graduate nurses who did not pass the exam in order to determine whether or not completing a certification exam preparation course was a variable that contributed to test performance. B. Compare the exam performance of a control group of graduate nurses who did not complete any certification exam preparation course with a group of graduate nurses who completed a specific certification exam preparation course prior to taking the exam. C. Interview five graduate nurses who completed a certification exam preparation course and detail their experiences with the course and then identify the number of graduate nurses who successfully completed the certification exam. D.Examine a group of studies on graduate nurses who completed certification exam preparation courses and then combine and analyze the results as if they were from one large study.

​B. Compare the exam performance of a control group of graduate nurses who did not complete any certification exam preparation course with a group of graduate nurses who completed a specific certification exam preparation course prior to taking the exam. Rationale: Randomized control trials​ (RCTs) are designed to illustrate a​cause-and-effect relationship by using a control group and an experimental group. An RCT is best illustrated by comparing the exam performance of a control group of graduate nurses who did not complete any certification exam preparation course with a group of graduate nurses who completed a specific certification exam preparation course prior to taking the exam. A​ meta-analysis examines a group of studies on a given topic and combines and analyzes the results as if they were from one large​ study; this design is best reflected by examining a group of studies on graduate nurses who completed certification exam preparation courses and then combining and analyzing the results as if they were from one large study. A case study is specific to one​ individual, issue, or​ event; this design is best reflected by interviewing five graduate nurses who completed a certification exam preparation course and detailing their experiences with the​ course, as well as identifying the number of graduate nurses who successfully complete the certification exam.​ Case-control studies compare individuals with and without a specific condition to identify predictive​ variables; this design is best reflected by comparing graduate nurses who successfully passed the certification exam with graduate nurses who did not pass the exam to determine whether or not completing a certification exam preparation course contributed to the success rates.

Which question is most appropriate for the nurse to consider when evaluating the impact of​evidence-based practice​ (EBP) on client​ outcomes? (Select all that​ apply.) A. Is the new practice beneficial to the​ nurse? B. Is the new practice being implemented​ correctly? C. Does the change in practice yield the intended​ results? D. Is the practice in accordance with the​ state's nurse practice​ act? E. Does the practice adhere to the American Nurses Association​ (ANA) standards of nursing​care?

​B. Is the new practice being implemented​ correctly? C. Does the change in practice yield the intended​ results? Rationale: Evaluation seeks to answer questions​ including, "Is the new practice being implemented​ correctly?" and​ "Does the change in practice yield the intended​results?" Nurses are legally bound to practice in accordance with their​ state's nurse practice act and to provide care that adheres to the ANA standards of nursing​ care; these concerns must be addressed before implementing care. The practice is intended to be beneficial to the client.

Which factor that inhibits​ evidence-based practice would be considered an external​ factor? A. Personal aversion to the idea B. Lack of support by management C. Resistance to change D. Thoughts that the process​ doesn't work

​B. Lack of support by management Rationale: Nurses need to assess the obstacles that inhibit them from using EBP more frequently. These may include internal​ factors, such as beliefs or​misconceptions, or external​ factors, such as lack of management support. Resistance to​ change, a personal aversion to the​ idea, and a thought that the process​ doesn't work are all examples of internal factors.

A teacher sends a child to the school nurse due to frequent thirst and urination. Upon​ assessment, the nurse suspects the child has type 1 diabetes mellitus. Which question should the nurse ask to gain data to support this​ suspicion? A."When did you last see your healthcare​ provider?" B."How is your​ appetite?" C."Do you play outside a​ lot?" D. "Have you noticed any bruises on your​ legs?"

​B."How is your​ appetite?" Rationale: Polydipsia,​ polyuria, and polyphagia are the three hallmark signs of type 1 diabetes mellitus.​ Therefore, the nurse would ask about the​ child's appetite. Playing outside is not related to the onset of type 1 diabetes mellitus. Asking when the child last saw the healthcare provider is irrelevant to the current situation. Bruising to the legs can be from injuries or​ leukemia, not type 1 diabetes mellitus.

The nurse is discussing some barriers that they have encountered with a journal club related to the topic. Which statement by the nurse describes potential​ barriers? (Select all that​ apply.) A. I am an​ administrator." B. "I work in a nursing​ home." C. "I don't have time to do those​ trainings." D. "I have been in nursing 20​ years; I do not need more​ training." E. "The hospital I work at stopped professional development based on​ funds."

​C. "I don't have time to do those​ trainings." D. "I have been in nursing 20​ years; I do not need more​ training." E. ​"The hospital I work at stopped professional development based on​ funds." Rationale: When a healthcare facility makes​ cuts, it has been seen that educational programs and trainings go by the wayside. The other barrier would be not making time for the staff to attend trainings that are offered. The​ overconfident, stuck-in-their-ways nurse is a difficult person to educate on change and an alternate way of thinking. Being an administrator or working in a nursing home is not a barrier to promoting​ evidence-based practice in a healthcare facility.

The nurse is caring for a child who is hospitalized for the treatment of diabetic ketoacidosis​ (DKA). The​ child's parents ask why their child is receiving potassium. Which response by the nurse is​ accurate? A. "Potassium is administered to treat​ acidosis." B ​"Potassium is administered to treat cerebral​ edema." C. "Potassium is administered to treat​ hypokalemia." D. "Potassium is administered to decrease blood glucose​ levels."

​C. "Potassium is administered to treat​ hypokalemia." Rationale: Potassium is administered to treat hypokalemia.​ Insulin, not​ potassium, is administered to decrease blood glucose levels. Sodium​ bicarbonate, not​ potassium, is administered to treat acidosis.​ Mannitol, not​ potassium, is administered to treat cerebral edema.

The nurse is caring for a child diagnosed with type 1 diabetes mellitus. The nurse should teach the child and parents that insulin dosing is based on which​ item? A. Weight B. Urine output C. Diet D. Age

​C. Diet Rationale: Insulin dose is based on​ diet, specifically carbohydrate intake. Insulin dose is not based on​ weight, age, or urine output.

The nurse is doing a case study on a client with​ long-term cardiovascular disease.Which would be the best way to target health promotion for treatment as it relates to​ evidence-based practice? A.Examining cultural views B.Following federal regulation C. Genomic standpoint D. Looking at the hospital data

​C. Genomic standpoint Rationale: Targeting health promotion from a genomic standpoint allows for more accurate risk​ prediction, diagnosis, and treatment of a variety of health​ alterations, such as heart​ disease, stroke,​ cancer, diabetes, and Alzheimer disease​ (HHS, 2016e). Goals associated with the promotion of global health include protecting the health of the national population as well as limiting the international transmission of infectious diseases during travel.​ Evidence-based resources related to global health and genomics are emerging.

The nurse is teaching the parents of a child with a new diagnosis of type 1 diabetes mellitus. Which information should the nurse include regarding the pathophysiology of the​ disease? A.Delta cell destruction causes type 1 diabetes mellitus. B.Hyperglycemia happens when​ 50% of alpha cells are damaged. C.Beta cells need help producing insulin. D. Beta cells are destroyed.

​D. Beta cells are destroyed. Rationale: Type 1 diabetes mellitus has a slow onset and symptoms are not evident until​ 80-90% of beta cells are​ destroyed, causing hyperglycemia. Beta cells are functional and need medication to help with insulin production in type 2 diabetes mellitus. Hyperglycemia happens from beta cell​ destruction, not alpha or delta cell destruction.

A nurse is caring for a client who has delivered a healthy newborn . The client is tense , refuses to hold the baby , and tells the nurse , " I have no idea how to handle having a baby . I wish this pregnancy had never happened . " Which of the following statements should the nurse make ? " Becoming a parent is a new experience for you . Let's talk about your concerns. " "You should be thrilled about having a healthy newborn. I would be. " " Why don't you hold the baby ? I'm sure it will make you feel better. " " How can you think that way? This is a joyous occasion and should be celebrated . "

" Becoming a parent is a new experience for you . Let's talk about your concerns . " Individuals perceptions of and responses to the stressor determine whether the stressor is positive (eustress) or negative (distress). by using therapeutic communication skills the nurse creates a safe environment for the client to express thoughts and feelings.

A nurse is caring for a college student admitted for acute alcohol intoxication . The client reports feeling overwhelmed and expresses an inability to cope with stressors at school . Which of the following statements should the nurse make ? "Drinking too much alcohol is not the best choice . I suggest you stop drinking. " "I can see why you're using alcohol to cope : you've got a lot going on. " "Let's talk about the coping methods that have worked for you in the past. "I've been stressed before too, but I tell myself that I can handle it. "

" Let's talk about the coping methods that have worked for you in the past . Nurses using therapeutic communication techniques and building on the clients strengths to improve coping skills. nurses use several therapeutic communication techniques to build a trusting relationship including active listening, asking open ended questions, seeking clarification, and offering observations.

A nurse is caring for a client who has migraine headaches and reports that they are " getting worse . " Which of the following questions should the nurse ask the client to determine if the headaches are a stress - related disorder ? ( Select all that apply . ) " What is the intensity of your migraine headaches ? " " How often do the migraine headaches occur ? " " Are you eligible for workers ' compensation due to the migraine headaches ? " " What type of support is available to you when you have a migraine headache ? " " What coping strategies do you use when you experience a migraine headache "

" What is the intensity of your migraine headaches ? " " How often do the migraine headaches occur ? " " What type of support is available to you when you have a migraine headache ? " " What coping strategies do you use when you experience a migraine headache " The onset and severity of stress related disorders is affected by the type, frequency, and intensity of the stressor, perception of the stressor, access to support systems, and the individuals ability to effectively cope with the stressor

A nurse is interviewing a client who is in distress and tells the nurse , " My ex - partner is suing for full custody of my children . I am so worried and don't know what to do . " Which of the following questions should the nurse ask to evaluate the client's coping skills ? " Can you describe your relationship with your ex - partner ? " " What happens when you feel worried like this ? " " What do you believe was your contribution to the relationship breakup ? " What strategies have you used in the past to deal with stress ? "

" What strategies have you used in the past to deal with stress ? " This question is appropriate because it focuses on investigating the clients ability to cope with and manage stressful situation

"The nurse is caring for a woman at 37 weeks gestation. The client was diagnosed with insulin-dependent diabetes mellitis (IDDM) at age 7. The client states, ""I am so thrilled that I will be breastfeeding my baby."" Which of the following responses by the nurse is best? 1. You will probably need less insulin while you are breastfeeding. 2. You will need to initially increase your insulin after the baby is born. 3. You will be able to take an oral hypoglycemic instead of insulin after the baby is born. 4. You will probably require the same dose of insulin that you are now taking."

"1. breastfeeding has an antidiabetogenic effect, less insulin is needed. (correct) 2. insulin needs will decrease due to antidiabetogenic effect of breastfeeding and physiological changes during immediate postpartum period. 3. client has IDDM, insulin required. 4. during third trimester insulin requirements increase due to increased insulin resistance"

"The client diagnosed with Type 1 diabetes has a glycosylated hemoglobin (A1c) of 8.1%. Which interpretation should the nurse make based on this result? 1.This result is below normal levels. 2.This result is within acceptable levels. 3.This result is above recommended levels 4.This result is dangerously high.

"1.The acceptable level for an A1c for a client with diabetes is between 6% and 7%, which corresponds to a 120-140 mg/dL average blood glucose level. 2.This result is not within acceptable levelsfor the client with diabetes, which is 6% to7%. 3.(CORRECT) This result parallels a serum blood glucoselevel of approximately 180 to 200 mg/dL. An A1 c is a blood test that reflects average blood glucose levels over a period of 2-3months; clients with elevated blood glucose levels are at risk for developing long-term complications. 4.An A1c of 13% is dangerously high; it reflects a 300-mg/dL average blood glucose level overthe past 3 months."

"An 18-year-old female client, 5'4'' tall, weighing 113 kg, comes to the clinic for a non-healing wound on her lower leg, which she has had for two weeks. Which disease process should the nurse suspect the client is developing? "A. Type 1 diabetes B. Type 2 diabetes C. Gestational diabetes D. Acanthosis nigricans"

"A: Type 1 diabetes usually occurs in young clients who are underweight. In this disease, there is no production of insulin from the beta cells in the pancreas. People with type 1 diabetes are insulin dependent with a rapid onset of symptoms, including polyuria, polydipsia, and polyphagia. CORRECT -->B. Type 2 diabetes is a disorder usually occurring around the age of 40, but it is now being detected in children and young adults as a result of obesity and sedentary lifestyles. Non-healing wounds are a hallmark sign of type 2 diabetes. This client weights 248.6 lbs and is short. C. Gestational diabetes occurs during pregnancy. There is no mention of this. D. Acanthosis nigricans (AN), dark pigmentation and skin creases in the neck, is a sign of hyperinsulinemia. The pancreas is secreting excess amounts of insulin as a result of excessive caloric intake. It is identified in young children and is a precursor to the development of type 2 diabetes."

The nurse is educating a pregnant client who has gestational diabetes. Which of the following statements should the nurse make to the client? Select all that apply. a. Cakes, candies, cookies, and regular soft drinks should be avoided. b. Gestational diabetes increases the risk that the mother will develop diabetes later in life. c. Gestational diabetes usually resolves after the baby is born. d. Insulin injections may be necessary. e. The baby will likely be born with diabetes f. The mother should strive to gain no more weight during the pregnancy.

"ANS: A, B, C, D Gestational diabetes can occur between the 16th and 28th week of pregnancy. If not responsive to diet and exercise, insulin injections may be necessary. Concentrated sugars should be avoided. Weight gain should continue, but not in excessive amounts. Usually, gestational diabetes disappears after the infant is born. However, diabetes can develop 5 to 10 years after the pregnancy"

During a diabetes screening program, a patient tells the nurse, "My mother died of complications of type 2 diabetes. Can I inherit diabetes?" The nurse explains that a.) as long as the patient maintains normal weight and exercises, type 2 diabetes can be prevented. b.) the patient is at a higher than normal risk for type 2 diabetes and should have periodic blood glucose level testing. c.) there is a greater risk for children developing type 2 diabetes when the father has type 2 diabetes. d.) although there is a tendency for children of people with type 2 diabetes to develop diabetes, the risk is higher for those with type 1 diabetes."

"B Rationale: Offspring of people with type 2 diabetes are at higher risk for developing type 2 diabetes. The risk can be decreased, but not prevented, by maintenance of normal weight and exercising. The risk for children of a person with type 1 diabetes to develop diabetes is higher when it is the father who has the disease. Offspring of people with type 2 diabetes are more likely to develop diabetes than offspring of those with type 1 diabetes."

Analyze the following diagnostic findings for your patient with type 2 diabetes. Which result will need further assessment? A) BP 126/80 B) A1C 9% C)FBG 130mg/dL D) LDL cholesterol 100mg/dL

"B) A1C 9% Rationale: Lowering hemoglobin A1C (to average of 7%) reduces microvascular and neuropathic complications. Tighter glycemic control(normal A1C < 6%) may further reduce complications but increases hypoglycemia risk."

"The nurse is caring for a patient whose blood glucose level is 55mg/dL. What is the likely nursing response? A. Administer a glucagon injection B. Give a small meal C. Administer 10-15 g of a carbohydrate D. Give a small snack of high protein food"

"C The client has low hypoglycemia. This is generally treated with a small snack."

"The client, an 18-year-old female, 5'4'' tall, weighing 113 kg, comes to the clinic for a wound on her lower leg that has not healed for the last two (2) weeks. Which diseaseprocess would the nurse suspect that the client has developed? 1.Type 1 diabetes. 2.Type 2 diabetes. 3.Gestational diabetes. 4.Acanthosis nigricans"

"Correct Answer: 2 Type 2 diabetes is a disorder that usually occurs around the age of 40, but it is now being detected in children and young adultsas a result of obesity and sedentary life-styles. Wounds that do not heal are a hall-mark sign of Type 2 diabetes. This client weighs 248.6 pounds and is short"

Which statement by the patient with type 2 diabetes is accurate. a. ""I am supposed to have a meal or snak if I drink alcohol"" b. ""I am not allowed to eat any sweets because of my diabetes."" c. I do not need to watch what I eat because my diabetes is not the bad kind."" d. The amunt of fat in my diet is not important; it is just the carbohydrates that raise my blood sugar."""

"Correct Answer: A Alcohol should be consumed with food to reduce the risk of hypoglycemia."

A client with diabetes melllitus has a blood glucose of 644mg/dl. The nurse intreprets that this client is most at risk of developing which type of acid base imbalance? " A. Metabolic acidosis B. Metabolic alkalosis C. Respiratory Acidosis D. Respiratory Alkalosis"

"Correct Answer: A, Metabolic Acidosis Rationale: DM can lead to metabolic acidosis. When the body does not have sufficient circulating insulin, the blood glucose level rises. At the same time, the cells of the body use all available glucose. The body then breaks down glycogen and fat for fuel. The by-products of fat metabolism are acidotic and can lead to the condition known as diabetic ketoacidosis."

"Which of the following persons would most likely be diagnosed with diabetes mellitus? A 44-year-old.. "A. Caucasian Woman B. Asian Woman C. African-American woman D. Hispanic Male

"Correct answer: African-American woman Rationale: Age-specific prevalence of diagnosed diabetes mellitus (DM) is higher for African-Americans and Hispanics than for Caucasians. Among those younger than 75, black women had the highest incidence."

"A patient with type 1 diabetes has received diet instruction as part of the treatment plan. The nurse determines a need for additional instruction when the patient says, "a. ""I may have an occasional alcoholic drink if I include it in my meal plan."" b. ""I will need a bedtime snack because I take an evening dose of NPH insulin."" c. ""I will eat meals as scheduled, even if I am not hungry, to prevent hypoglycemia."" d. ""I may eat whatever I want, as long as I use enough insulin to cover the calories.

"D. ""I may eat whatever I want, as long as I use enough insulin to cover the calories."" Rationale: Most patients with type 1 diabetes need to plan diet choices very carefully. Patients who are using intensified insulin therapy have considerable flexibility in diet choices but still should restrict dietary intake of items such as fat, protein, and alcohol. The other patient statements are correct and indicate good understanding of the diet instruction."

"The principal goals of therapy for older patients who have poor glycemic control are: A. Enhancing quality of life. B. Decreasing the chance of complications. C. Improving self-care through education. D. All of the above."

"D. All of the above. Rationale: The principal goals of therapy for older persons with diabetes mellitus and poor glycemic control are enhancing quality of life, decreasing the chance of complications, improving self-care through education, and maintaining or improving general health status."

"A nurse shoud recognize which symptom as a cardinal sign of diabetes mellitus? a. Nausea b. Seizure c. Hyperactivity d. Frequent urination

"D. Frequent Urination Polyphagia, polyuria, polydipsia, and weight loss are cardinal signs of DM. Other signs include irritability, shortened attention span, lowered frustration tolerance, fatigue, dry skin, blurred vision, sores that are slow to heal, and flushed skin."

"When an older adult is admitted to the hospital with a diagnosis of diabetes mellitus and complaints of rapid-onset weight loss, elevated blood glucose levels, and polyphagia, the gerontology nurse should anticipate which of the following secondary medical diagnoses? 1.Impaired glucose tolerance 2.Gestational diabetes mellitus 3.Pituitary tumor 4. Pancreatic tumor

"Pancreatic tumor Rationale: The onset of hyperglycemia in the older adult can occur more slowly. When the older adult reports rapid-onset weight loss, elevated blood glucose levels, and polyphagia, the healthcare provider should consider pancreatic tumor."

A nurse is caring for a client who has a new prescription for ferrous sulfate tablets twice daily for iron-deficiency anemia. The client asks the nurse why the provider instructed that she take the ferrous sulfate between meals. Which of the following responses should the nurse make? a. "Taking the medication between meals will help you avoid becoming constipated." b. "Taking the medication with food increases the risk of esophagitis." c. "Taking the medication between meals will help you absorb the medication more efficiently." d. "The medication can cause nausea if taken with food."

"Taking the medication between meals will help you absorb the medication more efficiently." Ferrous sulfate provides the iron needed by the body to produce red blood cells. Taking iron supplements between meals helps to increase the bioavailability of the iron.

A nurse is completing discharge instructions for a new mother and her 2-day-old newborn. The mother asks, "How will I know if my baby gets enough breast milk?" Which of the following responses should the nurse make? a. "Your baby should have a wake cycle of 30 to 60 minutes after each feeding." b. "Your baby should wet 6 to 8 diapers per day." c. "Your baby should burp after each feeding." d. "Your baby should sleep at least 6 hours between feedings."

"Your baby should wet 6 to 8 diapers per day." Newborns should wet 6 to 8 diapers per day. This is an indication that the newborn is getting enough fluids.

"A client who is started on metformin and glyburide would have initially presented with which symptoms? "a. Polydispisa, polyuria, and weight loss b. weight gain, tiredness, & bradycardia c. irritability, diaphoresis, and tachycardia d. diarrhea, abdominal pain, and weight loss

"a. Polydispisa, polyuria, and weight loss"Symptoms of hyperglycemia include polydipsia, polyuria, and weight loss. Metformin and sulfonylureas are commonly ordered medications. Weight gain, tiredness, and bradycardia are symptoms of hypothyroidism. Irritability, diaphoresis, and tachycardia are symptoms of hypoglycemia. Symptoms of Crohn's disease include diarrhea, abdominal pain, and weight loss."

*Shit question alert A nurse is providing teaching to a teenage client who is diagnosed with metabolic syndrome about making lifestyle changes to lose weight. Which client statement most indicates that the nurse's teaching has been successful? SATA A) "If I stick to this diet and exercise program, in a few weeks my clothes will fit better." B) "A lot of the weight I lose initially will be water, but afterward, I will start losing fat." C) "After a few weeks of diet and exercise, I will start to see changes in the way I feel." D) "I am at risk for developing health problems like diabetes if I don't change my lifestyle." E) "I need to make changes now, so I can live a long, healthy life."

*Note from Mary: I don't know why Pearson is saying that B is a correct choice, they seem to contradict it in the rationale... sooooo?? Proceed with caution & don't shoot the messenger ;) A, B, C Rationale: Teenage clients are unlikely to be motivated by thinking about the​ long-term effects of obesity.​ Instead, a teenage client would more likely be motivated by following​ short-term diet and exercise goals and an improvement in mood. Clients may be discouraged to discover that much of their initial weight loss is water weight. A teenager is unlikely to think about the serious consequences of developing a chronic disease like diabetes.

"A client is admitted to the hospital with signs and symptoms of diabetes mellitus. Which findings is the nurse most likely to observe in this client? Select all that apply: "1. Excessive thirst 2. Weight gain 3. Constipation 4. Excessive hunger 5. Urine retention 6. Frequent, high-volume urination

1, 4, 6 Rationale: Classic signs of diabetes mellitus include polydipsia (excessive thirst), polyphagia (excessive hunger), and polyuria (excessive urination). Because the body is starving from the lack of glucose the cells are using for energy, the client has weight loss, not weight gain. Clients with diabetes mellitus usually don't present with constipation. Urine retention is only a problem is the patient has another renal-related condition.

A nurse is planning to use evidence-based practice (EBP) to determine the best practice for dressing changes for clients. According to EBP, in what order should the nurse perform the following steps? - The nurse conducts a literature search to determine the presence of nursing literature on how to perform dressing changes. - The nurse asks a clinical questions about methods to perform a dressing change. - The nurse notices another nurse changing a dressing and recognizes that they perform the same procedure differently. - The nurse reads, organizes, and compares the results of the scientific literature search. - The nurse implements changes for performing dressing changes and collects data to determine client outcomes. - The nurse develops recommendations about dressing changes to present to an interdisciplinary team to update the policy and procedure.

1. The nurse notices another nurse changing a dressing and recognizes that they perform the same procedure differently. 2. The nurse asks a clinical question about methods to perform a dressing change. 3. The nurse conducts a literature search to determine the presence of nursing literature on how to perform dressing changes. 4. The nurse reads, organizes, and compares the results of the scientific literature search. 5. The nurse develops recommendations about dressing changes to present to an interdisciplinary team in order to update the policy and procedure. 6. The nurse implements changes for performing dressing changes and collects data to determine client outcomes.

"The nurse is discharging a client diagnosed with diabetes insipidus. Which statement made by the client warrants further intervention? 1."I will keep a list of my medications in my wallet and wear a Medi bracelet." 2."I should take my medication in the morning and leave it refrigerated at home." 3."I should weigh myself every morning and record any weight gain." 4."If I develop a tightness in my chest, I will call my health-care provider."

2."I should take my medication in the morning and leave it refrigerated at home.""1.The client should keep a list of medication being taken and wear a Medic Alert bracelet. 2. Medication taken for DI is usually every 8-12 hours, depending on the client. Theclient should keep the medication close at hand. 3.The client is at risk for fluid shifts. Weighing every morning allows the client to follow thefluid shifts. Weight gain could indicate too much medication. 4.Tightness in the chest could be an indicator that the medication is not being tolerated; if this occurs the client should call the health-care provider"

a nurse is interviewing a client with type 2 diabetes mellitus. which statement by the client indicated an understanding of the treatment for this disorder? 1. ""i take oral insulin instead of shots"" 2. ""by taking these medications I am able to eat more"" 3. ""when I become ill, I need to increase the number of pills I take"" 4. ""the medications I'm taking help release the insulin I already make""

4.)Clients with type 2 diabetes mellitus have decreased or imparied insulin secretion. Oral hypoglycemic agents are given to these clients to facilitate glucose uptake. Insulin injections may be given during times of stress-induced hyperglycemia. Oral insulin is not available because of the breakdown of the insulin by digestion. Options 1, 2 and 3 are incorrect

"The nurse is discussing the importance of exercising to a client diagnosed with Type 2diabetes whose diabetes is well controlled with diet and exercise. Which informationshould the nurse include in the teaching about diabetes? 1.Eat a simple carbohydrate snack before exercising. 2.Carry peanut butter crackers when exercising. 3.Encourage the client to walk 20 minutes three (3) times a week. 4.Perform warmup and cooldown exercises

4.Perform warmup and cooldown exercises "The client diagnosed with Type 2 diabetes whois not taking insulin or oral agents does notneed extra food before exercise.2.The client with diabetes who is at risk forhypoglycemia when exercising should carry asimple carbohydrate, but this client is not atrisk for hypoglycemia.3.Clients with diabetes that is controlled by dietand exercise must exercise daily at the sametime and in the same amount to control theglucose level. 4. [correct] All clients who exercise should perform warmup and cooldown exercises to helpprevent muscle strain and injury"

After not visiting their primary healthcare provider for over a decade, a 52-year-old man has a long list of health protection needs. The nurse can provide some of the teaching but thinks that the patient would benefit greatly from collaborative interventions. Which item on the patient's health protection plan involves an intervention that is collaborative? A. Scheduling a colonoscopy B. Handing the patient a "Choose My Plate" brochure C. Giving the family in-person counseling D. Providing individual telephone counseling

A

The nurse is caring for a 10-month-old infant. Two of the infant's teeth recently erupted, and the parent asks the nurse several questions about oral care for the infant. Which instruction should the nurse provide to the parent regarding the infant's oral care? A. "Use a wet washcloth to wipe the inside of your baby's mouth." B. "Wet your finger with water and use it to wipe the inside of your baby's mouth." C. "Brushing is not important since they are baby teeth." D. "Start by using a soft bristle toothbrush and fluoride toothpaste."

A

The nurse is caring for a 29-year-old who is 32 weeks pregnant and experiencing a normal pregnancy thus far. The patient does not have any significant preexisting disease or condition. The patient mentions, "I've noticed that my gums have been bleeding every time I floss my teeth." Which response by the nurse is accurate? A. "This is caused by a hormonal response change to your gum tissue." B. "This is caused by a proliferation of bacteria that occurs during pregnancy." C. "This is caused by increased circulating blood volume in pregnancy." D. "This is caused by improper oral hygiene techniques."

A

Which should the nurse include in a health promotion program as the ideal amount of weekly exercise for a group of older adults aged 70 plus? A. 150 minutes of moderate-intensity aerobic or 75 minutes of vigorous-intensity activities B. 60 minutes of moderate-intensity aerobic activity 3 times per week with some vigorous-intensity activity C. 90 minutes of moderate-intensity aerobic or 75 minutes of vigorous-intensity activities D. 150 minutes of moderate-intensity aerobic activity; vigorous-intensity activity only if approved by healthcare provider

A

Which statement accurately reflects that the client understands how to use the MyPlate plan? A- I will eat foods from each of the five food groups at every meal. B- You will send me a plate to use for my meals. C- This seems too hard to remember. Can I just eat less? D- I will use the MyPlate handout to increase my portion sizes.

A

A nurse is educating a group of older adults in a community center on weight management using the BMI scale. Using the client's height and weight to calculate BMI, which of the following clients has a healthy BMI? a. A client with a weight of 128 lb and height of 70 inches b. A client with a weight of 150 lb and height of 68 inches c. A client with a weight of 200 lb and height of 72 inches d. A client with a weight of 133 lb and a height of 60 inches

A client with a weight of 150 lb and height of 68 inches The formula for calculating BMI is weight in kilograms divided by the height in meters squared. The formula to convert pounds to kilograms is to divide the weight in pounds by 2.2 kilograms. The formula for converting inches to meters is to multiply the total inches times 0.0254 meters. 150 pounds divided by 2.2 equals 68.18 kilograms. 68 inches multiplied times 0.0254 inches equals 1.7272 meters. 1.7272 meters (squared) is 2.983 meters. 68.18 kilograms divided by 2.983 meters equals a BMI of 22.85. A BMI of 18.5-24.9 is considered a healthy weight.

A nurse at a health fair is assessing the weight status of four clients. Which of the following clients are classified as overweight? a. A female client who has a body mass index of 24 b. A male client who has a body mass index of 29 c. A female client who has a waist circumference of 101.6 cm (40 in) d. A male client who has a waist circumference of 96.52 cm (38 in)

A male client who has a body mass index of 29 A client who has a BMI of 25 to 29.9 is classified as overweight.

A nurse is working with a teen client who is attempting to lose weight. The client admits having difficulty being compliant with the diet prescribed by the HCP. Which suggestion by the nurse might assist the client in being compliant with the prescribed diet in a way that is sensitive to the client's age? A) "It can be difficult to avoid unhealthy foods if that's what your friends are eating, but try to choose healthier options when you can." B) "Write down the exact foods you eat so that you can see what how much you are eating." C) "Watch the nutrient content and number of calories in everything you eat." D) "Eat at the kitchen table so that you eat along with the rest of the family."

A) "It can be difficult to avoid unhealthy foods if that's what your friends are eating, but try to choose healthier options when you can." Rationale: Adolescence is a time of identity formation, and adolescents align with peers in regard to food selection. Keeping food diaries and monitoring the nutrient content and caloric values of food intake are helpful behavior modification strategies, but these don't take into account the age of the client, and studies on food consumption show that caloric information or nutrient content is not a major consideration in choice among adolescents. Parental food choices can have a strong impact on adolescents, but some adolescents rebel against these food choices, positively or negatively, and eating with the rest of the family is only beneficial if the family's habits are healthy.

The nurse is interviewing a client who is obese and has gained 6 pounds over the last 2 months. The client says, "My sister-in-law old me about nutritional bars that I can eat as a meal replacement. Will they help me to lose weight?" How should the nurse respond? A) "Nutritional bars are often high in calories and may cause weight gain." B) "Nutritional bars should be eaten with calcium and vitamin C supplements to ensure optimal effect." C) "Nutritional bars are most effective when eaten at the same time each day." D) "Nutritional bars must be spaced consistently throughout the day to be most effective."

A) "Nutritional bars are often high in calories and may cause weight gain." Rationale: Nutritional bars are used to provide protein and nutrients for people who do not get enough in their daily​ intake, and therefore are high in calories and can lead to weight gain in a client who is trying to lose weight. The time of day at which the bars are being eaten is not as important as how many calories they contain. Nutritional bars are not usually prescribed to be eaten with calcium or vitamin C supplements.

A nurse is teaching a client about snacks that are appropriate on a low-fat, low-sodium, and low-colesterol diet. Which of the following foodchoices by the client indicates the need for further teaching? A) A slice of cheese B) jam sandwich C) A cup of plain popcorn D) A small container of applesauce

A) A slice of cheese

The nurse is working with a morbidly obese client who is seeking help to lose weight at a bariatric clinic. When planning this client's care, which nursing diagnosis is the priority? A) Activity Intolerance B) Disturbed Body Image C) Defensive Coping D) Constipation

A) Activity intolerance Rationale: Along with diet, exercise is an important part of a weight loss program. A client with morbid obesity has a sedentary lifestyle and will have activity intolerance. Disturbed Body Image and Constipation may both be legitimate diagnoses, but Activity Intolerance is a greater priority if the client is to lose weight. There is no evidence that this client exhibits defensive coping.

After achieving a consistent weight loss of 2 pounds per week for a month and a half, the client reports that it has been weeks since any further weight loss. Which action should the nurse take first? A) Ask the client if there has been any changes in their exercise or diet patterns B) Tell the client that they might have reached a temporary plateau in their weigh loss C) Review the client's diet and exercise plan guidelines with the client D) Tell the client to weigh themselves and record it each week.

A) Ask the client if there has been any changes in their exercise or diet patterns Rationale: The nurse should first try to determine a reason for the change in weight loss. Although it is necessary for the client to weigh and record themselves each​ week, it is more important to assess the reason for the change before any other interventions are planned.

A nurse is planning care for a client who is receiving enteral feedings through an NG tube. Which of the following actions should the nurse plan totake first? A) Aspirate the client's stomach contents. B) Hang the feeding bag 30 cm (12 in) above the client. C) Label the feeding bag with the date and time of the start of the feeding. D) Warm the feeding to room temperature.

A) Aspirate the client's stomach contents.

A nurse is using evidence-based practice (EBP) to address the incidence of catheter-associated infections on a surgical unit. According to EBP, which of the following actions should the nurse take first? A) Identify a clinical problem B) Collect best evidence relevant to the question C) Evaluate Studies to determine validity D) Share the findings with others

A) Identify a clinical problem.

A nurse is completing discharge teaching with a client. Of the following barriers to learning the nurse identifies with this client, which should thenurse interpret as a need to postpone the session? A) Pain B) Hearing loss C) The client's culture D) Motor impairment

A) Pain

A client who is obese is suspected of having a metabolic syndrome. Which assessment finding will help confirm the suspicion? A) Taking the client's BP B) Checking the client's apical pulse C) Asking the client about their dietary intake D) Testing the client's urine for protein

A) Taking the client's BP Rationale: One of the characteristics of metabolic syndrome is elevated blood pressure. The other tests do not provide information that assists with the diagnosis of metabolic syndrome

The client diagnosed with Type I diabetes is found lying unconscious on the floor of the bathroom. Which interventions should the nurse implement first? A. Administer 50% dextrose IVP. B. Notify the health-care provider. C. Move the client to ICD. D. Check the serum glucose level.

A) admin 50% dextrose IVPThe nurse should assume the client is hypoglycemic and administer IVP dextrose, which will rouse the client immediately. If the collapse is the result of hyperglycemia, this additional dextrose will not further injure the client.

the nurse is preparing a teaching session about nutrition for a community health fair. Which information should the nurse include about obesity? SATA A) Food choices contribute to the development of obesity B) Refined foods, animal proteins and fats contribute to obesity C) Preventing obesity lowers the risk of developing hypertension D) Portion sizes help control body weight E) The prevalence of obesity has declined over recent years

A, B, C, D Rationale: Portion sizes and food choices both help control body weight and may help reduce the incidence of obesity. Refined​ foods, animal​ proteins, and fat intake contribute to obesity. Obesity is linked to the development of hypertension. The prevalence of obesity has increased​ 25% in adults and doubled in children and adolescents.

The community health nurse is working with a school district to prevent the development of obesity in school-age and adolescent children. Which strategy should the nurse emphasize? SATA A) Offer food choices that are nutritionally sound and help maintain a healthy body weight B) Incorporate 1 hour of physical activity into each child's school day C) Encourage parents to provide breakfast for their children before coming to school D) Notify parents of children who are obese E) Remove sugary beverages from vending machines on school property

A, B, E Rationale: Strategies to prevent the development of obesity in​ school-age and adolescent children include incorporating 1 hour of physical activity each​ day, limiting the intake of sugary​ beverages, and offering nutritionally sound food choices at school. Notifying parents of children who are obese may not help to prevent the problem. Encouraging parents to provide breakfast to children before arriving to school might be unrealistic because the nurse and school district might not be aware of each​ family's ability to provide breakfast for children each day.

A client is admitted with hyperosmolar hyperglycemic state​ (HHS) and a blood glucose level of 550​ mg/dL. Which intervention should the nurse expect to include in the plan of​ care? (Select all that​ apply.) A. Give normal saline intravenously. B. Assess level of orientation. C. Obtain blood for hemoglobin A1C. D. Monitor serum potassium levels. E. Provide education about type 2 diabetes mellitus.

A,B,D Rationale: HHS can cause changes to a​ client's level of consciousness ranging from lethargy to​ coma; therefore, the nurse should assess the​ client's level of orientation. The hyperosmolarity of the blood causes severe dehydration and depletion of electrolytes.​ Therefore, the priority care for a client with HHS is to provide isotonic or colloid solutions intravenously. Potassium is​ depleted, so it must not only be​ monitored, but also replaced. This client is acutely​ ill, so the hemoglobin A1C should be​ reviewed, but it is not a priority. Education should wait until the​ client's blood glucose level is stabilized and the client is alert enough to be receptive to the teaching.

The nurse is taking a health history from a client who has type 1 diabetes mellitus. Which client symptom may indicate the development of​ complications? (Select all that​ apply.) A.Dizziness B.Numbness in the feet C.Quick wound healing D.Vision changes E. Frequent voiding of urine

A,B,D,E Rationale: Dizziness, vision​ changes, numbness in the​ feet, and frequent voiding of urine may indicate that the client has developed complications of type 1 diabetes mellitus. Clients with type 1 diabetes mellitus frequently experience prolonged wound​ healing; therefore, a report of quick wound healing would not indicate that the client has developed a complication of type 1 diabetes mellitus.

The nurse is teaching a group of young adults regarding nonmodifiable risk factors for the development of type 1 diabetes mellitus. Which attendee statement indicates a need for further​ instruction? (Select all that​ apply.) A.​"Type 1 diabetes mellitus can be passed on from one recessive gene from one​ parent." B."There are genes such as the​ HLA-DR3 and​ HLA-DR4 genes that can cause type 1 diabetes​ mellitus." C."Type 1 diabetes mellitus can be caused by exposure to excessive heat and​ temperatures." D."I can develop type 1 diabetes mellitus from bacterial​ infections." E."Type 1 diabetes mellitus is caused by exposure to processing of metals and​ proteins."

A,C,D,E The individual with type 1 diabetes mellitus usually inherits the risk factor for the disorder from each parent. Environmental factors such as cold weather and exposure to a virus also contribute to the development of type 1 diabetes mellitus. The genes​ HLA-DR3 and​ HLA-DR4 have been identified in people with type 1 diabetes mellitus. Exposure to processing of metals contributes to the development of cirrhosis.

The nurse is managing care for a client weighing 165 pounds who was admitted for the treatment of diabetic ketoacidosis​ (DKA). Which intervention would be most appropriate for the nurse to include in the plan of​ care? (Select all that​ apply.) A.Place the client on a telemetry monitor. B. Provide a​ high-protein diet. C.Give 100 mL of normal saline bolus. D. Measure intake and output every hour. E. Administer​ sliding-scale regular insulin.

A,D Rationale: The nurse would calculate intake and output on an hourly basis to determine fluid needs. The client would be placed on a telemetry monitor to monitor for dysrhythmias related to shifts in potassium levels. The client with DKA would be acutely ill and if able to​ eat, would be placed on a​ carbohydrate-controlled diet. The nurse would administer normal saline boluses at​ 10-20 mL/kg. A volume of 100 mL is not sufficient. Insulin would be administered​ intravenously, not sliding scale.

Which statement made by a client with type 1 diabetes mellitus indicates an understanding of instruction provided regarding disease​ management? (Select all that​ apply.) A."I should administer insulin during the day in multiple​ injections." B."I should maintain my hemoglobin A1C levels at or below​ 8%." C."I should count calories consumed to determine insulin needs for each​ day." D."I should trim my toenails at an angle to prevent cutting the​ skin." E. "I should obtain blood glucose levels prior to each insulin​ injection."

A,E ​Rationale: For better blood glucose​ control, the healthcare provider would instruct the client to administer insulin throughout the day in multiple injections and to obtain blood glucose levels prior to each injection. Hemoglobin A1C levels should be below​ 6.5%. The client should be instructed to count​ carbohydrates, not calories. Toenails should be cut straight across with a clipper and the edges and corners smoothed with an emery board. If the client is unable to see his feet or reach them​ easily, someone else can trim his nails. If the nails are very thick or​ ingrown, if toes​ overlap, or if circulation is​ poor, then a podiatrist should cut the​ client's toenails.

The nurse is teaching a client about a newly prescribed iron supplement. Which information should the nurse include? Select all that apply. A- Explain how the supplement can cause constipation B- explain the need to increase fiber in the diet C- expect the stool to turn black D- suggest that the client avoid ingesting citrus fruits with the sub E- remind the client to ingest adequate amounts of fluid each day

A- Explain how the supplement can cause constipation B- explain the need to increase fiber in the diet C- expect the stool to turn black E- remind the client to ingest adequate amounts of fluid each day

The nurse is assessing an undernourished client. Which manifestation of the integumentary system should the nurse consider an expected finding? Select all that apply A- Petechiae B- spoon shaped nails C- dry brittle hair D- constipation E- muscle wasting

A- Petechiae B- spoon shaped nails C- dry brittle hair

The nurse conducting a research study needs to determine if an individual qualifies to be a participant. Which criteria must be met for​ inclusion? (Select all that​ apply.) A.The individual meets all the inclusion criteria. B.The individual has been given informed consent. C.The individual receives payment for participation. D.The individual must waive the right to anonymity. E. The individual is informed of all aspects of the study.

A. The individual meets all the inclusion criteria. B. The individual has been given informed consent. E. The individual is informed of all aspects of the study. ​Rationale: Research participants are defined as volunteers for a specific study project who meet all the inclusion​ criteria, have been informed of all aspects of the​study, and have given informed consent. Adherence to the ethical principle of justice requires protection of the research​ participant's anonymity. Payment is not a mandatory condition of participation in research.

A nurse is caring for a client diagnosed with hypertension. The​ client's healthcare provider suggests the client begin a regular exercise schedule. The client asks the​ nurse, "The healthcare provider said that exercise benefits the​ brain, but I​ don't see how it​ can." Which is the correct response by the​ nurse? A. "Exercise leads to increased oxygen to the​ brain, which improves the​ brain's thinking​ abilities." B. "Exercise increases oxygen flow to the​ brain, increasing the number of brain​ cells." C. "Exercise increases brain​ oxygen, resulting in an increase in the number of neurons in the​ brain." D. "Exercise opens the cerebral​ arteries, which improves​ short-term memory."

A."Exercise leads to increased oxygen to the​ brain, which improves the​ brain's thinking​ abilities." ​Rationale: Exercise increases ventilation and oxygen​ intake, improving gas​exchange, eliminating more​ toxins, and increasing oxygen to the​ brain, thus enhancing the​ brain's cognitive​ (thinking) function.​ Short-term memory is not improved with​ exercise, and the number of brain cells and neurons does not increase with exercise and increased oxygen flow.

A community health nurse is providing education to a group of adults about the types of exercise that promote health. Which statement will the nurse include in the​ teaching? (Select all that​ apply.) A. "The talk test may be easier to use than the​ heart-rate calculation for determining effort in aerobic​exercise." B. "Anaerobic exercises are exercises where the amount of oxygen taken into the body during exercise is greater than that used to perform the​ activity." C. "An example of an anaerobic exercise is​ weightlifting." D. ​"Isometric exercise, or isolated muscle​ contraction, creates an increase in blood flow to all parts of the​body." E. "Isotonic exercises like walking and swimming are also known as dynamic​ exercises."

A."The talk test may be easier to use than the​ heart-rate calculation for determining effort in aerobic​exercise." C. ​"An example of an anaerobic exercise is​ weightlifting." E. "Isotonic exercises like walking and swimming are also known as dynamic​ exercises." ​Rationale: An example of anaerobic exercise is weightlifting. Isotonic exercises are also known as dynamic exercises. The talk test may be easier to use than the​ heart-rate calculation for determining effort in aerobic exercise. Aerobic​ exercises, not​ anaerobic, are exercises where the amount of oxygen taken into the body during exercise is greater than that used to perform the activity. Isometric exercise causes a slight increase in heart rate and cardiac​ output, but it causes no noticeable increase in blood flow to other parts of the body.

A nurse is caring for an older adult who is hospitalized due to a fractured hip secondary to a​ ground-level fall. The nurse is assisting the client with oral care when the client asks the​ nurse, "Why am I having more issues with​cavities? I always brush and floss my​ teeth." Which is the correct response by the​ nurse? A. "This can happen as you age because your teeth lose​ enamel, making them more susceptible to damage and cavity​ formation." B. "This is not an abnormal finding with aging because you brush and floss​ regularly." C. ​"This can happen with aging because saliva production​ increases, making teeth more susceptible to cavity​ formation." D. "This is a normal finding with aging because gum disease causes tooth​ decay."

A."This can happen as you age because your teeth lose​ enamel, making them more susceptible to damage and cavity​ formation." ​Rationale: As a normal process of​ aging, the teeth become more susceptible to damage and decay due to the thinning of the tooth enamel. The other answer choices include responses that have incorrect​ information: saliva production does not increase​ (it decreases) with​ age; good oral hygiene​ (brushing and flossing​regularly) would decrease the likelihood of cavity formation even with​ aging; and periodontal disease​ (gum disease) causes the loss of​ teeth, not dental caries.

Which​ evidence-based resource would be appropriate for the nurse to integrate into the care of a pregnant​ client? A. Information on breastfeeding from Healthy People 2020 B. Teaching pamphlets on circumcision C. Safety instructions from a car seat manufacturer D. Consequences of illegal drug use from the local police department

A.Information on breastfeeding from Healthy People 2020 Rationale: Evidence-based resources use current best evidence in making decisions about the care of individual patients. Healthy People 2020 provides​science-based benchmarks to track and monitor progress towards health to motivate individuals. While there are multiple opportunities for educational​ topics, only those based in the science can be considered​ evidence-based. Healthy People 2020 strives to identify nationwide health improvement priorities. As​ such, breastfeeding information from this source would be considered​ evidence-based, while other teaching pamphlets or information from local agencies or manufacturers may not.

Which response is an example of a background​ question? (Select all that​ apply.) A.Why​ isn't long-acting insulin administered via the IV​ route? B. What is the pathophysiology of left ventricular heart​ failure? C. How does acupuncture compare with pain medication for treating chronic knee​ pain? D. How does incentive spirometry help prevent collapse of the small airways in the​ lungs? E. What is the link between healthcare​ providers' stethoscopes and​ healthcare-acquired infections?

A.Why​ isn't long-acting insulin administered via the IV​ route? B. What is the pathophysiology of left ventricular heart​ failure? D. How does incentive spirometry help prevent collapse of the small airways in the​ lungs? ​Rationale: Background questions are knowledgebased and seek more information about a​ topic, such as medications or diseases. Answers to background questions can be found in​ textbooks, drug​ guides, medical​ dictionaries, and other education resources. Foreground questions are practice based​ and, compared with background​ questions, they are narrower in scope. Foreground questions focus on a specific clinical issue and their answers identify useful information about direct client care that may guide the formulation of nursing interventions that improve client outcomes.

A nurse in a primary care clinic is caring for a client who has osteoporosis but no significant health history. Which statement made by the nurse is most appropriate when teaching this client about the benefits of​ exercise? A."Walking is an excellent choice of exercise for your​ condition." B. "Non-weight-bearing exercise is most beneficial in your​ condition." C. "Isometric exercise is an excellent choice for your​ condition." D. "Swimming is most beneficial for your​ condition."

A.​"Walking is an excellent choice of exercise for your​ condition." ​Rationale: Walking, or​ weight-bearing exercise, is most beneficial for the client with osteoporosis who can bear weight. Bone density and strength are maintained through​ weight-bearing activities. The stress of​ weight-bearing and​ high-impact movement maintains a balance between osteoblasts​ (bone-building cells) and osteoclasts​ (bone-resorption and​ bone-breakdown cells). Although swimming is a great choice for those who cannot bear​ weight, this client would best benefit from walking. Isometric exercises are beneficial for muscles but do not provide​weight-bearing benefits for the client with osteoporosis.

A client is discussing recent difficulties with mild anxiety due to stress at work. They ask the nurse about ways that they can manage the anxiety, because they really do not want to take medication. Which intervention should the nurse include? SATA A. Teaching the client how to recognize stress triggers B. Teaching the client how to differentiate between different levels of stress. C. Speaking slowly and using a low pitched voice with the client. D. Encouraging the client to use the self management and diversion techniques to cope with stress. E. instructing the client to reduce environmental stimuli

ABD

The nurse is preparing teaching material for parents regarding symptoms of anxiety in children and adolescents. Which symptom, common to both age groups, should the nurse include? SATA A. headaches and body aches B. shyness C. stomachaches D. excessive worrying E. muscle tension F. frequent need to urinate.

ACDE

the nurse believes that the client with serve anxiety will benefit from cognitive behavioral therapy. Which characteristic of CBT should the nurse consider? SATA a. The client can do CBT exercises b. The client can discontinue medications c. The client can remove stressors d. The client can safety confront fears e. The client can change unhealthy thoughts

ADE

A client reports a series of stressful events. They also report they are feeling very hopeless and empty. They are having difficulty solving even miner problems and are fantasizing what it would be like if all these things had not happened. Which of the indicators should be considered psychologic indicators? SATA a. difficulty solving minor problems b. hopelessness c. fantasizing d. feeling empty e. helplessness

B, D, E

Which suggestion should the nurse provide to a client with newly diagnosed type 2 diabetes mellitus regarding ways to increase​ activity? (Select all that​ apply.) A. Play card games at home. B. Take stairs at work. C. Limit computer time. D.Use a fitness tracker. E. Get a workout buddy.

B,C,D,E Rationale: Clients with newly diagnosed type 2 diabetes mellitus should incorporate a minimum of 150 minutes of exercise and activity per week into their lives. The nurse can instruct the client to take the stairs at​ work, use a fitness tracker to set goals and monitor​ activity, limit computer time because it is​ sedentary, and get a friend to go to the gym to exercise. Playing card games is a sedentary activity.

During a home visit, the nurse is concerned that the family is experiencing food insecurity. Which observation made by the nurse caused this concern? Select all that apply. A- the oldest daughter experiences abdominal cramps when ingesting milk products B- the mother prepares a meal for the children but she claims lack of appetite to avoid eating C- the children are visibly thin and argue over the food remaining on the serving plate D- the kitchen cabinets contain a limited amount of food for a family of 5 E- the mother is recovering from a second episode of pneumonia within 6 months

B- the mother prepares a meal for the children but she claims lack of appetite to avoid eating C- the children are visibly thin and argue over the food remaining on the serving plate D- the kitchen cabinets contain a limited amount of food for a family of 5 E- the mother is recovering from a second episode of pneumonia within 6 months

The nurse researcher is explaining​ evidence-based practice​ (EBP) to a group of nurses. Which statement is appropriate to include in the​ explanation? (Select all that​ apply.) A. "EBP promotes generalization of client​ care." B. ​"EBP incorporates the​ nurse's clinical​ expertise." C. "EBP considers the​ client's needs,​ values, and​ choices." D. "EBP tests hypotheses about​ health-related conditions." E."EBP is reflective of the best evidence from current​ research."

B. "EBP incorporates the​ nurse's clinical​ expertise." C."EBP considers the​ client's needs,​ values, and​ choices." D. "EBP is reflective of the best evidence from current​ research." ​Rationale: Evidence-based practice​ (EBP) combines the best evidence from the most current research​ available, the​ nurse's clinical​ expertise, and the​ client's preferences, including​ needs, values, and choices. EBP promotes individualization of client care. Nursing research tests hypotheses about​ health-related conditions and nursing care.

A nurse is caring for an older adult in the hospital and is providing the client with instruction on proper oral care. Which instructions will the nurse include in the​ teaching? A. "Use circular technique when​ brushing." B. "Use plain water to rinse the mouth after​ meals." C. "Use an​ alcohol-based mouth rinse after​ brushing." D. "Use mineral oil as a lip moisturizer if​ needed."

B. "Use plain water to rinse the mouth after​ meals." Rationale: When teaching the older adult about oral​ care, the nurse should instruct the client to rinse the mouth with plain water after meals to help remove residual food debris. Circular brushing techniques are not recommended. Use of mineral oil as a lip moisturizer is not recommended because aspiration of this may cause pneumonia.​ Alcohol-based mouth rinses are not recommended because these can irritate oral tissues.

A nurse working in a pulmonary rehabilitation clinic is conducting an exercise session for a group of clients with chronic obstructive pulmonary disease​ (COPD). Which exercise will the nurse include in the teaching​ session? (Select all that​ apply.) A. Stationary arm bicycle B. Lunges C. Squats D. Bicep curls E. Stair climbing

B. Lunges C. Squats E. Stair climbing ​Rationale: The client with COPD benefits most from​ lower-body exercises, including​ lunges, squats, and stair climbing. These exercises are performed as tolerated. Bicep curls and the use of stationary arm bicycles are​ beneficial; however, they are not as beneficial as​ lower-body exercises.

"The guidelines for Carbohydrate Counting as medical nutrition therapy for diabetes mellitus includes all of the following EXCEPT: a. Flexibility in types and amounts of foods consumed b. Unlimited intake of total fat, saturated fat and cholesterol c. Including adequate servings of fruits, vegetables and the dairy group d. Applicable to with either Type 1 or Type 2 diabetes mellitus

B. Unlimited intake of total fat, saturated fat and cholesterol"

A nurse is caring for a client who is postoperative from an open appendectomy. The client uses an overhead trapeze bar to transfer position in bed. Which type of exercise is demonstrated with this​ action? A. Isometric B. Isotonic C. Isodynamic D. Isokinetic

B.Isotonic ​Rationale: The client who uses an overhead trapeze bar to aid in transfer is demonstrating isotonic activity. Isometric activity is when the joint​ doesn't move but the muscle contracts​ (e.g., wall​ sits). Isokinetic exercise is when the muscle contracts against resistance.There are no exercises called isodynamic.

The nurse therapist is assessing an older adult. The client and the nurse are from different cultures. Which factor could complicate the nurses assessment of the client? SATA A. The clients work experience B. The clients normal. healthy cultural response C. The clients cognitive changes D. The clients age difference from the therapist E. The clients physical illness

BCE

A nurse is caring for a client who has a prescription for a stool test for guaiac. The nurse understands the purpose of the test is to check the stool for which of the following substances? a. Steatorrhea b. Blood c. Bacteria d. Parasites

Blood A guaiac test detects the presence of occult or hidden blood in the stool. The guaiac test is an extremely useful diagnostic screening test for the presence of colon cancer and gastrointestinal ulcers.

The nurse is caring for a 15-month-old toddler at a well-child clinic. The toddler's caregiver tells the nurse that the child still drinks a bottle of milk before bed. Which statement by the nurse is appropriate regarding the child's oral health? A. "It is best if your child drinks the milk from a cup instead of a bottle." B. "It is best if you give your child cow's milk instead of breast milk." C. "It is best if your child does not go to bed with the bottle." D. "It is best if your child doesn't drink a bottle at night."

C

The nurse is completing a history and physical assessment on a patient being evaluated for sleep disturbances. Which question asked by the nurse would be most effective in determining the patient's sleep quality? A. "Are you able to achieve several periods of deep sleep each night?" B. "How many hours do you sleep at night?" C. "Do you feel rested and energized when you wake up?" D. "Do you sleep at least 7 to 8 hours each night?"

C

The nurse is providing a health promotion class to expectant parents. Which should the nurse include when teaching about the sleep needs of infants and toddlers? A. Infants and toddlers should both sleep according to a regular schedule. B. A security object, such as a blanket, may provide security for both infants and toddlers when they sleep. C. As the infant becomes a toddler, the need for midmorning naps generally decreases. D. Infants and toddlers should both be placed on their back to sleep even if they are able to roll over.

C

The nurse is providing health promotion teaching to a middle-aged woman at risk of osteoporosis. The patient asks, "What is the benefit of weight-bearing exercises for osteoporosis?" Which response by the nurse provides accurate information? A. "Joint flexibility, stability, and range of motion are all increased by weight-bearing exercises." B. "This type of exercise helps to enlarge muscles and increase the efficiency of muscular contraction." THE CORRECT ANSWER C. "Weight-bearing exercises help maintain a balance between bone building and bone resorption and breakdown of cells." D. "These exercises help to maintain the size, shape, tone, and strength of your muscles."

C

The nurse is supervising a new unlicensed assistive personnel (UAP) who is providing oral care to an unconscious patient who cannot tolerate a soft-bristled brush. Which action by the nursing assistant would reflect appropriate care for this patient? A. Provides oral care daily in the morning. B. Uses an alcohol-based mouthwash on a swab to clean the mouth. C. Uses an oral swab soaked in saline to clean the mouth. D. Uses a syringe to gently

C

To a parent of a young child shares with the nurse that the child refuses play outside, go to parks, or participate in family gatherings where a dog might be present for fear of getting bitten. Which question should the nurse ask to determine if this behavior meets the DSM-5 criteria for phobias? A. does your child worry constantly about possibly encountering a dog? B. is your child frightened of all dogs? C. how long has your child been exhibiting this phobia? D. does your child have a panic attack if a dog is nearby ?

C

The nurse is teaching a client scheduled for Roux-en-Y gastric bypass surgery about potential post-surgical complications and how to reduce them. Which client statement best indicates that teaching has been effective? A) "I need to eat at least one meal a day that is high in simple carbs." B) "Complications of this surgery are likely to be limited to mild GI issues for several days." C)" I need to be alert for the indications of infection or malnutrition." D) "I will not continue my exercise program following this surgery."

C) "I need to be alert for the indications of infection or malnutrition." Rationale: Possible postoperative complications for a procedure such as a gastric bypass include anastomosis leak with peritonitis, abdominal wall hernia, gallstones, wound infections, deep venous thrombosis, nutritional deficiencies, and gastrointestinal (GI) symptoms. If the client recognizes a need to be alert to the signs of infection and malnutrition, this shows awareness of some of the complications that might be expected. Mild GI issues are not the only significant complication of this surgery. The client likely should continue the exercise program. Eating meals high in simple carbohydrates can bring on dumping syndrome, a complication in which stomach contents move rapidly through the small intestine, drawing fluid into the intestine by osmosis.

A nurse is assisting on a quality improvement committee to decrease the number of client falls occurring at night. After identifying the problem, which of the following is the next step the nurse should take? A) Implement the practice change. B) Evaluate the results of the change. C) Complete a literature review. D) Communicate the outcomes with others.

C) Complete a literature review.

A nurse prepares to replace the nearly empty container of total parenteral nutrition (TPN) for a client when she finds that there has been a delayin receiving the new container of solution from the pharmacy. Which of the following solutions should the nurse infuse until the next container of TPN solution becomes available? A) Lactated Ringer's B) 3% sodium chloride C) Dextrose 10% in water D) 0.9% sodium chloride

C) Dextrose 10% in water

A nurse is instructing a group of clients about nutrition. The nurse should include that which of the following is a trigger for the formation of vitamin D in the body? A) Calcium B) Vitamin A depletion C) Exposure to sunlight D) Weight-bearing exercise

C) Exposure to sunlight

An older client has begun to gain weight and is concerned about how weight might affect longevity. Which suggestion should the nurse provide? A) Stop eating bread, potatoes, and rice B) Eat fewer carb-dense foods C) Focus on portion sizes D) Drink more water

C) Focus on portion sizes Rationale: Drinking more water will not prevent weight gain as the client ages. Portion control is a part of getting balanced​ nutrition, so teaching the client about how to get and maintain adequate nutrition is important in this case. The client should not be taught to avoid particular food groups because those foods provide essential nutrients and vitamins important to maintain health.

The nurse is planning a teaching seminar for a group of young adult clients who are at risk for obesity. Which statement by the nurse best addresses their needs? A) There are drugs that are good to use to reduce weight. B) Obesity often leads to low self-esteem and depression C) Proper diet and exercise programs can not only prevent obesity but also potentially improve the ability to think and the positivity of self perception D) Maternal obesity often leads to menstrual irregularities and higher incidences of infertility.

C) Proper diet and exercise programs can not only prevent obesity but also potentially improve the ability to think and the positivity of self-perception. Rationale: The young adults who are at risk for obesity need education about changing lifestyles and the importance of preventing obesity as opposed to treating it. Education should include tips on eating healthy and exercising, which can lead to other benefits than preventing obesity. Drugs are used to manage obesity, not prevent it. Information about maternal obesity is not necessarily relevant to all individuals and does not address health promotion. Although obesity is a risk factor for low self-esteem and depression, emphasizing these negative effects does not empower clients to address their risk factors.

A nurse is admitting a client to an alcohol abuse program. The client states, "I'm here because of my boss. It was part of my job to go to partiesand drink with clients." The client's statement is an example of which of the following defense mechanisms? A) Reaction-formation B) Compensation C) Rationalization D) Suppression

C) Rationalization

A nurse is serving on a continuous quality improvement (CQI) committee that has been assigned to develop a program to reduce the number ofmedication administration errors following a sentinel event at the facility. Which of the following strategies should the committee plan to initiate first? A) Provide an inservice on medication administration to all the nurses. B) Require staff nurses to demonstrate competency by passing a medication administration examination. C) Review the events leading up to each medication administration error. D) Develop a quality improvement program for nurses involved in medication administration errors

C) Review the events leading up to each medication administration error.

A nurse is assisting with teaching a newly licensed nurse about the Hospital Consumer Assessment of Healthcare Providers and Systems (HCAHPS) tool. Which of the following information should the nurse include? A) The HCAHPS tool is provided to clients upon admission to a facility. B) The HCAHPS tool is completed during an in-person interview. C) The HCAHPS tool is issued to measure client satisfaction about health care service. D) The HCAHPS tool results are not shared with the public

C) The HCAHPS tool is issued to measure client satisfaction about health care service.

What is the definition of the basal metabolic rate? A) The amount of energy stored in fat each day B) The speed of triglyceride breakdown C) The cost in kilocalories of being alive D) The speed at which glucose is converted to energy

C) The cost in kilocalories of being alive Rationale: More than 70% of the energy expended each day goes to maintaining the basal metabolic rate (BMR)-essentially, the "cost" (in kilocalories) of being alive. It is not a measure of triglyceride breakdown, storage of energy in fat, or the conversion of glucose to energy.

A nurse is caring for a client who weighs 209 pounds and is 1.67 meters tall. The client eats a high-protein diet and lifts weights to increase muscle mass. The client presents with complications such as sleep apnea, which is often caused by obesity. Which statement regarding this client is true? A) A body mass index calculation should provide an accurate measure of the client's amount of fat. B) A body mass index calculation is the best possible means of measuring this client's fat if combined with the client's waist-to-hip ratio. C) This client's body mass index calculation might indicate a false positive for obesity. D) The client should be given a bioelectrical impedance test to most accurately measure the client's fat.

C) This client's body mass index calculation might indicate a false positive for obesity Rationale: This client's weight might be at least partially from body building efforts, and so a body mass index calculation, which only uses the parameters of weight and height, might not accurately indicate whether this client is actually obese. Even if combined with the client's waist-to-hip ratio, a BMI calculation is not the best possible means of measuring a client's fat, and neither is a bioelectrical impedance test. Underwater weighing is considered the most accurate way to determine body fat.

A nurse is instructing a group of clients about nutrition and eating foods high in iron. The nurse should include that which of the following aids in the absorption of iron? A) Fiber B) Vitamin A C) Vitamin C D) Oxalates

C) Vitamin C

The community health nurse is working with a school to start a new program that targets obese teenagers in the community. Which activity is an example of the best exercise plan for the students? A) Running in PE class for at least 10-20 minutes, 3 days/week B) Playing football in the park for an hour each Sunday C) Walking for 35 minutes, 6-7 days/week D) Lifting weights with friends 2-3 days/week

C) Walking for 35 minutes, 6-7 days/week Rationale: Exercise is recommended daily for 30 minutes to an hour. Highly aerobic​ exercise, when done in short bursts or once a​ week, is not helpful and may be dangerous if an individual has not been exercising. Weight lifting is not as helpful as aerobic​ exercise, unless does for​ 30-45 minutes, 3 to 4 times a week. Running is a good exercise​ option, but it should be done for a longer period.

In educating a client with diabetes, what response would reveal need for further education? A. I should avoid tights B. I should take good care of my toe nails C. I should not go more than 3 days without washing my feet D. I should avoid going barefoot and should wear clean socks

C)I should not go more than 3 days w/o washing my feet"The recommended self-care routine is to wash feet on a daily basis without soaking and carefully cleaning."

"A nurse is caring for a client admitted to the emergency department with diabetic ketoacidosis (DKA). In the acute phase, the priority nursing action is to prepare to: "A. Correct the acidosis B. Administer 5% dextrose intravenously C. Administer regular insulin inraVenously D. Apply a monitor for an electrocardiogram."

C. Administer regular insulin inraVenously Lack (absolute or relative) of insulin is the primary cause of DKA. Treatment consists of insulin administration (regular insulin), intravenous fluid administration (normal saline initially), and potassium replacement, followed by correcting acidosis. Applying an electrocardiogram monitor is not a priority action.

A​ 2-year-old child with cerebral palsy may benefit from the model known as a medical home. Which goal from Healthy People 2020 does the nurse identify for the​ client? A. Limiting who lives in the home B. Paying for their medication C. Coordinating their healthcare services D. Paying the mortgage for the home

C. Coordinating their healthcare services​ Rationale: Healthy People 2020 objectives include reducing child mortality rates as well as decreasing the incidence of transmission of preventable diseases among children. Increasing access to a medical homelong dash—which is a model of care that promotes​ physician-led, client-centered, coordinated health serviceslong dash—for children with special needs is also an objective. The services do not provide financial backing nor do they limit who lives in the home.

Which action by a parent of a​ 12-year-old child with a new diagnosis of type 1 diabetes mellitus indicates a need for further​ teaching? A.Counting carbohydrates with the child B. Allowing the child to check blood sugars C. Discouraging​ after-school sports D. Scheduling a baseline exam with an ophthalmologist

C. Discouraging​ after-school sports Rationale: Exercise is a part of blood glucose and disease management.​ Therefore, the nurse should reeducate the parent to allow​ after-school sports. The parent should involve the​ 12-year-old child, so counting carbohydrates with the child and allowing the child to perform​ self-blood glucose monitoring is appropriate. Due to potential retinopathy that can occur with​ diabetes, it is appropriate for the parent to schedule an ophthalmic appointment to determine baseline visual acuity.

A caregiver of a​ 10-year-old fifth grader reports to the nurse about the current behavior of the child. The teacher reported to the caregiver that the student is falling asleep in​ class, is distracted in interactions with the peer​ group, and has trouble​ concentrating, leading to poor grades on assignments. The caregiver also reports that the child often plays video games late into the night. Which issue does the nurse suspect as the cause of the​ behavior? A. Sundown syndrome B. Nocturnal emissions C. Screen time at night D. Waking up frequently at night due to nightmares

C. Screen time at night Rationale: School-age children need 10dash-11 hours of sleep per night. They may spend more time at the​ computer, playing video​ games, and watching​ TV, leading to difficulty falling asleep and fewer hours of sleep. A regular and consistent sleep schedule and bedtime routine need to be established. The client is not experiencing sundown syndrome. Nocturnal emissions and nightmares are not the cause of the​ client's symptoms.

Which finding in the medical record indicates a client has good control of type 1 diabetes​ mellitus? A. Blood pressure​ 150/90 mmHg B. Free of amputations C. Hemoglobin A1C​ 5.4% D. Fasting blood sugar 200​ mg/dL

C.Hemoglobin A1C​ 5.4% Rationale: The finding that the client is maintaining a hemoglobin A1C of less than​ 6.5% indicates good diabetic control over the past 3 months. The client not having amputations indicates good peripheral​ circulation, but it does not indicate good disease management. Blood pressure of​ 150/90 mmHg is​ elevated, but it does not indicate good diabetes control. The fasting blood sugar should be under 125​ mg/dL. The finding of 200​ mg/dL is elevated.

Which statement best describes validity of research​ evidence? A.It is reflective of the​ study's application to clinical practice. B. It is the​ study's ability to produce consistent results with each use. C.It is the degree to which the study measured what it intended to measure. D.It is used to determine the strengths and weaknesses of a study and its resulting evidence.

C.It is the degree to which the study measured what it intended to measure. Rationale: Validity is the degree to which the study measured what it intended to measure. Reliability is the​ study's ability to produce consistent results with each use. Usefulness is reflective of the​ study's application to clinical practice. Critical appraisal is used to identify strengths and weaknesses of a study and its resulting evidence.

A client is brought to the emergency department in an unresponsive state, and a diagnosis of hyperglycemic hyperosmolar nonketotic syndrome is made. The nurse would immediately prepare to initiate which of the following anticipated physician's prescriptions? 1. Endotracheal intubation 2. 100 units of NPH insulin 3. Intravenous infusion of normal saline 4. Intravenous infusion of sodium bicarbonate

CORRECT ANSWER: 3. Intravenous infusion of normal saline Rationale: The primary goal of treatment is hyperglycemic hyperosmolar nonketotic syndrome (HHNS) is to rehydrate the client to restore the fluid volume and to correct electrolyte deficiency. Intravenous fluid replacement is similar to that administered in diabetic keto acidosis (DKA) and begins with IV infusion of normal saline. Regular insulin, not NPH insulin, would be administered. The use of sodium bicarbonate to correct acidosis is avoided because it can precipitate a further drop in serum potassium levels. Intubation and mechanical ventilation are not required to treat HHNS.

A nurse is caring for a client who needs to increase his protein intake. The client tells the nurse some of the food he enjoys. Which of the following foods should the nurse recommend as the best source of protein among these suggestions? a. Yams b. Eggs c. Chicken d. Peanuts

Chicken One 3 oz portion of roasted chicken breast provides about 25 g of protein. This is the best source of protein among these options

A nurse is caring for a client who is in crisis following the breakup of a long - term relationship . The client tells the nurse , " I might as well just die . My life is over . " Which of the following actions should the nurse take first ? Explore past positive coping strategies . Establish a follow - up plan of care .. Conduct a suicidal risk evaluation . Display a neutral attitude .

Conduct a suicidal risk evaluation . The greatest risk to this client is injury to self or others. therefore the first action the nurse should take is to conduct a suicidal and homicidal risk evaluation Exploring positive coping strategies is an essential aspect of crisis intervention, but it is not the first action the nurse should take

After bariatric surgery, the client tells the nurse, "I don't want to go to any support groups. I would prefer to be independent and accomplish this on my own." Which response by the nurse is best? A) "Because you go through so many life changes after surgery, we recommend support groups." B) "tell me what kind of resources you think would benefit you after this surgery." C) "I hope you will reconsider; we have a ton of great groups to choose from!" D) "Support groups are know to bring about more successful weight loss after surgery."

B) "Tell me what kind of resources you think would benefit you after this surgery." Rationale: The nurse should always assess the​ client's needs and preferences. Although the other options offer good information about support​ groups, they fail to take the​ client's preferences into consideration.

The nurse is developing a weight-reduction plan for a client diagnosed with obesity, metabolic syndrome, and atherosclerosis. Which question should the nurse ask first? A) "What physical activities do you like to do?" B) "What do you think led to your obesity?" C) "What are your favorite types of foods?" D) "How old were you when you became overweight?"

B) "What do you think led to your obesity?" Rationale: The nurse should ascertain the​ client's self-perception of how they became obese to make an individualized​ weight-reduction plan. The other information is​ important, but it is important to consider the​ client's beliefs in​ planning, as it makes the client more likely to make the necessary changes.

A nurse is educating a group of older adults in a community center on jveight management using the BMI scale. Using the client's height andweight to calculate BMI, which of the following clients has a healthy BMI? A) A client with a weight of 128 lb and height of 70 inches B) A client with a weight of 150 Ib and height of 68 inches C) A client with a weight of 200 lb and height of 72 inches D) A client with a weight of 133 lb and a height of 60 inches

B) A client with a weight of 150 Ib and height of 68 inches

A home health nurse is at the home of a client with diabetes and arthritis. The client has difficulty drawing up insulin. It would be most appropriate for the nurse to refer the client to: "A) A social worker from the local hospital B) An occupational therapist from the community center C) A physical therapist from the rehabilitation agency D) Another client with diabetes mellitus and takes insulin"

B) An occupational therapist can assist a client to improve the fine motor skills needed to prepare an insulin injection.

A nurse is caring for a client who has a prescription for a stool test for guaiac. The nurse understands the purpose of the test is to check the stool for which of the following substances? A) Steatorrhea B) Blood C) Bacteria D) Parasites

B) Blood

Which weight pattern should the nurse expect to observe when a client's energy requirements are completely met by kilocalorie intake in food? A) Weight changes every day B) Weight does not change C) Weight decreases D) Weight increases

B) Weight does not change Rationale: In​ general, when energy required is completely met by kilocalorie​ intake, weight does not change. When a client ingests more than a​ person's energy​ requirements, the client gains weight. When a client does not ingest enough kilocalories to meet their energy​ requirements, they lose weight.​ Fluid, not​ kilocalories, causes daily weight fluctuation.

Which is the purpose of parenteral nutrition? Select all that apply. A- to Promote reducing the intake of fats and carbohydrates B- to provide nutrients when Foods cannot be consumed by mouth C- to facilitate adherence to reduced calorie eating plans D- to deliver calories during recovery from lap band or other bariatric surgical procedures E- to supply nutrients when needs cannot be met through oral intake

B, E

The nurse is performing a physical assessment of a child. Which assessment finding should cause the nurse to suspect type 2 diabetes​ mellitus? A. Presence of acanthosis nigricans B. Body mass index 21 ​kg/m2 C. Blood pressure of​ 110/78 mmHg D.Pale mucous membranes

A.Presence of acanthosis nigricans ​Rationale: Acanthosis nigricans is a condition in which the skin is velvety in texture and brownish black in color with hyperkeratotic​ plaques; it is usually found in skin folds. This condition is often found in clients with type 2 diabetes mellitus and should be reported to the healthcare provider. A blood pressure reading of​ 110/78 mmHg is a normal finding as is a body mass index of 21​ kg/m2. Pale mucous membranes could be a sign of​ anemia; darkened mucous membranes could indicate type 2 diabetes mellitus.

A nurse is interviewing a client who recently experienced an act of workplace violence when an armed person held the workers at gunpoint before police intervened . The client now reports being anxious and fears the gunman might return . The nurse should identify that client is experiencing which of the following types of crisis ? Situational Cultural Maturational Adventitious

Adventitious Adventitious Crisis occur from natural disasters such as floods hurricanes or fire or from acts of war criminal activity or terrorism Situational crisis commonly Stems from events such as divorce job loss unexpected event or a change in circumstance

A nurse is admitting a client who reports anorexia and is experiencing malnutrition. Which of the following laboratory findings should the nurse expect to be altered? a. Creatine kinase b. Troponin c. Total bilirubin d. Albumin

Albumin A low albumin is a measure of plasma proteins which reflects the nutritional condition of a client experiencing anorexia and malnutrition over an extended period of time.

A nurse in a health clinic is interviewing a client who is upset and reports that their stress " is too much to handle . " The client is unemployed , a single guardian to young children , and has periodic asthma attacks . Which of the following stress - related conditions is the client experiencing ? Post - traumatic stress disorder ( PTSD ) Allostatic load Chronic illness Alarm stage

Allostatic load Chronic exposure to elevated or fluctuated endocrine or neural responses causes excessive wear and tear on the body organs, resulting in allostatic load. the nurse should identify that the clients constant stress may be manifesting itself and physical ailments such as periodic asthma attacks. The alarm stage is associated with the fight or flight response.

A nurse working on an orthopedic unit is caring for four clients. Which of the following clients should the nurse identify as being at greatest risk for skin breakdown? a)An adolescent who has cervical fracture and is in a halo brace b)A young adult who has a femur fracture and is in skeletal balanced suspension traction c)A middle aged adult who has a fractured radius and an arm cast d)An older adult who has a hip fracture and is in buck's position

An older adult who has a hip fracture and is in buck's position

"1. A patient with newly diagnosed type 2 diabetes mellitus asks the nurse what ""type 2"" means in relation to diabetes. The nurse explains to the patient that type 2 diabetes differs from type 1 diabetes primarily in that with type 2 diabetes a. the pt is totally dependent on an outside source of insulin b. there is a decreased insulin secretion and cellular resistance to insulin that is produced c. the immune system destroys the pancreatic insulin-producing cells d. the insulin precurosr that is secreted by the pancreas is not activated by the liver

Answer B - Rationale: In type 2 diabetes, the pancreas produces insulin, but the insulin is insufficient for the body's needs or the cells do not respond to the insulin appropriately. The other information describes the physiology of type 1 diabetes

A nurse is caring for a client who has suction equipment in their room. The client asks the nurse, "Why do you check my suction equipment every day even though I am not using it?" Which of the following statements should the nurse make? A) "It is part of an outcome audit being performed." B) "It is part of the quality assurance plan of the unit." C) "It is part of the plan developed by your case manager." D) "It is part of the quality improvement program."

B) "It is part of the quality assurance plan of the unit."

A nurse is teaching a client who has a new prescription for ferrous sulfate. Which of the following statements by the client indicates an understanding of the teaching? a. "I will expect the color of my urine to be amber." b. "I should expect dark, tarry stools." c. "I should expect increased bruising." d. "I will not get as many infections."

"I should expect dark, tarry stools." Ferrous sulfate is an iron supplement used to treat clients who have iron deficiency anemia. An expected adverse effect of this medication is black, tarry stools.

A nurse is providing teaching to the parents of a child who has iron deficiency anemia and is taking iron supplements. Which of the following statements by the parents indicates an understanding of the teaching? a. "The medication should be administered in one large dose every day." b. "Restricting fiber from our child's diet will help absorption of the iron." c. "The medication will be more effective if it is administered with meals." d. "Our child's blood count will need to be monitored routinely for several weeks."

"Our child's blood count will need to be monitored routinely for several weeks." The child's response to treatment will be determined by monitoring hemoglobin and hematocrit levels through routine blood tests. Treatment can take up to 3 months to be effective.

A nurse is instructing a group of clients regarding calcium rich foods. Which of the following foods should the nurse include in the teaching as the best source of calcium? a. ½ cup ice cream b. 1 ounce swiss cheese c. 1 cup milk d. 1 cup cottage cheese

1 cup milk Of the four choices, milk contains the most calcium per serving. Milk contains 276 mg calcium per one cup serving.

A nurse is instructing a group of adult clients about nutrition. The nurse should include which of the following as the recommended amount of vegetables servings per day? a. 1/2 cup b. 1 cup c. 2 cups d. 2 1/2 cups

2 1/2 cups Based on a typical 2000-calorie diet, the daily vegetable serving should be at least 2 ½ cups per day.

The nurse is caring for a client who has normal glucose levels at bedtime, hypoglycemia at 2am and hyperglycemia in the morning. What is this client likely experiencing? A. Dawn phenomenon B. Somogyi effect C. An insulin spike D. Excessive corticosteroids"

"1. B The Somogyi effect is when blood sugar drops too low in the morning causing rebound hyperglycemia in the morning. The hypoglycemia at 2am is highly indicative. The Dawn phenomenon is similar but would not have the hypoglycemia at 2am."

"The nurse is teaching a community class to peole with Type 2 diabetes mellitus. Which explanation would explain the development of Type 2 diabetes? 1. The islet cells in the pancreas stop producing insulin. 2. The client eats too many foods that are high in sugar. 3 The pituitary gland does not produce vasopression. 4. The cells become resistant to the circulating insulin.

"1. This is the cause of Type 1 diabetes mellitus. 2. This may be a reason for obesity, which may lead to Type 2 diabetes, but eating too much sugar does not cause diabetes. 3. This is the explanation for diabetes insipidus, which should not be confused with diabetes mellitus. 4. (CORRECT) Normally insulin binds to special receptor sites on the cells and initiates a series of reactions involved in metabolism. In Type 2 diabetes these reactions are diminished primarily as a result of obesity and aging."

A nurse is teaching a group of adults about nutrition. The nurse should include which of the following amounts as an appropriate daily intake of fiber for adult women? a. 5 to 10 g b. 10 to 15 g c. 20 to 35 g d. 40 to 50 g

20 to 35 g The Adequate Intake (AI) for total fiber for women is 20 g per day; therefore, 10 g would not be adequate.

A client with type I diabetes is placed on an insulin pump. The most appropriate short-term goal when teaching this client to control the diabetes is: 1) adhere to the medical regimen 2) remain normoglycemic for 3 weeks 3) demonstrate the correct use of the administration equipment. 4) list 3 self care activities that are necessary to control the diabetes"

3.) is correct "1) this is not a short-term goal 2) this is measurable, but it's a long-term goal 3) this is a short-term goal, client oriented, necessary for the client to control the diabetes, and measurable when the client performs a return demonstration for the nurse 4) although this is measurable and a short-term goal, it is not the one with the greatest priority when a client has an insulin pump that must be mastered before discharge"

A nurse is instructing a group of clients about nutrition. The nurse's teaching plan should state that in order to limit saturated fat intake, the client should limit total fat intake to what percentage of total calories per day? a. 20% b. 25% c. 30% d. 33%

30% In order to limit saturated fat intake, the total percentage of fat intake per day is 35%.

Of which of the following symptoms might an older woman with diabetes mellitus complain? 1) anorexia 2)pain intolerance 3) weight loss 4) perineal itching

4) perineal itchingRationale: Older women might complain of perineal itching due to vaginal candidiasis.

A client asks the nurse. "Is drinking a protein drink sufficient as a meal?" Which is the nurses most appropriate response? A- Tell me more about the protein drink you are using B- no, you should never drink protein drinks C- add a piece of fruit to your Breakfast & Lunch routine D- yes, that is fine

A

Which statement describes the difference between NREM and REM sleep? A. REM sleep is characterized by four distinct stages, while NREM sleep has only one stage. B. During REM sleep, it is very difficult to awaken the sleeper, while an individual in NREM sleep is easily awakened. C. About 20% of sleep during the night is NREM sleep, while 80% is REM sleep. D. During REM sleep, the body and brain are very active, while in NREM sleep, body and brain activity is slower.

D

A nurse is discussing quality measurement with a newly licensed nurse. The nurse should include that medication reconciliation is an aspect of which of the following parts of quality measurement? A) Benchmark B) Structure C) Outcome D) Process

D) Process

A nurse is assisting with teaching a class about quality core measures. Which of the following information should the nurse include? A) A cost-effective analysis is used to identify quality core measures. B) Client satisfaction is an example of a quality core measure. C) Quality core measures are filled out by clients to evaluate health care facilities. D) Quality core measures are a standard of care for treatment.

D) Quality core measures are a standard of care for treatment.

Which Laboratory test measures protein levels? Select all that apply A- complete blood count B- serum electrolytes C- lipids levels D- serum albumin E- serum prealbumin

D, E

A nurse is caring for a client who delivered a healthy baby boy by cesarean birth 24 hours ago. The nurse notes that the client correctly uses her arms to help transfer in and out of bed. Which type of exercise is the client demonstrating when performing this​ activity? A.Anaerobic B. Aerobic C. Isometric D. Isotonic

D. Isotonic ​Rationale: The client who uses her arms to aid in transfer is demonstrating isotonic activity. Aerobic​ exercise, such as running or​ hiking, conditions the cardiovascular system. Isometric activity is that in which the joint​ doesn't move but the muscle contracts​ (e.g., wall​ sits). Anaerobic exercise utilizes anaerobic pathways to provide additional energy for a short time because muscles are unable to use enough oxygen from the bloodstream. Sprinting is an example of anaerobic exercise. Next Question

The nurse is teaching a group of clients newly diagnosed with type 1 diabetes mellitus. Which information should the nurse include in the​ teaching? A."Monitor blood glucose levels​ weekly." B. "Take beta blockers daily to control blood​ pressure." C."Have routine pedicures​ performed." D. "Schedule regular ophthalmology​ visits."

D. "Schedule regular ophthalmology​ visits." Rationale: The client with type 1 diabetes mellitus is at high risk for retinal damage.​ Therefore, the nurse would teach the client to schedule regular ophthalmology visits to monitor vision. The nurse would not encourage the client to have regular pedicures due to possible injury that can occur from macrovascular and microvascular deficits. Blood glucose levels should be monitored several times a​ day, not once a week. The client would be prescribed​ angiotensin-converting enzyme​ (ACE) inhibitors to protect the kidneys from vascular damage.

A nurse is performing an oral health nursing assessment on an adult client. The nurse notes the presence of pyorrhea. Which does this finding​ indicate? A. The presence of glossitis B. The presence of fungal disease C. The presence of stomatitis D. The presence of periodontal disease

D. The presence of periodontal disease ​Rationale: Pyorrhea is the presence of pus at the gums when pressed. This is a manifestation of periodontal​ disease, not fungal disease. Stomatitis is the inflammation of the​ mouth, whereas glossitis is the inflammation of the tongue.

The nurse is teaching a child with type 1 diabetes mellitus and his family about sick day guidelines. Which statement by the family indicates successful​ teaching? A."We will test for ketones when the blood glucose level reaches 160​ mg/dL." B. "We will test for ketones when the blood glucose level reaches 180​ mg/dL." C. "We will test for ketones when the blood glucose level reaches 200​ mg/dL." D. "We will test for ketones when the blood glucose level reaches 240​ mg/dL."

D. "We will test for ketones when the blood glucose level reaches 240​ mg/dL. Rationale: Blood glucose levels of 160​ mg/dL, 180​ mg/dL, and 200​ mg/dL are​ elevated, but they would not require testing for ketones. Once the blood glucose level exceeds 240​ mg/dL, the child and family should test the urine for ketones.

The nurse is caring for a client who received a daily​ intermediate-acting insulin dose at​ 8:00 a.m. At which time of the day should the nurse provide the client a snack to prevent​ hypoglycemia? A. 11:00 a.m. B. 9:00 p.m. C. 6:00 p.m. D. 2:00 p.m.

D. 2:00 p.m. Rationale: Intermediate-acting​ (NPH) insulin peaks 6-8 hours after the injection.​ Therefore, the nurse would prepare a snack for the client beginning at​ 2:00 p.m. If the client received short-acting insulin​ (regular), the snack would be required between 10 a.m. and​ 11:00 a.m. Giving a snack at​ 6:00 p.m. or​ 9:00 p.m. may be appropriate for​ long-acting insulins, but it is not appropriate for​ intermediate-acting insulins.

The nurse is providing education to the client on the effect of lifestyle factors on sleep. Which should the nurse include as a factor that negatively influences​ sleep? (Select all that​ apply.) A. Regular nighttime schedule B. Morning exercise C. Relaxation D. Irregular work schedule E. Evening exercise

D. Irregular work schedule E. Evening exercise Rationale: Factors that negatively impact sleep include evening exercise and an irregular work schedule. Morning​ exercise, relaxation, and regular nighttime schedule are known to enhance sleep

The nurse is critically appraising a research study. Which statement is reflective of an appraisal of the​study's reliability? A."Although the study was supposed to measure the​ client's wound​ healing, it really measured the​ client's satisfaction with​ care." B."The original study was repeated three​ times, but it did not produce consistent​ results." C."The study's results are not applicable to clinical​ practice." D. "The study's results are applicable to clinical​ practice."

B."The original study was repeated three​ times, but it did not produce consistent​ results." Rationale: Reliability is the​ study's ability to produce consistent results with each use. Validity is the degree to which the study measured what it intended to measure. Usefulness is reflective of the​ study's application to clinical practice.

A nurse is assisting a quality improvement team that is using the Plan-Do-Study-Act (PDSA) model to address an increase in pressure injuries on a medical unit. Which of the following actions should the nurse identify as an example of the "Do" step of the PDSA model? A) Reviewing the data collected on clients how received the new protocol. B) Developing a plan to initiate a change in client care C) Implementing new evidence-based practice protocol D) Accepting the new protocol in the unit's policy and procedure guidelines

C) Implementing a new evidence-based practice protocol

The nurse is teaching self-management to clients after gastric bypass surgery. Which information should the nurse include? A) That the development of flabby skin can be prevented through exercise B) The importance of choosing high-fat foods for at least 30% of intake C) The recommendation to drink fluids between meals but not with meals D) The choice of foods that is high in fiber to promote bowel function

C) The recommendation to drink fluids between meals but not with meals Rationale: Clients who drink fluids with meals tend to have dumping syndrome and diarrhea. They should be choosing foods that are low in fat and fiber. Exercise does not prevent the development of flabby skin.

Which type of exercise is also known as dynamic​ exercise? A. Isometric B. Isokinetic C. Isotonic D. Isodynamic

C. Isotonic ​Rationale: Isotonic exercises are also known as dynamic exercises. These exercises are said to be practiced when active movement occurs and muscles shorten and contract. There are no exercises called isodynamic. Isometric exercises are also known as static exercises because the muscle contracts but the joint does not move. Isokinetic exercises are known as​ resistive, where the muscle contracts against resistance.

A nurse is teaching a client who is at risk for iron-deficiency anemia about optimizing her dietary intake of iron. The nurse should explain that which of the following sources of iron is easiest for the body to absorb? a. Spinach b. Cantaloupe c. Chicken d. Lentils

Chicken Food sources of iron fall into two categories - heme iron (from lean red meat, poultry, and fish) and nonheme iron (from fruit, vegetables, grains, and dried peas and beans). The body more easily absorbs heme iron.

A nurse is providing dietary teaching to the parents of a newborn who is being breastfed. The nurse should instruct that the transition to whole milk can occur at which of the following ages? a. 6 months b. 8 months c. 10 months d. 12 months

12 months The transition to whole milk can occur at the age of 12 months.

"A diabetic patient has a serum glucose level of 824 mg/dL (45.7 mmol/L) and is unresponsive. Following assessment of the patient, the nurse suspects diabetic ketoacidosis rather than hyperosmolar hyperglycemic syndrome based on the finding of "a. polyuria b. severe dehydration c. rapid, deep respirations ) d. decreased serum potassium"

C is correct, Signs and symptoms of DKA include manifestations of dehydration such as poor skin turgor, dry mucous membranes, tachycardia, and orthostatic hypotension. Early symptoms may include lethargy and weakness. As the patient becomes severely dehydrated, the skin becomes dry and loose, and the eyeballs become soft and sunken. Abdominal pain is another symptom of DKA that may be accompanied by anorexia and vomiting. Kussmaul respirations (i.e., rapid, deep breathing associated with dyspnea) are the body's attempt to reverse metabolic acidosis through the exhalation of excess carbon dioxide. Acetone is identified on the breath as a sweet, fruity odor. Laboratory findings include a blood glucose level greater than 250 mg/dL, arterial blood pH less than 7.30, serum bicarbonate level less than 15 mEq/L, and moderate to large ketone levels in the urine or blood ketones.

"What insulin type can be given by IV? Select all that apply: A. Glipizide (Glucotrol) B. Lispro (Humalog) C. NPH insulin D. Glargine (Lantus) E. Regular insulin

E) Regular insulinThe only insulin that can be given by IV is regular insulin.

A nurse is caring for a child who is 2hr postoperative following a tonsillectomy. Which of the following fluid items should the nurse offer the child at this time? a)crushed ice b)orange juice c)vanilla milkshake d)cranberry juice

crushed ice

"Prediabetes is associated with all of the following except: a. Increased risk of developing type 2 diabetes b. Impaired glucose tolerance c. Increased risk of heart disease and stroke d. Increased risk of developing type 1 diabetes"

"ANSWER: D Persons with elevated glucose levels that do not yet meet the criteria for diabetes are considered to have prediabetes and are at increased risk of developing type 2 diabetes. Weight loss and increasing physical activity can help people with prediabetes prevent or postpone the onset of type 2 diabetes."

"Excessive thirst and volume of very dilute urine may be symptoms of: A. Urinary tract infection B. Diabetes insipidus C. Viral gastroenteritis D.Hypoglycemia"

"Correct answer: B Diabetes insipidus is a condition in which the kidneys are unable to conserve water, often because there is insufficient antidiuretic hormone (ADH) or the kidneys are unable to respond to ADH. Although diabetes mellitus may present with similar symptoms, the disorders are different. Diabetes insipidus does not involve hyperglycemia."

A nurse is providing teaching for a client who has binge-eating disorder and is morbidly obese. The client has been prescribed orlistat. Which of the following statements indicates to the nurse that the client understands the teaching? a. "I will take my dose of orlistat every morning an hour before breakfast." b. "I will eat a no-fat diet to prevent side effects from the medication." c. "I will stop taking orlistat and call my doctor if my urine gets darker in color." d. "I will feel less hungry during meals while I am taking orlistat."

"I will stop taking orlistat and call my doctor if my urine gets darker in color." Orlistat can cause severe liver damage; therefore, the client should be taught manifestations of liver damage, including dark-colored urine, light-colored stools, jaundice, anorexia, vomiting, and fatigue.

A 54-year-old patient admitted with type 2 diabetes, asks the nurse what "type 2" means. Which of the following is the most appropriate response by the nurse? "1. ""With type 2 diabetes, the body of the pancreas becomes inflamed." 2. "With type 2 diabetes, insulin secretion is decreased and insulin resistance is increased." 3. "With type 2 diabetes, the patient is totally dependent on an outside source of insulin." 4. "With type 2 diabetes, the body produces autoantibodies that destroy b-cells in the pancreas.""

"Right Answer: 2 Rationale: In type 2 diabetes mellitus, the secretion of insulin by the pancreas is reduced and/or the cells of the body become resistant to insulin"

Blood sugar is well controlled when Hemoglobin A1C is... a. Below 7% b. Between 12%-15% c. Less than 180 mg/dL d. Between 90 and 130 mg/dL"

"a. Below 7% A1c measures the percentage of hemoglobin that is glycated and determines average blood glucose during the 2 to 3 months prior to testing. Used as a diagnostic tool, A1C levels of 6.5% or higher on two tests indicate diabetes. A1C of 6% to 6.5% is considered prediabetes."

A nurse is planning nutritional teaching for a client who is experiencing fatigue due to iron deficiency anemia. Which of the following foods should the nurse recommend to the client? a. 1.5 oz raisins b. 8 oz black tea c. 1 cup canned black beans d. 8 oz whole milk

1 cup canned black beans The nurse should recommend canned black beans as they contain the greatest amount of iron at 4.56 mg per serving.

A nurse is providing teaching about nutrition to a group of clients. The nurse should include that which of the following foods contains the highest level of thiamine per serving? a. 1 hard-boiled egg b. 1 cup dried pears c. 1 cup whole grain wheat flour d. 1 cup brussel sprouts

1 cup whole grain wheat flour Whole or enriched grains contain 0.981 mg thiamine, which is the highest level of thiamine.

"Which of the following is accurate pertaining to physical exercise and type 1 diabetes mellitus? "1. Physical exercise can slow the progression of diabetes mellitus. 2. Strenuous exercise is beneficial when the blood glucose is high. 3. Patients who take insulin and engage in strenuous physical exercise might experience hyperglycemia. 4. Adjusting insulin regimen allows for safe participation in all forms of exercise."

1) physical exercise can slow the progression of diabetes mellitusRationale: Physical exercise slows the progression of diabetes mellitus, because exercise has beneficial effects on carbohydrate metabolism and insulin sensitivity. Strenuous exercise can cause retinal damage, and can cause hypoglycemia. Insulin and foods both must be adjusted to allow safe participation in exercise.

A nurse is caring for a cient with type 1 diabetes mellitus. which client complaint would alert the nurse to the presence of a possible hypoglycemic reaction? "1. Tremors 2. Anorexia 3. Hot, dry skin 4. Muscle cramps

1) tremorsdecreased blood glucose levels produce autonomic nervous system symptoms, which are manifested classically as nervousness, irritability, and tremors. option 3 is more likely for hyperglycemia, and options 2 and 4 are unrelated to the signs of hypoglycemia.

"What will the nurse teach the client with diabetes regarding exercise in his or her treatment program? 1. During exercise the body will use carbohydrates for energy production, which in turn will decrease the need for insulin. 2. With an increase in activity, the body will use more carbohydrates; therefore more insulin will be required. 3. The increase in activity results in an increase in the use of insulin; therefore the client should decrease his or her carbohydrate intake. 4. Exercise will improve pancreatic circulation and stimulate the islets of Langerhans to increase the production of intrinsic insulin.

1. During exercise the body will use carbohydrates for energy production, which in turn will decrease the need for insuli"Rationale: As carbohydrates are used for energy, insulin needs decrease. Therefore during exercise, carbohydrate intake should be increased to cover the increased energy requirements. The beneficial effects of regular exercise may result in a decreased need for diabetic medications in order to reach target blood glucose levels. Furthermore, it may help to reduce triglycerides, LDL cholesterol levels, increase HDLs, reduce blood pressure, and improve circulation."

"The nurse administered 28 units of Humulin N, an intermediate-acting insulin, to a client diagnosed with Type 1 diabetes at 1600. Which action should the nurse implement? 1. Ensure the client eats the bedtime snack. 2. Determine how much food the client ate at lunch. 3. Perform a glucometer reading at 0700. 4. Offer the client protein after administering insulin.

1: ensure the client eats the bedtime snack"1. Humulin N peaks in 6-8 hours, making the client at risk for hypoglycemia around midnight, which is why the client should receive a bedtime snack. This snack will prevent nighttime hypoglycemia. (Correct) 2. The food intake at lunch will not affect the client's blood glucose level at midnight. 3. The client's glucometer reading should be done around 2100 to assess the effectiveness of insulin at 1600. 4. Humulin N is an intermediate-acting insulin that has an onset in 2-4 hours but does not peak until 6-8 hours."

A nurse is preparing a plan of care for a client with diabetes mellitus who has hyerglycemia. The priority nursing diagnosis would be: 1. Deficient knowledge 2. Deficient fluid volume 3. Compromised family coping 4. Imbalanced nutrition less than body requirements

2) deficient fluid volumeAn increased blood glucose level will cause the kidneys to excrete the glucose in the urine. This glucose is accompanied by fluids and electrolytes, causing an osmotic diuresis leading to dehydration. This fluid loss must be replaced when it becomes severe.

A nurse is preparing a teaching plan for a client with diabetes Mellitus regarding proper foot care. Which instruction is included in the plan? 1. Soak feet in hot water 2. apply a moisturizing lotion to dry feet but not between the toes 3. Always have a podiatrist cut your toenails, never cut them yourself 4. avoid using mild soap on the feet

2. The client is instructed to use a moisturizing lotion on the feet and to avoid applying the lotion between the toes.

"The client diagnosed with type 1 diabetse is receiving Humalog, a rapid-acting insulin, by sliding scale. The order reads blood glucose level: <150, zero (0) units; 151 to 200, three (3) units; 201 to 250, six (6 units); >251, contact health-care provider. The unlicensed assistive personnel (UAP) reports to thenurse the client's glucometer reading is 189. How much insulin should the nurse administer to the client?

3 unitsThe client's result is 189, which is between 151 and 200, so the nurse should administer 3 units of Humalog insulin subcutaneously.

The nurse is caring for a client with long-term Type 2 diabetes and is assessing the feet. Which assessment data would warrant immediate intervention by the nurse? 1)The client has crumbling toenails 2)The client has athlete's feet 3)The client has a necrotic big toe 4)The client has thickened toenails."

3) Nectrotic big toe"1)Crumbling toenails indicate tinea unguium, which is a fungus infection of the toenail. 2)Athlete's foot is a fungal infection that is not life threatening. 3)A necrotic big toe indicates "dead" tissue. The client does not feel pain in the lower extremity and does not realize there has been an injury and therefore does not seek treatment. Increased blood glucose levels decrease oxygen supply that is needed to heal the wound and increase the risk for developing an infection. 4)Big, thick toenails are fungal infections and would not require immediate intervention by the nurse; 50% of the adult population has this."

"A nurse performs a physical assessment on a client with type 2 diabetes mellitus. Findings include a fasting blood glucose of 120 mg/dL, temp of 101 F, pulse of 88 bpm, respirations of 22, and blood pressure of 100/72. Which finding would be of most concern to the nurse? "1. Pulse 2. Respiration 3. Temperature 4. Blood pressure"

3) temp. An elevated temperature may indicate infection. Infection is a leading cause of hyperglycemic hyperosmolar nonketotic syndrome or diabetic ketoacidosis. The other findings noted in the question are within normal limits.

"The nurse is teaching a class on atherosclerosis. Which statement describes the scien-tific rationale as to why diabetes is a risk factor for developing atherosclerosis? 1.Glucose combines with carbon monoxide, instead of with oxygen, and this leads tooxygen deprivation of tissues. 2.Diabetes stimulates the sympathetic nervous system, resulting in peripheralconstriction that increases the development of atherosclerosis. 3.Diabetes speeds the atherosclerotic process by thickening the basement membraneof both large and small vessels. 4.The increased glucose combines with the hemoglobin, which causes deposits of plaque in the lining of the vessels.

3.Diabetes speeds the atherosclerotic process by thickening the basement membraneof both large and small vessels."1.Glucose does not combine with carbonmonoxide.2.Vasoconstriction is not a risk factor for devel-oping atherosclerosis. 3.This is the scientific rationale why diabetesmellitus is a modifiable risk factor for atherosclerosis. 4.When glucose combines with the hemoglobinin a laboratory test called glycosylated hemo-globin, the result can determine the client'saverage glucose level over the past three (3)months"

A client with diabetes mellitus demonstratees acute anxiety when first admitted for the treatment of hyperglycemia. The most appropriate intervention to decrease the client's anxiety would be to 1. administer a sedative 2. make sure the client knows all the correct medical terms to understand what is happening 3. ignore the signs and symptoms of anxiety so that they will soon disappear 4. convey empathy, trust, and respect toward the client

4. The most appropriate intervention is to address the client's feelings related to the anxiety

A patient has type 2 diabetes, obesity, hypertension, and hyperlipidemia. The patient tells the nurse, "I want to lose weight and exercise, but I can never find the time. Plus, I have so much weight to lose, what's the point?" How should the nurse respond to this patient? A. "Losing just 5% of your body weight is extremely beneficial." B. "If you don't exercise, you will die at a young age." C. "It's really a matter of putting your mind to it." D. "Diet is more important than exercise anyway when it comes to weight loss."

A

Which manifestation should the nurse expect to occur as a result of the impact of exercise on the respiratory system? A. Increased mental clarity B. Increased heart rate variability C. Improved quality of sleep D. Increase in the body's metabolic rate

A

While teaching the client about niacin, the client asks the nurse: "what brand of niacin is best?" Which is the nurses correct response? A-Let me tell you about the side effects of various brands B- the brand is not important. Buy one that cost the least C- get the store brands D- I will ask the doctor which is the best brand

A

A nurse is assessing four female clients for obesity. Which of the following clients have manifestations of obesity? a. A client who has a body fat of 22% b. A client who has a BMI of 28 c. A client who has a waist circumference of 81.3 cm (32 in) d. A client who weighs 28% above ideal body weight

A client who weighs 28% above ideal body weight For a female client, obesity is classified as a weight 20% greater than ideal weight. A client whose weight is 28% above ideal body weight is classified as obese.

A nurse is assisting with teaching a class about sentinel events. The nurse should include that which of the following situations is a sentinel event? A) A nurse infused an incompatible blood product to a client. B) A client fell getting out of bed. C) A client who is confused walks out of the nursing unit. D) A nurse receives a needle stick injury from a contaminated needle

A) A nurse infused an incompatible blood product to a client. Rationale: A sentinel event is a serious reportable event that results in death, permanent harm, or severe injury to a CLIENT.

A nurse working in a community health center is counseling a teen regarding a suspected eating disorder. The adolescent is or normal weight but admits to periods of overeating, especially when his parents fight. This client's eating habits best demonstrate which risk factor for obesity? A) Cultural and environmental factors B) Heredity C) Low socioeconomic status D) Physical inactivity

A) Cultural and environmental factors Rationale: This client most often overeats because of stress, which is an environmental risk factor for obesity. The client's stress comes from the environment, such as problems at school or at home, and not from heredity, low socioeconomic status, or physical inactivity, although these are all risk factors as well.

A nurse in an emergency department is assessing a client who has traumatic injuries following an assault. The client sits quietly and calmly in theexamination room and states, "I'm fine." The nurse should recognize the client's behavior as which of the following reactions? A) Denial B) Displacement C) Projection D) Undoing

A) Denial

A nurse is explaining National Patient Safety Goals (NPSGs) to a newly licensed nurse. The nurse should include that which of the following is a goal addressed in the NPSGs? A) Improving staff communication B) Improving staff retention C) Increasing client satisfaction D) Increasing client involvement in their plan of care

A) Improving staff communication

A nurse is reviewing a health care facility's policy that states to use a gauze dressing over IV insertion sites. After completing a literature review, the nurse discovers that evidence-based practice (EBP) indicates to use a transparent dressing over IV sites. Which of the following actions should the nurse take next? A) Recommend changing the procedure to the policy and procedure committee. B) Evaluate the results of the change in procedure C) Implement the change into clinical practice D) Communicate the outcomes of the procedure change with others

A) Recommend changing the procedure to the policy and procedure committee.

"When assessing the patient experiencing the onset of symptoms of type 1 diabetes, which question should the nurse ask? a. ""Have you lost any weight lately?"" b. ""Do you crave fluids containing sugar?"" c. ""How long have you felt anorexic?"" d. ""Is your urine unusually dark-colored?""

A) lost any weight?"a. Weight loss occurs because the body is no longer able to absorb glucose and starts to break down protein and fat for energy. b. The patient is thirsty but does not necessarily crave sugar- containing fluids. c. Increased appetite is a classic symptom of type 1 diabetes. d. With the classic symptom of polyuria, urine will be very dilute."

Which nursing intervention should be implemented for a client at risk of nutritional imbalances? Select all that apply. A- Providing nutritional education B- calculating caloric needs C- reducing body Mass index D- using nutritional supplements E- promoting healthy eating habits

A, E

The nurse is providing teaching to a client with a new diagnosis of type 1 diabetes mellitus. The nurse should instruct the client about incorporating which treatment to help manage the​ disease? (Select all that​ apply.) A.Exercise B. Fluid restriction C. Medication D. Daily weight checking E. Nutrition

A,C,E Clients with type 1 diabetes mellitus are treated with​ exercise, nutrition, and medication. Fluid restriction and daily weight checking are not part of the treatment plan for clients with type 1 diabetes mellitus.

The nurse is demonstrating how to prepare a dinner menu using the MyPlate plan. Which food item should the nurse include. Select all that apply. A- Slices of roast beef B- one glass of milk C- sliced peaches D- steamed broccoli E- one cupcake

A- Slices of roast beef B- one glass of milk C- sliced peaches D- steamed broccoli

During a preschool​ screening, the caregiver of a​ 3-year-old child asks the nurse how many hours of sleep the child requires each night. Which response by the nurse is​ appropriate? A. "A 3-year-old child needs 10 to 13 hours of sleep each​ night." B. "A 3-year-old child needs 7 to 9 hours of sleep each​ night." C. "A 3-year-old child needs 8 to 10 hours of sleep each​ night." D. "A 3-year-old child needs 14 to 17 hours of sleep each​ night."

A. "A 3-year-old child needs 10 to 13 hours of sleep each​ night." ​Rationale: Growing children require more sleep than adults. The​ 3-year-old child requires 10 to 13 hours of sleep each night. Infants need 14 to 17 hours of sleep in 24 hours. Adolescents require 8 to 10 hours of sleep each night. Adults need 7 to 9 hours of sleep each night.

A nurse is caring for an older adult who complains of dry mouth. The client​ states, "Food just​ doesn't taste the same​ anymore." The nurse understands that the​ client's dry mouth may be the cause of the​ client's altered taste. Which statement is true regarding saliva and​ taste? A. "Decreased saliva production thins the oral mucosa and contributes to dry​ mouth, so a decrease in saliva will decrease​ taste." B. "Because saliva is produced in the​ tongue, a decrease in saliva will decrease​ taste." C. "Saliva increases taste by increasing the function of the taste​ buds, so a decrease in saliva will decrease​taste." D. "A decrease in saliva will decrease taste because saliva mechanically breaks down food into smaller​parts."

A. "Decreased saliva production thins the oral mucosa and contributes to dry​ mouth, so a decrease in saliva will decrease​ taste." ​Rationale: Saliva moistens food and provides enzymes to digest starches. As saliva​ decreases, it contributes to thinning of the oral​ mucosa, contributing to dry mouth and altered taste. Saliva does not increase the function of the taste buds. Saliva is produced by the salivary​ glands, not the tongue. Saliva chemically breaks down​ food; teeth mechanically break down food.

The nurse is providing care to a client at a sleep disorder clinic. Which assessment finding does the nurse expect during REM​ sleep? A.Decrease in voluntary muscle tone B. Tachypnea C. Decrease in eye movement D. Tachycardia

A. Decrease in voluntary muscle tone ​Rationale: During REM​ sleep, the nurse would expect the client to have a decrease in voluntary muscle tone. A decrease in heart rate and respiratory rate is also​ expected, not tachycardia or tachypnea. An​ increase, not a​ decrease, in eye movement is expected during REM sleep.

which are symptoms of hypoglycemia? A. irritability, B. drowsiness c. Abdominal pain D. nausea and vomiting

A. Irritability: signs of hypoglycemia include irritability, shaky feeling, hunger, headache, dizziness. Other symptoms are hyperglycemia.

The nurse is preparing to present the benefits of​ evidence-based practice​ (EBP) as it relates to nursing to colleagues. Which benefit should the nurse​ include? (Select all that​ apply.) A. Receiving the best clientoutcomes B. Increasing the accuracy of charting C. Contributing to the knowledge of nursing D. Increasing teamwork in healthcare facilities E. Assisting in the delivery of​ high-quality nursing care

A. Receiving the best clientoutcomes C. Contributing to the knowledge of nursing E. Assisting in the delivery of​ high-quality nursing care ​Rationale: Some basic activities can provide a foundation for implementing EBP. These strategies can spark the necessary stimulus to engage in behaviors that encourage best practice. Participating in EBP contributes to the knowledge of nursing and client care in​ today's healthcare systems and delivery of​ high-quality nursing care for best outcomes. Utilizing EBP​ doesn't increase the accuracy of​charting, as that is all dependent on the nurse and the knowledge of what they add to the record. EBP governs nursing​ practice, but teamwork is something that needs to be encouraged by management in each individual healthcare facility.

Which information should the school nurse provide when teaching a group of adolescents the risk factors for type 2 diabetes​ mellitus? A. Get sufficient exercise and activity. B. Monitor blood glucose levels. C. Increase carbohydrate intake. D. Limit the amount of protein intake.

A.Get sufficient exercise and activity. Rationale: Frequently, children with type 2 diabetes mellitus develop the disease from a sedentary lifestyle and obesity.​ Therefore, the nurse would instruct the adolescents regarding the benefits of exercise and activity. Children should limit the amount of carbohydrates and include a normal amount of protein. They do not need to monitor blood glucose levels unless a diagnosis of type 2 diabetes mellitus is made.

"The risk factors for type 1 diabetes include all of the following except: a. Diet b. Genetic c. Autoimmune d. Environmental"

A: Type 1 diabetes is a primary failure of pancreatic beta cells to produce insulin. It primarily affects children and young adults and is unrelated to diet.

The nurse is teaching a client who is experiencing anxiety about possible medications that can be used to manage the symptoms of anxiety. Which medication should the nurse discuss with the client? SATA A. Antipsychotic B. Antidepressant C. benzodiazepine D. Cortisol E. Beta blocker

ABCE

An external insulin pump is prescribed for a client with DM. The client asks the nurse about the functioning of the pump. The nurse bases the response on the information that the pump: a. Gives small continuous dose of regular insulin subcutaneously, and the client can self-administer a bolus with an additional dosage from the pump before each meal. b. Is timed to release programmed doses of regular or NPH insulin into the bloodstream at specific intervals. c. Is surgically attached to the pancreas and infuses regular insulin into the pancreas, which in turn releases the insulin into the bloodstream. d. Continuously infuses small amounts of NPH insulin into the bloodstream while regularly monitoring blood glucose levels"

ANSWER A. An insulin pump provides a small continuous dose of regular insulin subcutaneously throughout the day and night, and the client can self-administer a bolus with additional dosage from the pump before each meal as needed. Regular insulin is used in an insulin pump. An external pump is not attached surgically to the pancreas.

A nurse in a community clinic is interviewing a client who is distressed and reports being unable to sleep following a neighborhood fire several days ago . The client has hypertension , tachycardia , and is diaphoretic . The nurse should identify that the client is experiencing which of the following types of stress ? A. Acute stress B. Post - traumatic stress disorder ( PTSD ) C. Episodic acute stress D. Chronic stress

Acute stress Most episodes of acute stress do not have lingering affects it is when it prolonged stress levels linger could lead to issues

"The nurse caring for a 54-year-old patient hospitalized with diabetes mellitus would look for which of the following laboratory test results to obtain information on the patient's past glucose control? a. prealbumin level b. urine ketone level c. fasting glucose level d. glycosylated hemoglobin level

Answer d: A glycosylated hemoglobin level detects the amount of glucose that is bound to red blood cells (RBCs). When circulating glucose levels are high, glucose attaches to the RBCs and remains there for the life of the blood cell, which is approximately 120 days. Thus the test can give an indication of glycemic control over approximately 2 to 3 months.

A nurse is planning care for a client who is receiving enteral feedings through an NG tube. Which of the following actions should the nurse plan to take first? a. Aspirate the client's stomach contents. b. Hang the feeding bag 30 cm (12 in) above the client. c. Label the feeding bag with the date and time of the start of the feeding. d. Warm the feeding to room temperature.

Aspirate the client's stomach contents. The first action the nurse should take using the nursing process is to assess the residual stomach contents. The nurse should measure the stomach contents to assess whether the feeding is being absorbed by the client. The nurse might delay the tube feeding for a high residual to reduce the risk of aspiration.

A nurse is providing teaching about iron deficiency anemia to the parents of a toddler. Which of the following should the nurse recommend as a method of preventing iron deficiency anemia? a. Avoid a diet that consists primarily of milk. b. Administer fat-soluble vitamins daily. c. Include fluoridated water in the toddler's diet. d. Limit intake of high-protein foods.

Avoid a diet that consists primarily of milk. Milk is a poor source of iron and a diet that consists primarily of milk places the toddler at risk for iron deficiency anemia.

A nurse is caring for a patient with asthma who is hospitalized due to an acute exacerbation of the disease. The nurse teaches the patient that exercise may be beneficial in the treatment of asthma. Which exercise should the nurse recommend? A. Swimming B. Yoga C. Cycling D. Running

B

A public health nurse is working with a community group to arrange a series of seminars about environmental control. Which response by the residents reveals a need for additional education? A. "We should look at the food chain, from farmers to the grocery store." B. "We should talk about ways to immunize kindergartners." C. "We must follow water conservation measures." D. "We should figure out how to reduce the use of cars for short trips."

B

A sedentary patient who is obese has been encouraged to begin to exercise. The patient asks the nurse if there are other ways in which weight might be lost without so much dependence on exercise. Which response by the nurse might encourage the patient to begin an exercise regimen? A. You could initially try changing your diet; that is an effective approach for some people." B. "Exercise has many benefits other than weight loss; it supports overall optimum mental and physical health." C. "I know it can be difficult, but it is vital to weight loss—you will likely be unsuccessful if you don't exercise." D. "While research on the effects and benefits of exercise can be mixed, it is important that you at least give it a try."

B

How many hours of sleep does an adolescent require as compared to an older adult? A. Adolescents require more hours than they did as a child but require less than older adults. B. Adolescents require on average 8-10 hours of sleep versus older adults who require on average 7-8 hours. C. Adolescents require on average 9-11 hours of sleep versus older adults who require on average 6-7 hours. D. Adolescents require on average less hours of sleep than older adults.

B

The nurse is completing a health history on a 2-year-old toddler. Which question by the nurse should elicit the most relevant information concerning oral health for the toddler? A. "Do you believe that your child generally consumes a balanced diet?" B. "Do you assist your toddler with brushing their teeth with a soft toothbrush after they eat?" C. "When did your toddler get their first tooth?" D. "Do you rinse your child's mouth with water after each meal?"

B

The nurse is working with a client who is having a panic attack. The client has been pacing back and forth in the back hallway for the past 45 minutes. How should the nurse immediately respond to this behavior? A. Contacting the unit counselor to meet with the client immediately to Discuss the cause of the panic. B. Allowing the client to pace to help diffuse energy C. Attempting to divert the client with another type of activity. D. Administering anxiolytics as prescribed

B

The school nurse is teaching a group of adolescents about oral health.Which nutrients should the nurse include in the presentation to prevent tooth decay? A. Vitamins B, E, and K and potassium B. Vitamins A, C, and D and calcium C. Iron, zinc, and chromium D. Folic acid and niacin

B

Which statement reflects a comparison of acute versus chronic illness? A. Chronic and acute illnesses can both generally be resolved, returning the patient to their normal level of wellness. B. Chronic illnesses generally have a slow onset and continue, whereas acute illnesses appear and subside abruptly. C. Acute illnesses are almost always followed by development of chronic illness requiring lifelong management. D. Most acute illnesses require medical intervention, whereas chronic illnesses can often be managed with only lifestyle changes.

B

A middle-aged client who works at a sedentary job has a BMI of 31.5. The client asks the nurse, "What healthy choices can I make to lose my extra weight?" Which response by the nurse is best? A) "You should consider a more active profession that permits you some daily exercise." B) "Focus on making healthy food choices and controlling your portions." C) "Have you considered starting a new workout plan?" D) "Muscle weighs more than fat, which could account for some of your weight."

B) "Focus on making healthy food choices and controlling your portions." Rationale: When clients express​ weight-loss goals, the three most crucial areas on which to focus are​ diet, exercise, and behavior modification. Working out is an important part of weight​ loss, but it is significantly more effective when paired with a structured eating plan. The nurse should prioritize healthy food choices and portion control. Telling the client that muscle weighs more than fat is​ misleading, as it is unlikely that this client has more muscle mass than fat. A more active profession may or may not assist the​ client's weight loss.

The nurse is reviewing a client's weight-reduction plan with the client. Which client statement indicates that the nurse's review has been successful? A) "I should weigh myself at the same time every morning and night." B) "I may lose a lot of weight quickly on a low-carb diet, but I might gain it all back over time" C) "I should weigh all of my food on a scale to choose the right portion size." D) "I should stick to a 500-700 calorie diet for the quickest weight loss."

B) "I may lose a lot of weight quickly on a low-carb diet, but I might gain it all back over time." Rationale: Fad diets make weight loss achieved by clients so much more difficult to maintain because they are so restrictive. Clients should weigh themselves​ weekly, not daily.​ Very-low-calorie diets are not necessary for clients in the overweight category of​ obesity, and they must be supervised very closely. Portion sizes can be measured in ways other than weighing.

A nurse is assisting with using the Plan-Do-Study-Act (PDSA) model to decrease client falls in a long-term care facility. The nurse should identify that developing guidelines to decrease falls is included in which of the following steps of the PDSA model? A) Do B) Plan C) Study D) Act

B) Plan

A nurse at a college campus mental health counseling center is caring for a student who just failed an examination. The student spends thesession berating the teacher and the course. The nurse should recognize this behavior as which of the following defense mechanisms? A) Conversion B) Projection C) Undoing D) Regression

B) Projection

A nurse is assessing a client and discovers the infusion pump with the client's total parenteral nutrition (TPN) solution is not infusing. The nurseshould monitor the client for which of the following conditions? A) Excessive thirst and urination B) Shakiness and diaphoresis C) Fever and chills D) Hypertension and crackles

B) Shakiness and diaphoresis

The nurse is counseling a client who is obese. Which observation is most important to note? A) The client's dislike of vegetables B) The client's difficulty standing from a seated position C) The client's shortness of breath after brisk walking D) The client's negative self-image caused by obesity

B) The client's difficulty standing from a seated position Rationale: In treating clients with​ obesity, clients are educated about​ exercise, diet, and behavior modification. It is important to note any​ self-imposed restrictions that clients may have due to excess weight in order to plan successful workout plans for them. It is not as important that the client does not like​ vegetables, as there are other options to help them achieve balanced nutrition without having to directly eat vegetables. Shortness of breath is a commonly reported symptom of obese people who have performed physical activity.​ Finally, it is important to note negative​ self-image and issues with​ self-esteem, but it is easier to focus on the​ short-term goal of establishing an exercise​ routine, as this has the potential of leading to higher​ self-esteem as the client loses weight.

One of the benefits of Glargine (Lantus) insulin is its ability to: a.Release insulin rapidly throughout the day to help control basal glucose. b. Release insulin evenly throughout the day and control basal glucose levels. c. Simplify the dosing and better control blood glucose levels during the day. d. Cause hypoglycemia with other manifestation of other adverse reactions.

B)Release insulin evenly throughout the day and control basal glucose levels"Glargine (Lantus) insulin is designed to release insulin evenly throughout the day and control basal glucose levels.

A group of students who are studying for final exams talking about ways to better deal with stressful events in life? One student suggest during finals week, we should avoid biogenic stressors. How should the students follow that advice SATA? a. Don't stay up all night study b. Don't drink fluids with caffeine in them c. Don't smoke cigarettes d. Don't go outside in freezing temperatures e. Don't worry about test results.

B, C, D

The nurse is caring for an adult client with a BMI of 26.8 who complains of sleep apnea and gout. The nurse anticipates that treatment of this patient for obesity will consist of which therapies? SATA A) Pharmacotherapy B) Diet C) Exercise D) Behavior modification E) Surgery

B, C, D -Diet -Exercise -Behavior Modification Rationale: Clients with a BMI of 25-26.9 with two or more comorbidities-sleep apnea and gout in this case-would be treated with the therapies of diet, exercise, and behavior modification, but likely not pharmacotherapy or surgery.

The nurse notices that a client recently lost 10 lb. Which question or statement should the nurse use to learn more about the weight loss? Select all that apply. A- Do you eat enough? B- Share with me what your daily eating habits look like. C- I noticed that you have lost weight. D- You need to eat more dessert. E- Tell me about how you do your food shopping.

B, C, E

The nurse learns that a client follows a vegan diet. Which suggestion should the nurse give the client to ensure an adequate intake of vitamins and minerals? Select all that apply. A- Include soft cheeses B- drink soy milk fortified with vitamin B12 C- put eggs into a salad every week D- add tofu and lentils into the diet E- increase the intake of citrus fruits

B, D, E

The nurse concludes that a client is demonstrating cardiovascular manifestations of undernutrition. Which assessment finding supports this conclusion? Select all that apply A- Lethargy B- low blood pressure C- amenorrhea D- slow respiratory rate E- irregular heartbeat

B, E

After performing a health history and physical assessment for a​ client, the nurse suspects type 2 diabetes mellitus. Which assessment finding is consistent with the​ nurse's suspicion?​ (Select all that​ apply.) A. Decreased urination B. Hyperglycemia C. Extreme thirst D. Acanthosis nigricans E. Hypertension

B,C,D,E Rationale: Symptoms that would lead the nurse to conclude the client has type 2 diabetes mellitus are extreme​ thirst, hyperglycemia,​ hypertension, and acanthosis nigricans​ (a condition in which the skin is velvety in texture and brownish black in color with hyperkeratotic​ plaques). A client with type 2 diabetes mellitus would have increased and not decreased urination.

The nurse is planning to teach a class regarding factors that influence food choices. Which factor should the nurse include. Select all that apply A- level of hunger B- cost C- emotion D- convenience E- availability

B- cost C- emotion D- convenience E- availability

​Evidence-based practice​ (EBP) is very prescriptive across the lifespan. Which client would be the focus for decreasing the incidence of pressure ulcers as presented in Healthy People 2020​? A. A​ 16-year-old who is hospitalized after knee surgery B. A​ 65-year-old client who is bedridden in a nursing home C. A​ 12-year-old whosleeps​ 10?12 hours a day D. A​ 35-year-old who is in a rehab facility following neck fusion surgery

B. A​ 65-year-old client who is bedridden in a nursing home ​Rationale: For older​ adults, Healthy People 2020 objectives address topics including increasing the number of individuals who are up to date on basic preventive care and decreasing the incidence of health alterations such as pressure ulcers and fall related injuries. The older adult who is bedridden would be the focus of pressure ulcer prevention due to thinner skin and no mobility. Lying in the bed put pressure on the bony prominences.

The nurse is working a health fair and teaching the public about risk factors for type 1 diabetes mellitus. Which ethnicity would the nurse include as having the highest risk in the United​ States? A.Hispanic B.Caucasian American C.African American D.Asian American

B. Caucasian American Rationale: Caucasian Americans have a higher risk of developing type 1 diabetes mellitus than Asian​ Americans, African​ Americans, or Hispanics.

A client asks the home care nurse about the difference between acute and chronic illness. Which information does the nurse give the client about the characteristics of chronic​ illness? (Select all that​ apply.) A. Chronic illness does not have exacerbations. B. Chronic illness can remain for life. C. Chronic illness can have remissions. D. Chronic illness usually lasts 6 months or more. E. Chronic illness has a slow onset.

B. Chronic illness can remain for life. C. Chronic illness can have remissions. D. Chronic illness usually lasts 6 months or more. E. Chronic illness has a slow onset. ​Rationale: Chronic illness can have both remissions and exacerbations. It usually lasts 6 months or more and can remain for life. It has a slow onset.

A client recently diagnosed with type 2 diabetes mellitus reports difficulty managing the disease. To which professional should the nurse refer the client for help with caloric​ intake? A. Primary healthcare provider B. Dietitian C. Social worker D. Personal trainer

B. Dietitian Rationale: The dietitian would best be able to help the client develop meal plans and incorporate foods that the client likes. The social worker would help the client find community resources to meet financial needs. A personal trainer would help the client increase activity. The primary healthcare provider manages the disease process as well as the multidisciplinary healthcare team.

The nurse enters the room of a sleeping client whose eyes are rolling from side to side and whose respiratory rate and heart rate have decreased. The client is easily aroused and​ states, "I​ wasn't asleep." Which state of​ non-REM (NREM) sleep was the client​ in? A. Stage N2 B. Stage N1 C. Stage N3 D. Stage N4

B. Stage N1 Rationale: Stage N1 is the stage of light sleep in which the client is easily​ aroused, the respiratory rate and heart rate decrease​ slightly, and the eyes roll slowly from side to side. Stage N2 is the stage of light sleep in which body processes slow down and the eyes are still. Stage N3 is the stage of deep sleep in which it is difficult to arouse the​ client, muscles are​ relaxed, and reflexes are diminished. Stage N4 is the stage of deep sleep that differs from Stage N3 in the number of delta waves produced.

A nurse is caring for a newborn with a congenital malformation of the oral soft palate. Which problem will this client be at risk for if the malformation is not​ repaired? (Select all that​ apply.) A. Breathing problems because the soft palate stents the airway open B. Swallowing problems because the soft palate rises during swallowing to direct food into the esophagus C. Breathing problems because the soft palate includes​ cilia, which keep foreign bodies out of the trachea D. Swallowing problems because the soft palate includes the​ uvula, which aids in swallowing E. Breathing problems because the soft palate closes off the trachea when swallowing

B. Swallowing problems because the soft palate rises during swallowing to direct food into the esophagus D. Swallowing problems because the soft palate includes the​ uvula, which aids in swallowing Rationale: Alterations of the soft palate may lead to swallowing​ problems, increasing the risk of aspiration. This is because the soft palate rises during swallowing to direct food into the esophagus. The soft palate ends at the back of the mouth at a fold called the uvula. The pressure of the bolus against the uvula causes a​ reflex, and the soft palate rises in order to close off the nasopharynx to prevent food from entering into the nasal cavity. Congenital soft palate malformations do not cause problems with the act of external respiration. The soft palate does not contain cilia.

The nurse is caring for a client with a​ long-term history of type 1 diabetes mellitus who has developed peripheral vascular disease. The nurse is unable to palpate the​ client's pedal pulses and the skin is cold to the touch. Which​ long-term goal is most appropriate for this​ client? A. The client will remain free of injury. B. The​ client's skin integrity will remain intact. C. The client will remain free from infection. D. The​ client's fasting blood glucose levels will stay between 70 and 110​ mg/dL.

B. The​ client's skin integrity will remain intact. Rationale: The client has impaired circulation as evidenced by cold skin and absent pedal pulses that indicate a risk for impaired skin integrity due to gangrene. There is no evidence the client is at risk for injury or has an infection. Having fasting blood glucose levels in the normal range indicates good disease​ management, but it does not relate to the impaired circulation.

Which statement best reflects the​ nurse's correct understanding of principles related to​evidence-based practice​ (EBP)? (Select all that​ apply.) A. "Client choices do not affect​ evidence-based practice." B. "Client-centered care is an outcome of​ evidence-based practice." C. "The nurse's level of expertise influences​ evidence-based practice." D. "Evidence-based practice promotes individualization of client​ care." E.​"Evidence-based practice requires application of all research evidence about a given​ topic."

B."Client-centered care is an outcome of​ evidence-based practice." C."The nurse's level of expertise influences​ evidence-based practice." D."Evidence-based practice promotes individualization of client​ care." ​Rationale: Evidence-based practice​ (EBP) combines the best evidence from the most current research​ available, the​ nurse's clinical​ expertise, and the​ client's preferences, including​ needs, values, and choices. EBP promotes individualization of client care and provides best practice for​ client-centered care.

A new mother who gained 55 pounds during her pregnancy is seen by the​ nurse-midwife for a​ 6-week follow-up after her delivery. The mother reports that she has 30 pounds of​ "baby weight" to lose. The mother tells the nurse that she is having trouble sleeping and asks why this is occurring because her infant is now sleeping through the night. Which response by the nurse is the most​ appropriate? A. "Hormonal shifts are causing your insomnia. I will prescribe you a​ sedative." B. ​"Your lack of weight loss may be the problem.​ Let's discuss a plan for​ this." C. "You must be worried about the​ baby's health. I will refer you to a​ counselor." D. "Once you become a​ parent, you have to get used to sleepless​ nights."

B."Your lack of weight loss may be the problem.​ Let's discuss a plan for​ this." ​Rationale: Weight gain has been associated with reduced total sleep​ time, interrupted​ sleep, and earlier awakening. It is appropriate for the nurse and the midwife to discuss a plan for this new mother to lose weight. Hormonal shifts can cause alterations in sleep​ patterns/insomnia in a new mother 6 weeks postpartum.​Sedation, however, is not the appropriate treatment. There is no evidence to support that the client is worried about her​ baby's health. Becoming a parent does not automatically cause sleep disturbances.

"A client is taking Humulin NPH insulin daily every morning. The nurse instructs the client that the mostlikely time for a hypoglycemic reaction to occur is: A) 2-4 hours after administration B) 4-12 hours after administration C) 16-18 hours after administration D) 18-24 hours after administration

B: Rationale: Humulin is an intermediate acting insulin. The onset of action is 1.5 hours, it peaks in 4-12 hours, and its duration is 24 hours. Hypoglycemic reactions to insulin are most likely to occur during the peak time.

The nurse is working with an older adult who is having side effects from medications for an anxiety disorder. The nurse wants to refer the client psychotherapy, but the client is adamant and states "I don't want to see a psychiatrist, that's for crazy people! Which reassurance should the nurse give the client? SATA A. with therapy, you can stop taking your medications right away B. therapy can help manage symptoms of anxiety C. therapy added to medications has more success than medications alone D. other professionals offer therapy besides psychiatrist E. therapist see many people who do not have mental disorders

BCDE

A patient tells the nurse that they have been doing isotonic exercises to strengthen their abdominal muscles. How should the nurse respond to this comment? A. "It would be best to combine isotonic, isometric, and isokinetic exercises if you want to achieve optimal results." B. "The repetitive nature of aerobic exercise would probably be more effective for strengthening your abdominal muscles." C. "While isotonic exercises can increase muscle tone and mass, isometric exercises may be more useful for strengthening abdominal muscles." D. "It would be better to do isokinetic exercises such as weight lifting to strengthen your abdominal muscles."

C

A young adult reports difficulty sleeping. Which factor should the nurse state that negatively affects an individual's ability to sleep? A. Avoiding activities, such as homework, for the hour prior to bed B. Avoiding use of electronic devices for an hour before retiring C. Exercising late in the day D. Maintaining a regular nighttime schedule

C

The home care nurse is providing follow-up care for a patient who experienced a traumatic spinal cord injury and is now paralyzed from the waist down. Which intervention addresses the patient's vulnerability that impacts the patient and their role in the family secondary to this injury? A. Encourage involvement and awareness of other necessary healthcare professionals to support healthy aspects of patient lifestyle. B. Help the patient to express thoughts and feelings concerning the recent life change and to support effective coping. C. Provide information to the patient to allow patient participation in decision making and the ability to stay in control of life and family decisions. D. Reinforce desirable lifestyle changes to help the patient and family, and make them a permanent part of patient's daily routine.

C

A nurse is instructing a group of clients about nutrition. The nurse's teaching plan should state that in order to limit saturated fat intake, client should limit total fat intake to what percentage of total calories per day? A) 20% B) 25% C) 30% D) 33%

C) 30%

The nurse is providing discharge teaching to a client with metabolic syndrome on behavior modification strategies for losing weight. Which nursing intervention is best? A) Instructing the client to use their smart phone to track their caloric intake B) Suggesting that the client celebrate with a "cheat meal" every time they make a weight-loss goal C) Asking about situations that increase the client's appetite D) Telling the client to eat small, frequent meals, rather than fewer, larger meals at regularly scheduled times each day

C) Asking about situations that increase the client's appetite Rationale: Successful behavior modification focuses on how and when a person eats and takes some importance away from aspects like calorie counting. In​ weight-loss programs, rewards that are not food are usually recommended. Clients should be taught to eat at designated mealtimes each day in behavior modification.

A nurse is assisting with developing a quality improvement (QI) plan to reduce the incidence of medication errors in an acute care unit. The nurse should recognize that which of the following actions is included in the identifying step of the QI process? A) Develop a plan to implement change. B) Evaluate the results of the change C) Recognizing the need for a change D) Implementing the plan of change

C) Recognizing a need for a change

A nurse is instructing a group of clients regarding nutrition. The teaching should state that which of the following groups of foods contains the highest level of carbohydrates? A) Milk, eggs, and cheese B) Butter, oil and avocados C) Rice, potatoes, and oranges D) Chicken, green beans, and apples

C) Rice, potatoes, and oranges

Which statements regarding upper body obesity are accurate? SATA A) Upper body obesity is also called peripheral obesity B) Upper body obesity is when the waist-to-hip ration in men is greater than than 0.8 or greater than 1 in women C) Upper body obesity is associated with a greater risk of hypertension D) Young women tend to have more intra-abdominal fat than men E) Postmenopausal women tend to have upper body obesity

C, E Rationale: Upper body obesity (also called central obesity) is identified by a waist-to-hip ratio of greater than 1 in men or 0.8 in women. Upper body obesity is associated with a greater risk of complications such as hypertension, abnormal blood lipid levels, heart disease, stroke, and elevated insulin levels. Men tend to have more intra-abdominal fat than women, although women develop a central fat distribution pattern after menopause.

A patient is admitted with diabetes mellitus, has a glucose level of 380 mg/dl, and a moderate level of ketones in the urine. As the nurse assesses for signs of ketoacidosis, which of the following respiratory patterns would the nurse expect to find?" A-Central apnea B-Hypoventilation C-Kussmaul respirations D- Cheyne-Stokes respirations"

C-Kussmaul respirationsIn diabetic ketoacidosis, the lungs try to compensate for the acidosis by blowing off volatile acids and carbon dioxide. This leads to a pattern of Kussmaul respirations, which are deep and nonlabored.

The nurse is conducting a health fair to screen for type 2 diabetes mellitus. Which participant should the nurse consider to be at highest​ risk? A. 30-year-old nurse who works in an intensive care unit B. 60-year-old retired architect who works at job site C. 50-year-old office worker who sits at the computer D. 40-year-old kindergarten teacher who works in a classroom

C. 50-year-old office worker who sits at the computer Rationale: A sedentary lifestyle is a risk factor for type 2 diabetes mellitus. The​ 50-year-old office worker who sits at the computer would be at highest risk for type 2 diabetes mellitus. All the other participants are physically active and are at lower risk.

A nurse is teaching a client who has type 1 diabetes mellitus about the peak time of neutral protamine hagedorn (NPH) inulin. Which of the following statements by the client indicates an understanding of the teaching? a. "NPH insulin peaks in 1 to 5 hours." b. "NPH insulin is peakless." c. "NPH insulin peaks in 6 to 14 hours." d. "NPH insulin peaks in 12 to 24 hours."

C. NPH insulin has an onset of 60 to 120 min, peaks in 6 to 14 hr, and has a duration of 16 to 24 hr.

The nurse is preparing a presentation on risk factors for type 2 diabetes mellitus. Which ethnic group should the nurse include as being amongst the highest diagnosed with this​ disease? A. Caucasian Americans B. Asian Americans C.American Indians D. African Americans

C.American Indians Rationale: The ethnicities that have the highest incidence of type 2 diabetes mellitus are the American Indians and Alaska Natives at​ 15.9%. Incidence rates for Asian​ Americans, African​ Americans, and Caucasian Americans are​ 9%, 13.2%, and​ 7.6%, respectively.

A nurse is caring for a client with type 1 diabetes mellitus. Which client complaint would alert the nurse to the presence of a possible hypoglycemic reaction ? A. Tremors B. Anorexia C. Hot, Dry skin D. Muscle cramps

Correct Answer A Decreased blood glucose levels produce autonomic nervous system symptoms, which are manifested classsically as nervousness, irritability, and tremors. Option C is more likely to occur with hyperglycemia. Options B and D are unrealted to the signs of hyperglycemia

The nurse is teaching a group of adults about the effects of exercise on various body systems. Which statement provides the rationale for how beneficial exercise is? A. "Adequate exercise promotes more efficient blood flow and increases peristalsis, excreting wastes more effectively, and enhancing problem solving and emotional stability." B. "The metabolic rate is elevated by exercise, leading to greater ability to solve problems and enhanced emotional stability." C. "Regular exercise helps improve the quality of sleep, resulting in better ability to solve problems and enhanced emotional stability." D. "Exercise results in deeper breathing which eliminates more toxins, thus enhancing problem solving and emotional stability."

D

A nurse is instructing a young adult client about healthful sleep habits. Which of the following statements should the nurse identify as an indication that the client needs further teaching? A) "I don't take naps throughout the day." B) "I go to bed and get up routinely at the same time each day." C) "I have a small snack and take a bath before going to bed each day." D) "I watch television until I fall asleep at night."

D) "I watch television until I fall asleep at night."

A nurse is teaching à client's adult son about how to position the client when administering enteral feedings at home. Which of the followingstatements by the son indicates an understanding of the teaching? A) "I will allow him to be in the position where he is most comfortable during the feeding. B) "I will elevate the head of the bed 10 degrees during the feeding. C) "I will turn him on his left side during the feeding.' D) "I will have him sit in his chair during the feeding."

D) "I will have him sit in his chair during the feeding."

A male client presents with suspected metabolic syndrome. The HCP orders anthropometry. Which waist measurement should the nurse anticipate? A) 32 inches B) 19 inches C) 28 inches D) 41 inches

D) 41 inches Rationale: Men with a waist measurement of 102 cm​ (40 in) or greater and women with a waist measurement of 88 cm​ (35 in) or greater have a higher risk for complications of obesity.

A nurse is a member of a committee that is using evidence-based practice (EBP) to decrease the incidence of central line infections. The nurse should identify that which of the following interventions is part of the implementation step of EBP? A) Recognizing a high incidence of central line infections B) Evaluating whether the practice changes decreased the infection rate. C) Researching the best practices to reduce the rate of central line infections. D) Incorporating the new practice into client care.

D) Incorporating the new practice into client care

A nurse is caring for a client who has a new prescription for a low-sodium diet. The client's family has requested to bring in some of the client'sfavorite foods. Which of the following food items should the nurse tell the family members to omit? A) Boiled rice B) Flat bread C) Broiled fish fillet D) Pickled vegetables

D) Pickled vegetables

A teenage client states, "I'm confused. I've always been so focused on my weight, but now my health teacher showed our class a MBI calculator online. Why does BMI matter?" Which response by the nurse is accurate? A) "The BMI explains an individual's development according to their age." B) "The BMI predicts the development of diabetes and heart disease." C) "The BMI provides important data for statistical tables that measure the average human." D) "The BMI is used to determine a person's nutritional status."

D) The BMI is used to determine a person's nutritional status." Rationale: Body mass index​ (BMI) is a calculation used to classify​ individuals' nutritional status as​ underweight, within normal​ limits, or​ overweight/obese. A​ person's BMI is neither used to predict the development of diseases like diabetes or heart disease nor is it used to explain an​ individual's development according to age. Its data are not used in statistical tables.

Which is the formula for calculating a patient's BMI? A- [(weight in pounds/(height in inches X height in inches)] X 7003 B- [(weight in pounds/(height in inches X height in inches)] X 73 C- [(weight in pounds/(height in feet X height in feet)] X 703 D- [(weight in pounds/(height in inches X height in inches)] X 703

D- [(weight in pounds/(height in inches X height in inches)] X 703

A pregnant client tells the nurse that she has no idea how she is going to provide food for her to preschool-aged children in addition to the new baby. Which intervention should the nurse Implement? Select all that apply. A- Review the importance of adequate nutrition during pregnancy B- encourage the client to obtain additional employment C- discuss the financial support provided by the children's father D- recommend the women, infants, and children (wic) program E- provide information about the supplemental nutrition assistance program (snap)

D- recommend the women, infants, and children (wic) program E- provide information about the supplemental nutrition assistance program (snap)

A nurse is providing discharge instructions for a client who has been diagnosed with angina. The nurse educates the client on aerobic​ exercise, as per the healthcare​ provider's orders. The client asks the​ nurse, "How will I know when I am working out​ adequately?" Which is the correct response from the​ nurse? A. "The goal for aerobic exercise is to maintain target heart rate within 60-​85% of resting heart​ rate." B. "The goal for aerobic exercise is to maintain target heart rate within 40-​55% of maximum heart​ rate." C. "When you are performing aerobic​ exercise, you​ shouldn't be able to carry on a​ conversation." D. ​"When you are performing aerobic​ exercise, you should be able to carry on a conversation without labored​ breathing."

D."When you are performing aerobic​ exercise, you should be able to carry on a conversation without labored​ breathing." ​Rationale: The goal for aerobic exercise is to maintain a target heart rate within 60dash-​85% of the maximum heart​ rate, not 60dash-​85% of the resting heart rate or 40dash-​55% of the maximum heart rate.​ However, because heart rates vary among​individuals, the talk test​ (the ability to carry on a conversation while​ exercising) is being used to replace the target heart rate goal and is the best answer response.

"A frail elderly patient with a diagnosis of type 2 diabetes mellitus has been ill with pneumonia. The cliet's intake has been very poor, and she is admitted to the hospital for observation and management as needed. What is the most likely problem with this patient? "A. Insulin resistance has developed. B. Diabetic ketoacidosis is occuring. C. Hypoglycemia unawareness is developing. D. Hyperglycemic hyperosmolar non-ketotic coma.

D.Illness, especially with the frail elderly patient whose appetite is poor, can result in dehydration and HHNC. Insulin resisitance is inidcated by a daily insulin requirement of 200 units or more. Diabetic ketoacidosis, an acute metabolic condition, usually is caused by absent or markedly decreased amounts of insulin.

18. The benefits of using an insulin pump include all of the following except: "a. By continuously providing insulin they eliminate the need for injections of insulin b. They simplify management of blood sugar and often improve A1C c. They enable exercise without compensatory carbohydrate consumption d. They help with weight loss

D: Using an insulin pump has many advantages, including fewer dramatic swings in blood glucose levels, increased flexibility about diet, and improved accuracy of insulin doses and delivery; however, the use of an insulin pump has been associated with weight gain.

A nurse is caring for a client who has pancreatic cancer that is unresponsive to treatment . The client is experiencing significant weight loss and fatigue , but when the nurse asks how they are feeling , they respond with , " Great ! I'm going to beat this cancer . " Which of the following defense mechanisms is the client using ? Regression Projection Repression Denial

Denial Denial refers to the refusal to acknowledge or accept reality about a situation despite what might be obvious to others to avoid the emotional impact . repression refers to concealing unpleasant or painful thoughts, memories , or beliefs in hopes of forgetting about them entirely. Projection refers to attributing negative or uncomfortable thoughts feelings or motives

A nurse is instructing a group of clients regarding nutrition. Which of the following is a good source of omega-3 fatty acids that the nurse should include in the teaching? a. Fish b. Corn oil c. Dietary supplements d. Leafy green vegetables

Fish Fish is an excellent source of omega-3 fatty acids.

A nurse in a pediatric clinic is caring for a child who has iron deficiency anemia and a new prescription for ferrous sulfate tablets. Which of the following instructions should the nurse provide the parents regarding administration of this medication? a. Give with a 240 mL (8 oz) glass of milk. b. Administer at mealtimes. c. Give with orange juice. d. Administer at bedtime.

Give with orange juice. Citrus fruit or juice aids absorption of this medication.

A nurse is caring for a client whose partner was recently hospitalized with COVID - 19 . The client is experiencing manifestations related to the alarm stage of general adaptation syndrome ( GAS ) . For which of the following manifestations should the nurse monitor ? ( Select all that apply . ) Hypertension . Dilated pupils Increased state of arousal Bradycardia Lethargy

Hypertension . Dilated pupils Increased state of arousal during the alarm stage of GAS the clients fight or flight response is mobilized to meet the threat . the nurse should expect rising hormone levels during this phase to cause hypertension. Rising hormone levels during this phase cause dilated pupils as well as increased state of arousal. Horizing hormone levels during this alarm phase can cause an increase in heart rate and a heightened state of alertness

A nurse is conducting an educational session for clients who report experiencing stress - related disorders . A client asks the nurse which part of the body activates the stress response . Which of the following responses should the nurse provide ? Sympathetic nervous system ( SNS ) Adrenal glands Hypothalamus Adrenocorticotropic hormone

Hypothalamus Hypothalamus activates the stress response . when a stress response is triggered The hypothalamus sends signals to the pituitary and adrenal glands The adrenal glands are involved in the stress response however the hypothalamus activates this response

A nurse is providing information about pain control for a client who has acute pain following a subtotal gastric resection. Which of the following client statements indicates an understanding of pain control? a)I will call for pain medication before the previous dose wears off b)I will call for pain medication as my pain starts to increase again c)I will wait for you to evaluate my pain before asking for more medication d)I will ask for less medication to avoid addiction

I will call for pain medication before the previous dose wears off

A nurse is assessing a client who was sexually assaulted 6 months ago and has been diagnosed with post traumatic stress disorder ( PTSD ) . Which of the following manifestations should the nurse expect ? ( Select all that apply . ) Intrusive memories of the event Flashbacks of the event Poor work relationships Exaggerated startle response when reminded of the event Frequent episodes of diarrhea

Intrusive memories of the event Flashbacks of the event Exaggerated startle response when reminded of the event

A nurse is teaching a client who is obese about orlistat. The nurse should instruct the client to report which of the following findings as an adverse effect of the medication? a. Drowsiness b. Constipation c. Oily fecal spotting d. Dark-colored stools

Oily fecal spotting Oily fecal spotting is an adverse effect of orlistat, because of the GI tract's decreased absorption of fat.

A nurse is completing an admission assessment on an adolescent client who is a vegetarian. He eats milk products but does not like beans. Which of the following items should the nurse suggest the client order for lunch to provide the nutrients most likely to be lacking in his diet? a. Peanut butter and jelly sandwich b. Baked potato topped with sour cream c. Bagel with cream cheese d. Fruit salad

Peanut butter and jelly sandwich A vegetarian diet may be low in protein, especially if the client does not substitute protein-rich beans for meat protein. Peanut butter is an excellent source of protein. A peanut butter and jelly sandwich, especially if prepared on protein-enriched bread, can provide almost 20 grams of protein.

A nurse is talking about implementing self - care strategies to cope with the stress of caregiving with the partner of a client who has dementia . Which of the following strategies reported by the partner should the nurse identify as an example of effective coping ? A. Practicing deep breathing while sitting outside B. Sitting by the client's bedside and drinking coffee C. Going out onto the patio to smoke a cigarette when feeling stressed D. Drinking a glass of wine every night before falling asleep .

Practicing deep breathing while sitting outside Eating nutritious meals participating in active exercise and engaging in mindfulness activities such as deep breathing are examples of healthy coping

A nurse is caring for a client who has been charged with partner violence against their spouse . The client is angry , pacing , and yells out , " I wouldn't lose my temper if my spouse would just leave me alone . It's their fault . " The nurse should identify the client is displaying which of the following defense mechanisms ? Projection Compartmentalization Repression Regression

Projection Projection involves attributing negative or uncomfortable thoughts feelings or motives unto another individual to avoid dealing with them as ones own.

A nurse is creating a dietary plan for an adult female client who has a hemoglobin level of 9.8 g/dL. Which of the following foods should the nurse recommend? a. Carrots b. Raisins c. Maple syrup d. Orange juice

Raisins Foods high in iron are recommended to improve a low hemoglobin level. Raisins are a high source of iron.

A nurse is planning a diet for a client who is iron deficient. Which of the following foods high in iron should the nurse include in the plan? a. Oranges b. Cashews c. Red meat d. Yogurt

Red meat Red meat is a good source of iron. If the client is vegetarian, kidney beans with a high iron content are a good substitute.

A nurse is instructing a group of clients regarding nutrition. The teaching should state that which of the following groups of foods contains the highest level of carbohydrates? a. Milk, eggs, and cheese b. Butter, oils, and avocados c. Rice, potatoes, and oranges d. Chicken, green beans, and apples

Rice, potatoes, and oranges This group of foods contains the highest level of carbohydrates.

A nurse is providing care for a client who is 2 days postoperative following abdominal surgery and is about to progress from a clear liquid diet to full liquids. Which of the following items should the nurse tell the client he may now request to have on his meal tray? a. Cranberry juice b. Flavored gelatin c. Skim milk d. Chicken broth

Skim milk Full liquids include milk and milk products, so the client may now ask for skim milk.

Risk factors for type 2 diabetes include all of the following except: a. Advanced age b. Obesity c. Smoking d. Physical inactivity"

Smoking "Additional risk factors for type 2 diabetes are a family history of diabetes, impaired glucose metabolism, history of gestational diabetes, and race/ethnicity. African-Americans, Hispanics/Latinos, Asian Americans, Native Hawaiians, Pacific Islanders, and Native Americans are at greater risk of developing diabetes than whites."

A nurse is caring for a client who reports experiencing stress over an upcoming surgical procedure . Which of the following statements describes the characteristics of stress ? Stress is an easily defined phenomenon regardless of viewpoint and discipline . Stress is a condition in which the body responds to physical , emotional , or environmental changes affecting one's state of equilibrium . Stress only affects the individual and does not affect the person's family , friends , or other associates . The lack of definition regarding stress does not pose a problem for the client or the nurse .

Stress is a condition in which the body responds to physical , emotional , or environmental changes affecting one's state of equilibrium . Stress can be caused by physical emotional environmental or mental or mental Changes which can be positive or negative depending on the persons perception or appraisal of the stressor.

A nurse is planning care for a client who is recently divorced with two young children . The client reports difficulty sleeping , feeling hopeless, and being estranged from family . The nurse should plan to monitor the client for which of the following potential manifestations of chronic stress ? Systemic infection Exaggerated startle response Recurring nightmares Suicide

Suicide Chronic stress can cause or exacerbate serious health problems such as depression , anxiety , cancer , and suicide

A nurse is teaching a client who has iron deficiency anemia about ferrous sulfate. Which of the following instructions should the nurse include in the teaching? a. Take the ferrous sulfate at bedtime. b. Take the ferrous sulfate with an antacid. c. Take the ferrous sulfate between meals. d. Take the ferrous sulfate with yogurt.

Take the ferrous sulfate between meals. The client should take the medication between meals for optimal absorption.

A nurse is reviewing the laboratory results of a client who has a pressure ulcer. The nurse should identify an evaluation in which of the following laboratory values as an indication that the client has developed an infection? a)BUN b)potassium c)RBC count d)WBC count

WBC count

The nurse is caring for a patient newly diagnosed with celiac disease. When developing a teaching plan, which foods should the nurse include as one to avoid? Select all that apply. A- Eggs B- peanuts C- wheat D- Rye E- fruit

Wheat and Rye

A nurse teaches clients about the International Self-Care Foundation's seven pillars of self-care. Which of the following client statements indicates an understanding of the teaching? a. "I will perform moderate exercise several times each week." b. "I will wash my hands for 10 seconds before I eat." c. "I will increase my dietary intake of simple sugars." d. "I will limit my sun exposure to 1 hour in the middle of the day."

a

A nurse is caring for a client who has type 1 diabetes mellitus and is in need of a long-acting insulin preparation. The nurse anticipates receiving a prescription for which of the following insulins? a. insulin glargine b. insulin aspart c. insulin glulisine d. insulin lispro

a Long-acting insulin, such as insulin glargine, is intended to provide basal glucose control. The dosage is typically once daily at the same time each day.

A nurse is teaching a class about searching the internet for research articles to use for evidence-based changes. Which of the following information should the nurse include? a) "Use Quackwatch to verify credible websites." b) "Avoid using websites with a .gov domain for research articles." c) "Read the abstract to understand each step of the research study." d) "Use non-peer-reviewed articles to research data."

a) "Use Quackwatch to verify credible websites." - Quackwatch is a website that monitors internet websites for misinformation and credibility. The nurse should instruct students to use resources such as Quackwatch to verify health care information.

A nurse is teaching a class about citing references to use for evidence-based changes. Which of the following requires citing a reference to avoid plagiarism? (Select all that apply.) a) Direct quotations from a journal article b) A photograph depicting equipment c) Ideas published in a theoretical paper d) A personal website used to initiate a PICOT question e) Paraphrased sentence from an article

a) Direct quotations from a journal article b) A photograph depicting equipment c) Ideas published in a theoretical paper e) Paraphrased sentence from an article - Credit should be given to the original author for every outside source used to avoid plagiarism.

A nurse at a hospital is interviewing a newly admitted client. The client tells the nurse they would like to adhere to their cultural beliefs during their hospitalization. Which of the following actions should the nurse take to provide the client with culturally competent care? (Select all that apply.) a. Listen to the client's request with respect and compassion. b. Let the client know that their cultural beliefs will be checked on after the assigned nursing break. c. Reassure the client that they can practice their cultural beliefs if safe to self and others. d. Explain to the client that this is not possible in a public space. e. Provide resources to meet the client's cultural needs.

a, c, e

A nurse is admitting a client who reports anorexia and is experiencing malnutrition. Which of the following laboratory findings should the nurse expect to be altered? a)creatinine kinase b)troponin c)total bilirubin d)albumin

albumin

A nurse is collaborating with an interdisciplinary team to develop a plan of care for a client. Which of the following tools should the nurse use to guide the delivery of care? a) A PICOT question b) Critical pathway c) Qualitative research d) Hierarchy of evidence

b) Critical pathway - A critical pathway is a guide used to standardize a client's medication, exams, procedures, and rehabilitation during acute care through discharge home. Critical pathways improve communication among members of the interdisciplinary team, minimize treatment delays and complications, and reduce lengths of stay and treatment costs. They provide evidence for comprehensive treatment.

A nurse is teaching a newly licensed nurse about health literacy. Which of the following information should the nurse include? a. The client's signature on the discharge instructions ensures they understood the instructions. b. Clients who can read and write demonstrate health literacy. c. A client's comprehension of education can be affected by low health literacy. d. Health literacy affects only a small portion of clients who are cared for within the health care system.

c

The family of the client share with the nurse that the client has always been a hoarder, but since their father died, the behavior has gotten to the point where the home is no longer safe. Which classification of disorders related to stress and copying does the client behavior fit? a. depression and depression related disorders b. anxiety disorder c. obsessive compulsive disorder d. trauma and stressor related disorders

c

A college student present to the counseling center because they are having difficulty dealing with having been informed that, due to continued poor grades, they are being dismissed from the college. Which statement by the student to the nurse counselor should indicate the use of the ego defense mechanism projection? a. its really not all my fault. The major I was in and the number of credit hours I was taking would be difficult for anyone to be successful. b. while I'm very upset and my parents will be angry. I can just transfer to another school and still get my degree. c. I don't think that all of the grades are accurate, if they aren't, then I can stay in school. d. if my instructor had let me take an exam I missed when I was sick, I wouldn't be in this situation.

d

The client contacts the nurse asking to see the healthcare provider due to fears that they have " something terribly wrong" This client frequently professes the same fear that something is wrong in one body system or another, and has had multiple types of diagnostic over the years. The client also reports vague symptoms such as difficulty sleeping, headache, muscle tension, and digestive issues. Which anxiety disorder should the nurse most likely expect that the client is experiencing? a. phobias b. panic disorder c. obsessive compulsive disorder d. generalized anxiety

d

The nurse presents information about the "fear worry center" in the brain. Which information should the nurse include about how the fear center affects the risk for anxiety disorders? a. perfusion balances the risk for anxiety disorders b. oxygenation decreases the risk for anxiety disorders c. hormone secretion blocks the risk for anxiety disorders d. hypersensitivity increases the risk for anxiety disorders.

d

A nurse is teaching a client who has a new diagnosis of type 2 diabetes mellitus about metformin. The nurse should explain that this type of medication works by which of the following mechanisms? a. increasing insulin secretion by the pancreas b. delaying carbohydrate digestion c. increasing the cellular response to insulin d. reducing hepatic glucose production

d Biguanides reduce hepatic glucose production while increasing insulin action on muscle glucose uptake.

A nurse is reviewing self-administration of insulin using a pre-filled pen with a client who started using the pen the previous week. The client asks what can be done to help reduce injection pain. Which of the following instructions should the nurse give the client? a. Agitate the syringe slightly before injection. b. Store the pens with the needle pointing upward. c. Inserting the needle rapidly minimizes injection pain. d. Keep the pen at room temperature for a few minutes.

d Injecting room-temperature insulin is less painful than injecting cold insulin.

A nurse is teaching a newly licensed nurse about using credible databases for research studies. Which of the following sources should the nurse include? a) An internet website with a .com domain b) A local newspaper repository c) A personal podcast d) A peer-reviewed journal

d) A peer-reviewed journal - A peer-reviewed journal is a credible, scientific database of scholarly published scientific research. It is reviewed by expert peers to ensure credibility.

A nurse is collecting data from a group of clients regarding their feelings and experiences of living with congestive heart failure. In which of the following types of research studies is the nurse participating in? a) Translational research b) Quantitative c) Mixed methods d) Qualitative

d) Qualitative - Qualitative research is a research method that explores experiences and feelings from the perspective of the client. Data obtained from qualitative research do not undergo statistical analysis that is associated with other types of research.

A nurse has created a table of evidence to organize findings from a literature search. Which of the following components of the table provides information for implementing a change in practice? a) Title b) Author c) Research method d) Recommendations

d) Recommendations - The recommendations component of the table of evidence provides information for implementing a change in practice. Recommendations are based on research findings to improve client outcomes.

When taking a health history, the nurse screens for manifestations suggestive of diabetes type I. Which of the following manifestations are considered the primary manifestations of diabetes type I and would be most suggestive of diabetes type I and require follow-up investigation? a. Excessive intake of calories, rapid weight gain, and difficulty losing weight b. Poor circulation, wound healing, and leg ulcers, c. Lack of energy, weight gain, and depression d. An increase in three areas: thirst, intake of fluids, and hunger

d. An increase in three areas: thirst, intake of fluids, and hunger "The primary manifestations of diabetes type I are polyuria (increased urine output), polydipsia (increased thirst), polyphagia (increased hunger). Excessive calorie intake, weight gain, and difficulty losing weight are common risk factors for type 2 diabetes. Poor circulation, wound healing and leg ulcers are signs of chronic diabetes. Lack of energy, weight gain and depression are not necessarily indicative of any type of diabetes."

A nurse is preparing a client's evening dose of risperone hen the tablet falls on the countertop. Which of the following actions should the nurse take? a)use the tablets packaging to pick it up from the counter b)wash the tablet off with alcohol and place it in a clean medication cup c)discard the tablet and obtain another dose of medication d)place the tablet directly into a medication cup

discard the tablet and obtain another dose of medication

A nurse is developing a plan of care for a client who has cellulitis of the leg. Which of the following interventions should the nurse include in the plan? a)enforce strict bedrest for three days b)apply fresh ice packs every 4hrs c)elevate the affected leg on two pillows d)apply antibiotic ointment to the wound with dressing changes

elevate the affected leg on two pillows

A staff nurse has applied for a promotion. The hiring manager insinuates that if there was a sexual relationship between the two of them, the nurse's promotion request would get increased consideration. Which of the following actions should the staff nurse take first? a)tell the hiring manager in clear terms that this conduct causes feeling of discomfort and that the behavior should stop immediately. b)report the behavior to the nurse manager c)create a written document of the incident and store the document in a safe place. d)seek help from a trustworthy friend

ell the hiring manager in clear terms that this conduct causes feeling of discomfort and that the behavior should stop immediately.

A nurse is caring for a young adult client who says he is experiencing increased anxiety and an ability to concentrate. Which of the following responses should the nurse take? a)it should like your having a difficult time b)have you talked to your parents about this yet c)why do you think you are so anxious d)how long has this been going on

it should like your having a difficult time

A nurse is talking with a client who is about to start using transcutaneous electrical nerve stimulation(TENS) to manage chronic pain. which of the following statements should the nurse identify as an indication that the client needs further teaching? a)I wish I didn't have to attach the electrodes to my skin b)it's unfortunate that I have to be in the hospital for this treatment c)I'll need to shave the hair off the skin where I place the electrodes d)I hope I don't have to take as many pain pills

it's unfortunate that I have to be in the hospital for this treatment

A nurse is caring for an older adult client who states, "I am afraid I may fall while walking in the bathroom during the night." Which of the following actions should the nurse take? a)limit the clients fluid intake b)obtain a bedside commode for the client's use c)leave a nightlight on in the client's room d)put the side rails up and tell the client to call the nurse before voiding

leave a nightlight on in the client's room

A client smoking in his bathroom has dropped a cigarette butte into a wastepaper basket, which begins to smolder. Which of the following actions is the nurse's priority? a)close the fire doors on the unit b)activate the fire alarm c)move any clients in the immediate vicinity d)use a fire extinguisher to put out the fire

move any clients in the immediate vicinity

A nurse is teaching a client about nutritional requirements necessary to promote wound healing. Which if the following nutrients should the nurse include in the teaching? a)protein b)calcium c)vit B d)vit D

protein

A hospice nurse us caring for a client who has terminal cancer and takes PO morphine for pain relief. The client reports that he had to increase the dose of morphine this week to obtain relief. Which of the following scenarios should the nurse document as the explanation for this situation? a)the client not been taking the medication properly b)the client is experiencing episodes of confusion c)the client has become addicted to the medication d)the client developed a tolerance to the medication

the client developed a tolerance to the medication

A nurse at an extended-care facility is instructing a class of assistive personel (AP) about client use of assistive devices during ambulation. Which of the following instructions should the nurse give the APs about the clients use of a cane? a)when the client moves, he should move the cane forward first b)the grip should be level with the clients waist c)the client should first move the strong leg then the weak one d)the client should hold the cane on the weak side of the body

when the client moves, he should move the cane forward first

The nurse is caring for a child with type 2 diabetes mellitus. Which item in this​ child's history should the nurse recognize as a risk factor for this​ disease? (Select all that​ apply.) A. High-fat diet B. Race C. Sex D. Obesity E.Family history

​A,B,C,D Rationale: Obesity, a​ high-fat diet, a family history of diabetes​ mellitus, and race are risk factors for developing type 2 diabetes mellitus. Sex does not play a role in the risk for type 2 diabetes mellitus.

The nurse is teaching the caregivers of an adolescent with a new diagnosis of type 2 diabetes mellitus what they should do every 3 months to monitor the disease. The adolescent is currently taking metformin​ (Glucophage). Which information should the nurse​ include? (Select all that​ apply.) A. Discuss alcohol and drug use. B. Review blood glucose logs. C. Obtain an eye exam. D. Monitor hemoglobin A1C. E. Assess injection sites.

​A,B,D Rationale: An adolescent with type 2 diabetes mellitus who takes metformin​ (Glucophage) should monitor the hemoglobin A1C and blood glucose logs every 3 months. The nurse should also discuss alcohol and drug abuse and its effects on type 2 diabetes mellitus every 3 months. An eye exam should be obtained​ annually, not quarterly. The nurse need not assess injection sites because the adolescent does not use insulin.

In a professional development​ seminar, the educator shares barriers that they may encounter as they try to implement​ evidence-based practice​ (EBP) into their care. Which statement most accurately describes these potential​ barriers? (Select all that​ apply.) A. Client workload demands B. Nursing misperceptions about EBP C. Providing accountability for nursing care D. Ensuring credibility of the nursing profession E. Lack of access to continuing education programs

​A. Client workload demands B. Nursing misperceptions about EBP E. Lack of access to continuing education programs Rationale: Potential barriers to implementation of​ evidence-based practice​ (EBP) include lack of access to continuing education​ programs, nursing misperceptions about​ EBP, and client workload demands. Benefits of EBP include providing accountability for nursing care and ensuring credibility of the nursing profession.

The nurse is developing a teaching plan for carbohydrate counting for a client newly diagnosed with type 1 diabetes mellitus. Which type of carbohydrate should the nurse instruct the client to​ restrict? A. Complex carbohydrates B. Dietary fructose C. Refined sugars D. Simple sugars

​C. Refined sugars Rationale: Refined sugars come from sugar cane and are used as natural sweeteners. The client should restrict the intake of refined sugars. Simple sugars are found in​ fruit, honey, and dairy products. Dietary​ fructose, which comes from dietary fruit and vegetable​ consumption, causes a slower rise in blood glucose levels. Complex carbohydrates come from​ peas, beans, whole​ grains, and vegetables.

The healthcare provider prescribes metformin​ (Glucophage) to a client with newly diagnosed type 2 diabetes mellitus. Which information should the nurse provide to the​ client? A. This medication is used for clients who are unable to inject insulin. B. This medication is only used in the adult population due to side effects. C. This medication can take up to 3 months to show effectiveness. D. This medication is unsafe for use by pregnant and lactating women.

​C. This medication can take up to 3 months to show effectiveness. Rationale: Metformin​ (Glucophage) is a relatively safe medication to use in the treatment of type 2 diabetes mellitus.​ However, it may take up to 3 months to show effectiveness. Metformin is used to stimulate insulin​ production, not used in place of insulin. It is safe for pregnant and lactating women and for children.

The nurse is designing a nursing clinical research study. Which research question is most appropriate for this type of​ study?(Select all that​ apply.) A. What factors influence an​ individual's choice to enroll in online nursing​ education? B.How do communication styles influence the​ nurse's perception of nurse​ educators? C. How do environmental noise levels affect the​ client's pain perception​ postanesthesia? D. Is there a relationship between acupressure and incidence of nausea in clients who are receiving chemotherapy​ medications? E.What is the relationship between the duration of​ nurse-client interaction and the​ client's level of satisfaction with nursing​ care?

​C.How do environmental noise levels affect the​ client's pain perception​ postanesthesia? D. Is there a relationship between acupressure and incidence of nausea in clients who are receiving chemotherapy​ medications? E.What is the relationship between the duration of​ nurse-client interaction and the​ client's level of satisfaction with nursing​ care? Rationale: Nursing research is a systematic and strict scientific process that tests hypotheses about​ health-related conditions and nursing​ care; as an extension of nursing​ research, nursing clinical research seeks answers that will ultimately improve client care.

The nurse researcher is encouraging a nurse to incorporate​ evidence-based practice​ (EBP) into client care. Which action should the researcher encourage the nurse to​ do? A. Reviewing a​ client's record B. Looking at past data C. Participating in a research project D. Speaking with the hospitalist

​C.Participating in a research project Rationale: It would be beneficial for the graduate nurse to participate in a research project to be a part of an EBP​ process, so they could see firsthand how the process works in action. Reviewing a​ client's chart may be a way to look at what treatment was​ performed, but it would not reveal if it was evidence based. Looking at past data will not provide a look at​ evidence-based practice. A hospitalist may be able to share their opinion of​ evidence-based practice, but that does not expose the nurse to the process.

A client newly diagnosed with type 2 diabetes mellitus asks the nurse how to​ "get rid​ of" this disease. How should the nurse​ respond? A. ​"Type 2 diabetes mellitus cannot be cured. It will eventually progress to type 1​ diabetes." B. "You seem concerned about this diagnosis and we will do our best to help you control​ it." C. "You will always have type 2 diabetes mellitus. You cannot get rid of​ it." D. "Type 2 diabetes mellitus can sometimes be eliminated by weight​ loss, diet, and​ exercise."

​D. "Type 2 diabetes mellitus can sometimes be eliminated by weight​ loss, diet, and​ exercise." Rationale: Type 2 diabetes occurs in people who live a sedentary​ lifestyle, are​ obese, and eat a​ high-carbohydrate diet.​ Therefore, the nurse would explain to the client that the disease may be eliminated with​ diet, exercise, and weight loss. The first statement stating the client will always have type 2 diabetes mellitus is inaccurate because the disease can be eliminated. Type 1 and type 2 diabetes mellitus are two separate disorders with commonalities. One type does not progress to the other. Although it is accurate that the client is concerned about the​ diagnosis, this response does not answer the​ client's question.

The charge nurse is providing an educational seminar for a group of nurses. Which factor should the nurse expect to be discussed as an​ evidence-based practice topic for older adults that correlates with Healthy People 2020​? A. Screening protocols for sickle cell B. Transmission of infectious disease during travel C. Obesity prevention D. Fall-related injuries

​D. Fall-related injuries Rationale: For older​ adults, Healthy People 2020 objectives address topics including increasing the number of individuals who are up to date on basic preventive care and decreasing the incidence of health alterations such as pressure ulcers and fall related injuries. For the general care of​ adults, goals associated with the promotion of global health include protecting the health of the national population as well as limiting the international transmission of infectious diseases during travel.​ Evidence-based resources related to newborns and infants address topics such as screening protocols for phenylketonuria and sickle cell disease.


Related study sets

Chapter 55: Management of Patients With Urinary Disorders H & I

View Set

Chap. 2: Prenatal Development and Prenatal Care

View Set

Chapter 8 Setting up the physical Environment

View Set

Chapter 8 (Social Disorganization Crim 2331)

View Set

PMK-EE E4: Warfighting and Readiness

View Set

Concept Covered: Electron Transport Chain, The Mitochondria, Overview of Respiration, Krebs Cycle, Glycolysis

View Set